ORTHOPEDIC MCQS BANK OITE 96
ORTHOPEDIC MCQS BANK OITE 96
- 96.1 A 4‑year‑old girl has had increasing left hip pain over the past 48 hours and has
- stopped walking. Her temperature is 38°C (100.4°F). Hip range of motion is
- limited. Aspiration of the hip reveals a WBC of 68,000/mm' and rare
- Gram‑positive cocci. The next step in management should be
- 1‑ multiple aspirations and lavage.
- 2‑ antibiotics after culture results.
- 3‑ bone scan.
- 4‑ operative joint drainage.
- 5‑ bed rest and observation.
- Question 96.1
- Answer = 4
- Reference(s)
- Frymoyer JW (ed): Orthopaedic Knowledge Update 4. Rosemont, IL, American Academy of Orthopaedic Surgeons, 1993, pp 505‑513. Nord KD, Dore DD, Deeney VF, et al: Evaluation of treatment modalities for septic arthritis with histological grading and analysis of levels of uronic acid, neutral protease, and interleukin‑1. J Bone Joint Surg 1995;77A:258‑265.
- 96.2 A 59‑year‑old woman has disabling pain at the base of her thumb. A
- radiograph is shown in Figure 1. What is the best surgical option for this
- patient?
- 1‑ Metacarpal abduction osteotomy
- 2‑ Trapeziometacarpal arthrodesis
- 3‑ Partial trapeziectomy and silicone condylar implant arthroplasty
- 4‑ Trapeziectomy and silicone trapezial implant arthroplasty
- 5‑ Trapeziectomy and tendon interposition arthroplasty
- Figure 1
- back
- Question 96.2
- Answer = 5
- Reference(s)
- Kleinman WB, Eckenrode JF: Tendon suspension sling arthroplasty for thumb trapeziometacarpal arthritis. J Hand Surg 1991;16:983‑991. Burton RI, Pellegrini VD Jr: Surgical management of basal joint arthritis of the thumb: Ligament reconstruction with tendon interposition arthroplasty. J Hand Surg 1986;11A:324‑332.
- 96.3 A 2‑year‑old boy with a supracondylar fracture of the distal humerus is treated
- with closed reduction and splinting. Postreduction radiographs are shown in
- Figures 2a and 2b. The physician should now recommend
- 1‑ casting for 3 weeks.
- 2‑ casting for 6 weeks.
- 3‑ repeating the closed reduction, followed by percutaneous pinning.
- 4‑ open reduction and pin fixation.
- 5‑ skin traction for 10 days, then repeating the closed reduction and casting.
- Fig. 2a
- Fig. 2b
- back
- Question 96.3
- Answer = 3
- Reference(s)
- Boyd DW, Aronson DD: Supracondylar fractures of the humerus: A prospective study of percutaneous pinning. J Pediatr Orthop 1992;12:789‑794. Pirone AM, Graham HK, Krajbich JI: Management of displaced extension‑type supracondylar fractures of the humerus in children. J Bone Joint Surg 1988;70A:641‑650.
- 96.4 What muscles are innervated by the superior gluteal nerve?
- 1‑ Piriformis, gluteus medius, and gluteus minimus
- 2‑ Piriformis, gluteus maximus, and gluteus medius
- 3‑ Gluteus maximus, gluteus medius, and gluteus minimus
- 4‑ Tensor fascia lata, gluteus medius, and gluteus maximus
- 5‑ Tensor fascia lata, gluteus medius, and gluteus minimus
- back
- Question 96.4
- Answer = 5
- Reference(s)
- Buttock, Hip Joint, and Thigh, in Hollinshead WH (ed): Anatomy for Surgeons: Back and Limbs. New York, NY, Harper & Row, 1969, p 686.
- 96.5 A 25‑year‑old man who abuses drugs develops a soft‑tissue abscess over his forearm following an injection. The bacterial organisms most likely to cause an infection (either monomicrobial or polymicrobial) in this scenario are
- 1‑ Streptococcus and Eikenella corrodens.
- 2‑ Streptococcus and Staphylococcus.
- 3‑ Staphylococcus and Serratia.
- 4‑ Staphylococcus and diphtheroids.
- 5‑ Staphylococcus and Mebsiella.
- back
- Question 96.5
- Answer = 2
- Reference(s)
- Schnall SB, Holtom PD, Lilley JC: Abscesses secondary to parenteral abuse of drugs: A study of demographic and bacteriological characteristics. J Bone Joint Surg 1994;76A:1526‑1530.
- 96.6 Cephalosporin antibiotics are effective against bacteria by inhibiting what
- mechanism?
- 1‑ Production of DNA gyrase
- 2‑ Peptidoglycan synthesis
- 3‑ RNA synthesis
- 4‑ Ribosomal protein synthesis
- 5‑ Spheroplast growth
- Question 96.6
- Answer = 2
- Reference(s)
- Mandell GL, Sande MA: Antimicrobial agents, in Gilman AG, Goodman LS, Gilman A (eds): Goodman and Gilman's The Pharmacological Basis of Therapeutics, ed 6. New York, NY, MacMillan Publishing Co, 1980, pp 11261161.
- 96.7 During a posterior approach to the hip joint, profuse bleeding is encountered
- during incision of the quadratus femoris. The bleeding is most likely from which
- artery?
- 1‑ Superior gluteal
- 2‑ Inferior gluteal
- 3‑ Lateral femoral circumflex
- 4‑ Medial femoral circumflex
- 5‑ Posterior femoral circumflex
- Question 96.7
- Answer = 4
- Reference(s)
- Hoppenfeld S, de Boer P: The hip, in Hoppenfeld S, deBoer P (eds): Surgical Exposures in Orthopaedics: The Anatomic Approach. Philadelphia, PA, JB Lippincott, 1984, pp 301‑356.
- 96.8 Which of the following best describes the relationship of the tibia to the femur
- during a positive reverse pivot‑shift test?
- 1‑ Tibia reduced with knee flexion and subluxates posteriorly in knee extension
- 2‑ Tibia reduced with knee flexion and subluxates anteriorly in knee extension
- 3‑ Tibia reduced with knee flexion and fibula subluxates posteriorly in knee
- extension
- 4‑ Tibia subluxated posteriorly with knee flexion and reduces in knee extension
- 5‑ Tibia subluxated anteriorly with knee flexion and reduces in knee extension
- answer
- back
- Question 96.8
- Answer = 4
- back to this question
- next question
- Reference(s)
- Jakob RP, Hassler H, Staeubli HU: Observations on rotatory instability of the lateral compartment of the knee: Experimental studies on the functional anatomy and the pathomechanism of the true and the reversed pivot shift sign. Acta Orthop Scand Suppl 1981;191:1‑32.
- 96.9 Which of the following conditions produces the greatest reduction in the
- torsional strength of a tubular bone?
- 1‑ Circular defect of one half of the bone diameter
- 2‑ Square defect of one fourth of the bone diameter
- 3‑ Square defect of one half of the bone diameter
- 4‑ 50% reduction of the endosteal bone thickness
- 5‑ Narrow longitudinal defect equal to the bone diameter in length
- back
- Question 96.9
- Answer = 5*
- Reference(s)
- Chao EYC, Aro HT: Biomechanics of fracture fixation, in Mow VC, Hayes WC (eds): Basic Orthopaedic Biomechanics. New York, NY, Raven Press, 1991, pp 293‑336. Frankel VH, Burstein AH: Orthopaedic Biomechanics. Philadelphia, PA, Lea & Febiger,1970,pp 293‑336. *item deleted after statistical review
- 96.10 A 30‑year‑old man who has weakness, deformity, and pain on rotation due to
- malunion of a distal radial fracture is undergoing corrective osteotomy. A dorsal
- opening wedge osteotomy with iliac crest graft has restored radial length and
- volar tilt, but there is now a marked reduction in passive pronation and
- supination of the forearm. The next step in treatment is to
- 1‑ revise the correction of the radius until rotation improves.
- 2‑ add an ulnar shortening osteotomy.
- 3‑ perform a distal ulna excision.
- 4‑ perform a hemiresection arthroplasty of the distal radioulnar joint.
- 5‑ perform a distal radioulnar joint arthrodesis with proximal pseudarthrosis of
- the ulna.
- Question 96.10
- Answer = 4
- Reference(s)
- Fernandez DL: Malunion of the distal radius: Current approach to management, in Heckman JD (ed): Instructional Course Lectures 42. Rosemont, IL, American Academy of Orthopaedic Surgeons, 1993, pp 99‑113. Fernandez DL: Radial osteotomy and Bowers arthroplasty for malunited fractures of the distal end of the radius. J Bone Joint Surg 1988;70A:1538‑1551.
- 96.11 Spinal cord injury most commonly occurs after what type of cervical injury?
- 1‑ Jefferson fracture
- 2‑ Hangman's fracture
- 3‑ Odontoid fracture
- 4‑ Clayshoveler's fracture
- 5‑ Bilateral facet dislocation
- answer
- back
- Question 96.11
- Answer = 5
- back to this question
- next question
- Reference(s)
- Bohlman HH, Ducker TB, Lucas JT: Spine and spinal cord injuries, in Rothman RH, Simeone FA (eds): The Spine. 1982, pp 661‑756.
- 96.12 What is the major cause of failure of a cemented total hip arthroplasty after 15
- to 20 years?
- 1‑ Femoral loosening
- 2‑ Acetabular loosening
- 3‑ Bone stock loss
- 4‑ Wear of the bearing surface
- 5‑ Stem breakage
- Question 96.12
- Answer = 4*
- Reference(s)
- Poss R, Brick GW, Wright RJ, et al: The effects of modern cementing techniques on the longevity of total hip arthroplasty. Orthop Clin North Am 1988;19:591‑598. *item deleted after statistical review
- 96.13 A 35‑year‑old man has a closed comminuted distal humeral fracture after falling
- from a roof at work. One year later, he has 35 to 85 degrees of elbow flexion and 0
- to 30 degrees of forearm supination. Which of the following defines the limitation
- in his upper extremity?
- 1‑ Temporary disability
- 2‑ Permanent disability
- 3‑ Impairment
- 4‑ Handicap
- 5‑ Work capacity assessment
- back
- Question 96.13
- Answer = 2
- Reference(s)
- Andriacchi TP: Functional analysis of pre and post‑knee surgery: Total knee arthroplasty and ACL reconstruction. J Biomech Eng 1993;115:575‑581. Kasser JR (ed): Orthopaedic Knowledge Update 5. Rosemont, IL, American Academy of Orthopaedic Surgeons, 1996, pp 29‑40.
- 96. 14 An 18‑month‑old boy who was an unrestrained passenger in a motor vehicle accident has
- the injury shown in Figure 3. He is neurologically normal. The physician should recommend
- 1‑ bed rest and halter traction for 6 weeks.
- 2‑ posterior atlantoaxial arthrodesis with wire fixation.
- 3‑ Philadelphia collar.
- 4‑ reduction and Minerva cast immobilization.
- 5‑ reduction and anterior internal fixation with an odontoid screw.
- Fig. 3
- answer
- back
- Question 96.14
- Answer = 4
- back to this question
- next question
- Reference(s)
- Evans DL, Bethem D: Cervical spine injuries in childhood. J Pediatr Orthop 1989;9:563‑568.
- 96.15 A 15‑year‑old boy has had pain in his left knee for the past 9 months, and over the
- last 2 months, he has noticed a slowly enlarging firm mass in his left thigh.
- Examination confirms a firm, immobile mass deep in the left midthigh. His
- erythrocyte sedimentation rate is 22 mm/hr (normal up to 20 mm/hr). A lateral
- femur radiograph is shown in Figure 4a. MRI scans, including a sagittal
- T2‑weighted image and an axial T1 weighted image are shown in Figures 4b and 4c.
- Hematoxylin and eosin staining of tissue from an incisional biopsy is shown in
- Figure 4d. A bone scan and chest CT scan show no other areas of involvement.
- The most likely diagnosis is
- 1‑ osteosarcoma.
- 2‑ chondrosarcoma.
- 3‑ Ewing sarcoma.
- 4‑ chronic osteomyelitis.
- 5‑ eosinophilic granuloma.
- Go to next slide for figures and answer link
- back
- Fig. 4a Fig. 4b
- Fig. 4c
- Fig. 4d
- Question 96.15
- Answer = 3
- Reference(s)
- Ewing's sarcoma in Huvos AG (ed): Bone Tumors: Diagnosis, Treatment, and Prognosis, ed 2. Philadelphia, PA, WB Saunders, 1991, pp 523‑552. Wold LE, McLeod RA, Sim FH, et al: Atlas of Orthopaedic Pathology. Philadelphia, PA, WB Saunders, 1990,pp 210‑215.
- 96.16 What is the preferred initial treament of a 5‑year‑old boy with the fracture shown
- in Figure 5?
- 1‑ Closed reduction using extension and cast immobilization
- 2‑ Closed reduction using pronation and cast immobilization
- 3‑ Closed reduction of the ulna and transarticular pin fixation of the radius
- 4‑ Open reduction and intramedullary rod fixation of the ulna
- 5‑ Open reduction and plate fixation of the ulna
- Fig. 5
- Question 96.16
- Answer = 1
- Reference(s)
- Bado JL: The Monteggia lesion. Clin Orthop 1967;50:71‑86.
- 96.17 Which of the following terms refers to a moment created by two equal,
- noncolinear, parallel but oppositely directed forces, such as when a bottle cap is
- opened with the thumb and index finger?
- 1‑ Couple
- 2‑ Resultant
- 3‑ Tensor
- 4‑ Shear force
- 5‑ Shear vector
- Question 96.17
- Answer = 1
- Reference(s)
- Simon SR (ed): Orthopaedic Basic Science. Rosemont,1L, American Academy of Orthopaedic Surgeons, 1994, pp 397‑446.
- 96.18 Low‑dose warfarin given to a patient with deep vein thrombosis acts principally
- to
- 1‑ bind to antithrombin lII.
- 2‑ bind to factors Xa and thrombin factors.
- 3‑ interfere with the metabolism of factors II, VII, XI, and X.
- 4‑ reduce endothelial damage.
- 5‑ reduce platelet aggregation.
- Question 96.18
- Answer = 3*
- Reference(s)
- Simon SR (ed): Orthopaedic Basic Science. Rosemont, IL, American Academy of Orthopaedic Surgeons, 1994, pp487‑517. *item deleted after statistical review
- 96.19 A 25‑year‑old man collides with the sideboards while playing ice hockey and
- sustains a T10 burst fracture with 25% anterior wedging. He is neurologically
- intact and a CT scan reveals 20% canal compromise. Recommended treatment
- should include
- 1‑ bed rest for 2 weeks.
- 2‑ immobilization with a brace.
- 3‑ anterior fusion of T7 to T12.
- 4‑ posterior fusion of T7 to T12.
- 5‑ anterior and posterior fusion of T7 to T12.
- Question 96.19
- Answer = 2
- Reference(s)
- Weinstein IN, Collalto P, Lehmann TR: Thoracolumbar burst fractures treated conservatively: A long‑term follow‑up. Spine 1988;139:33‑38. McEvoy RD, Bradford DS: The management of burst fractures of the thoracic and lumbar spine: Experience in 53 patients. Spine 1985;10:631‑637.
- 96.20 What is the most common location of nerve compression in a patient with
- radial tunnel syndrome?
- 1‑ Fibrous bands at the radial tunnel entrance
- 2‑ Fan‑shaped radial recurrent vessels
- 3‑ Thin, tendinous margin of the extensor carpi radialis brevis muscle
- 4‑ Fibrous, proximal border of the superficial belly of the supinator
- 5‑ Distal border of the superficial supinator
- Question 96.20
- Answer = 4
- Reference(s)
- Gelberman RH, Eaton R, Urbaniak JR: Peripheral nerve compression. J Bone Joint Surg 1993;75A:1854‑1878. Eversmann WW Jr‑. Entrapment and compression neuropathies, in Green DP (ed): Operative Hand Surgery, ed 2. New York, NY, Churchill‑Livingstone, 1988, pp 1423‑1478.
- 96.21 A 35‑year‑old man has persistent symptoms after a fall, despite nonoperative treatment. MRI scans are shown in Figures 6a and 6b. Based on the MRI scans, which of the following is the best description of the patient's symptoms?
- 1‑ Numbness on the lateral side of the foot, weakness of ankle plantar flexion, loss of ankle jerk
- 2‑ Numbness on the lateral side of the foot, weakness of the extensor hallucis longus, loss of knee jerk
- reflex
- 3- Numbness at the medial calf, weakness of ankle dorsiflexion, loss of knee jerk reflex
- 4‑ Numbness of the lateral calf, weakness of ankle plantar flexion, loss of knee jerk reflex
- 5‑ Numbness in the first dorsal web space, weakness of the extensor hallucis longus
- Fig. 6a Fig. 6b
- Question 96.21
- Answer = 1
- Reference(s)
- Hoppenfeld S: Physical Examination of the Spine and Extremities. New York, NY, Appleton‑Century‑Crofts, 1976.
- 96.22 For an all‑polyethylene tibial component, what is considered to be adequate
- thickness (in mm) of the polyethylene needed to minimize wear?
- 1‑ 3
- 2‑ 5
- 3‑ 8
- 4‑ 11
- 5‑ 13
- Question 96.22
- Answer = 3
- Reference(s)
- Poss R (ed): Orthopaedic Knowledge Update 3. Park Ridge, IL, American Academy of Orthopaedic Surgeons, 1990, pp 185‑206.
- 96.23 What is the most common postoperative complication associated with
- adolescent hallux valgus?
- 1‑ Arthrofibrosis
- 2‑ Osteonecrosis
- 3‑ Infection
- 4‑ Loss of fixation
- 5‑ Deformity recurrence
- back
- Question 96.23
- Answer = 5
- Reference(s)
- Clark HR, Veith RG, Hansen ST Jr: Adolescent bunions treated by the modified Lapidus procedure. Bull Hosp J Dis 1987;47:109‑122. Coughlin MJ: Roger A Mann Award: Juvenile hallux valgus. Etiology and treatment. Foot Ankle Int 1995;16:682‑697.
- 96.24 A 13‑year‑old boy with sickle cell disease has left distal thigh pain. His
- temperature is 38.5°C (101.3 °F) and his WBC is 14,500/mm' (normal 3,500 to
- 10,500/mm'). Plain radiographs are unremarkable. The best initial management is
- 1‑ hydration.
- 2‑ administration of antibiotics.
- 3‑ administration of steroids.
- 4‑ aspiration of the distal femur.
- 5‑ an MRI scan.
- Question 96.24
- Answer = 1*
- Reference(s)
- Niemann KMW: Diseases related to the hematopoietic system, in Morrissy RT (ed): Lovell and Winter's Pediatric Orthopaedics, ed 3. Philadelphia, PA, JB Lippincott, 1990, vol 1, pp 263‑295. *item deleted after statistical review
- 96.25 A 35‑year‑old man falls from a height and lands on his outstretched hand. He now has elbow pain and swelling, and tenderness over the distal radioulnar joint. Radiographs reveal a moderately displaced fracture of the radial head involving two thirds of the articular surface, and a distal radioulnar dislocation. Initial management should be
- 1‑ a short period of immobilization, followed by active range of motion.
- 2‑ internal fixation of the radial head.
- 3‑ internal fixation of the radial head and distal radioulnar pinning.
- 4‑ prosthetic replacement of the radial head.
- 5‑ prosthetic replacement of the radial head with medial collateral ligament repair.
- answer
- back
- Question 96.25
- Answer = 3
- back to this question
- next question
- Reference(s)
- Frymoyer JW (ed): Orthopaedic Knowledge Update 4. Rosemont, IL, American Academy of Orthopaedic Surgeons, 1993, pp 319‑333. Edwards GS Jr, Jupiter JB: Radial head fractures with acute distal radioulnar dislocation: Essex‑Lopresti revisited. Clin Orthop 1988;234:61‑69.
- 96.26 A symptomatic patient with positive electrodiagnostic tests is about to undergo
- decompression of the posterior tibial nerve and the plantar branches. What
- muscle must be mobilized or divided to trace the plantar nerves?
- 1‑ Abductor hallucis
- 2‑ Adductor hallucis
- 3‑ Flexor hallucis brevis
- 4‑ Flexor digitorum brevis
- 5‑ Flexor accessorius
- Question 96.26
- Answer = 1
- Reference(s)
- Dellon AL, MacKinnon SE, Seiler WA IV: Susceptibility of the diabetic nerve to chronic compression. Ann Plast Surg 1988;20:117‑119. Havel PE, Ebraheim NA, Clark SE, et al: Tibial nerve branching in the tarsal tunnel. Foot Ankle Int 1988;9:117‑119. Tarsal tunnel syndromes, in Hallett M, Millender LH, Dawson DM, et al (eds): Entrapment Neuropathies, ed 2. Boston, MA, Little Brown & Co, 1990, pp 291‑305
- 96.27 The primary homeostatic regulators of serum calcium are parathyroid hormone
- and
- 1‑ 1, 25‑dihydroxyvitamin D.
- 2‑ 24‑, 25‑dihydroxyvitamin D.
- 3‑ vitamin D.
- 4‑ serum phosphate.
- 5‑ calcitonin.
- Question 96.27
- Answer = 1
- Reference(s)
- Mankin HJ: Metabolic bone disease, in Jackson DW (ed):Instructional Course Lectures 44. Rosemont, IL, American Academy of Orthopaedic Surgeons, 1995, pp 3‑29.
- 96.28 When is total knee arthroplasty a contraindication in a hemophilic patient?
- 1‑ With factor levels above 90%
- 2‑ In an ankylosed knee
- 3‑ In the presence of flexion contracture greater than 10 degrees
- 4‑ In the presence of an inhibitor
- 5‑ In the presence of a positive HIV test
- Question 96.28
- Answer = 4
- Reference(s)
- Figgie MP, Goldberg VM, Figgie HE III, et al: Total knee arthroplasty for the treatment of chronic hemophilic arthroplasty. Clin Orthop 1989;248:98‑107. Kjaersgaard‑Andersen P, Christiansen SE, Ingerslev J, et al: Total knee arthroplasty in classic hemophilia. Clin Orthop 1990;256:137‑146. Westring D, Ries M, Dee R: Hematologic disorders, in Dee R, Mango E, Hum LC (eds): Principles of Orthopaedic Practice. New York, NY, McGraw‑Hill, 1988, vol 1, pp 290‑294.
- 96.29 Patients with posterior interosseous nerve syndrome are most likely to have
- which of the following symptoms?
- 1‑ Pain at the lateral epicondyle
- 2‑ Motor weakness
- 3‑ Motor weakness and loss of sensation
- 4‑ Loss of sensation
- 5‑ Elbow pain and loss of sensation
- Question 96.29
- Answer = 2
- Reference(s)
- Gelberman RH, Eaton R, Urbaniak JR: Peripheral nerve compression. J Bone Joint Surg 1993;75A:1854‑1878. Eversmann WW Jr Entrapment and compression neuropathies, in Green DP (ed): Operative Hand Surgery, ed 2. New York, NY, Churchill‑Livingstone, 1988, pp 1423‑1478.
- 96.30 Postmortem studies in humans with well‑functioning porous‑coated acetabular
- components indicate which of the following characteristics of bone ingrowth in
- the prosthetic interface?
- 1‑ Occupies less than 35% of the prosthetic surface
- 2‑ Occurs only at the screw sites
- 3‑ Occurs in 65% of the surface
- 4‑ Occurs in 90% or more of the acetabular component
- 5‑ Develops along with granulomas in the nonossified areas of the interface
- Question 96.30
- Answer = 1
- Reference(s)
- Engh CA, Zettl‑Schaffer KF, Kukita Y, et al: Histological and radiographic assessment of well functioning porous coated acetabular components: A human postmortem retrieval study. J Bone Joint Surg 1993;75A:814‑824.
- 96.31 Following total knee arthroplasty, the tendency for patellar dislocation is most
- commonly associated with which of the following conditions?
- 1‑ Internal rotation of the tibial component
- 2‑ External rotation of the tibial component
- 3‑ External rotation of the femoral component
- 4‑ Medial translation of the patellar component
- 5‑ Medial translation of the femoral component
- Question 96.31
- Answer = 1
- Reference(s)
- Frymoyer JW (ed): Orthopaedic Knowledge Update 4. Rosemont, IL, American Academy of Orthopaedic Surgeons, 1993, pp 603‑624.
- 96.32 A 30‑year‑old woman has plantar and lateral pain at the fifth metatarsal head
- associated with a plantar callus. A radiograph shows a wide 4‑5 intermetatarsal
- angle. Orthotic and nonsurgical treatment have failed. Surgical treatment should
- consist of
- 1‑ excision of the fifth metatarsal head.
- 2‑ excision of the lateral capsule and capsulorraphy.
- 3‑ osteotomy at the base of the fifth metatarsal.
- 4‑ long oblique osteotomy of the fifth metatarsal shaft.
- 5‑ fifth toe amputation.
- Question 96.32
- Answer = 4
- Reference(s)
- Coughlin MJ: Etiology and treatment of the bunionette deformity, in Greene WB (ed): Instructional Course Lectures XXXIX. Park Ridge, IL, American Academy of Orthopaedic Surgeons, 1990, pp 37‑48. Kitaoka HB, Holiday AD Jr: Metatarsal head resection for bunionette: Long‑term follow‑up. Foot Ankle Im 1991;11:345‑349. Coughlin MJ: Treatment of bunionette deformity with longitudinal diaphyseal osteotomy with distal soft tissue repair. Foot Ankle Int 1991;11:195‑203.
- 96.33 A 40‑year‑old woman has shoulder pain and winging of the scapula following a
- thoracotomy. Which of the following nerves has most likely been injured?
- 1‑ Suprascapular
- 2‑ Long thoracic
- 3‑ Dorsal scapular
- 4‑ Musculocutaneous
- 5‑ Axillary
- Question 96.33
- Answer = 2
- Reference(s)
- Hollinshead WH (ed): Anatomy for Surgeons: Back and Limbs. Harper & Row, 1982, pp 259‑337.
- 96.34 A 49‑year‑old man who sustains a rotator cuff tear undergoes open repair of the injury. One year later, the patient has persistent pain and weakness. Imaging studies show a persistent rotator cuff tear. Which of the following factors indicates a poor prognosis for repeat rotator cuff repair?
- 1‑ Previous acromioplasty
- 2‑ Previous deltoid splitting approach
- 3‑ Previous lateral acromionectomy
- 4‑ Small‑ to moderate‑sized tear
- 5‑ Avulsion of the supraspinatus tendon
- answer
- back
- Question 96.34
- Answer = 3
- back to this question
- next question
- Reference(s)
- Bigliani LU, Cordasco FA, McIlveen S1, et al: Operative treatment of failed repairs of the rotator cuff. J Bone Joint Surg 1992;74A:1505‑1515.
- 96.35 During a revision total hip replacement, a retractor is placed beneath the rectus
- tendon over the anterior brim of the pelvis at the level of the anterior inferior
- iliac spine. The retractor slips many times and is replaced over the top of the
- rectus tendon rather than beneath it. What structure in immediate proximity to
- the retractor is most likely to be damaged?
- 1‑ Profunda femoris artery
- 2‑ Femoral vein
- 3‑ Femoral artery
- 4‑ Femoral nerve
- 5‑ Lateral femoral cutaneous nerve
- Question 96.35
- Answer = 4
- Reference(s)
- Hoppenfeld S, deBoer P: The hip, in Hoppenfeld S, deBoer P (ads): Surgical Exposures in Orthopaedics: The Anatomic Approach. Philadelphia, PA, JB Lippincott, 1984, pp 301‑356.
- 96.36 A 2‑year‑old girl who is otherwise healthy has severe bowing of both lower extremities that appears to be getting worse. Physical examination is normal except for the symmetric bowing of her lower extremities. A radiograph is shown in Figure 7. Laboratory studies disclosed the following values: normal serum protein; serum calcium, 8.9 mg/dl (normal 8.7 to 10.5 mg/dl), phosphorus, 3 mg/dl (normal 4.0 to 7.0 mg/dl), alkaline phosphatase, 1050 IU/I (normal 65 to 380 IU/1), parathyroid hormone, 25 pmol/1(normal 1.0 to 5.2 pmol/1), and 25‑hydroxyvitamin D, 8 ng/ml (normal 14 42 ng/ml). Which of the following is the best treatment for this patient's condition?
- 1‑ Vitamin D
- 2‑ Phosphate supplements
- 3‑ Osteotomy of the tibia to unload the medial plateau
- 4‑ Osteotomy of the tibia and femur to maintain a
- level joint
- 5‑ Observation
- Fig. 7
- Question 96.36
- Answer = 1
- Reference(s)
- Mankin HJ: Metabolic bone disease. J Bone Joint Surg 1994;76A:760‑788. Mankin HJ: Rickets, osteomalacia, and renal osteodystrophy: Part II. 1 Bone Joint Surg 1974;56A:352‑386.
- 96.37 When performing fasciotomy of the leg for compartment syndrome, release of
- the fascia of the lateral compartment will decompress which of the following
- neurovascular structures?
- 1‑ Anterior tibial artery and deep peroneal nerve
- 2‑ Deep peroneal nerve
- 3‑ Superficial peroneal nerve
- 4‑ Peroneal artery
- 5‑ Perforating peroneal artery
- Question 96.37
- Answer = 3
- Reference(s)
- Last RJ: Anatomy Regional and Applied, ed 6. Edinburgh, Scotland, Churchill‑Livingstone, 1978, pp 170‑171. Butterworth‑Heinemann, Oxford, England, 1994, pp 161‑164. Rorabeck CH: Compartment syndromes, in Browner BD, Jupiter JB, Levine AM, et al (ads): Skeletal Trauma. Philadelphia, PA, WB Saunders, 1992, pp 285‑310.
- 96.38 What ligament acts as the primary restraint to inversion and anterior talar
- translation at 15 degrees of ankle plantar flexion?
- 1‑ Anterior talofibular
- 2‑ Posterior talofibular
- 3‑ Calcaneofibular
- 4‑ Deltoid
- 5‑ Anterior syndesmosis
- Question 96.38
- Answer = 1
- Reference(s)
- Colville MR: Reconstruction of the lateral ankle ligaments, in Jackson DW (ed): Instructional Course Lectures 44. Rosemont, IL, American Academy of Orthopaedic Surgeons, 1995, pp 341‑348. Johnson EE, Markolf KL: The contribution of the anterior talofibular ligament to ankle laxity. J Bone Joint Surg 1983;65A:81‑88.
- 96.39 What radiographic finding is the best indication of long bone healing?
- 1‑ Diffuse osteopenia near the fracture site
- 2‑ Loss of a distinct fracture line
- 3‑ Dense trabeculae near the fracture line
- 4‑ Restoration of cortical continuity
- 5‑ Abundant callus with calcification
- Question 96.39
- Answer = 4
- Reference(s)
- Cornell CN: Current assessment of fracture healing, in Brighton Cr. Friedlaender GE, Lane JM (eds): Bone Formation and Repair. Rosemont, IL, American Academy of Orthopaedic Surgeons, 1994, pp 500‑501. Panjabi MM, Walter SD, Karuda M, et al: Correlation of radiographic analysis of healing fractures with strength: A statistical analysis of experimental osteotomies. J Orthop Res 1985;3:212‑218.
- 96.40 A 27‑year‑old woman has had pain and numbness in the thumb, index, and long fingers for 5 months. Hoffman‑Tinel's sign is positive over the median innervated digits with percussion over the median nerve at the wrist. Two‑point discrimination is 5 mm in all digits, and there is no weakness or thenar atrophy. Electrodiagnostic studies show no denervation potentials and a 2‑ms delay in the median nerve motor and sensory terminal latency. Recommended treatment should be
- 1‑ open decompression of the carpal tunnel.
- 2‑ endoscopic decompression of the carpal tunnel.
- 3‑ median nerve decompression at the proximal forearm
- 4‑ steroid injection and splinting.
- 5‑ oral nonsteroidal anti‑inflammatory medications and vitamin B6.
- Question 96.40
- Answer = 4
- Reference(s)
- Gelberman RH, Aronson D, Weisman MH: Carpal‑tunnel syndrome: Results of a prospective trial of steroid injection and splinting. J Bone Joint Surg 1980;62A:1181‑1184.
- 96.41 An asymptomatic 3‑1/2‑year‑old girl with a normal gait presents with the
- anteroposterior radiograph shown in Figure 8a. Figure 8b is a radiograph with the
- hips held in abduction and internal rotation. No hip instability is noted on physical
- examination. Recommended management of the left hip should consist of
- 1‑ observation.
- 2‑ abduction splinting at night.
- 3‑ open reduction through an anterior approach.
- 4‑ acetabular shelf.
- 5‑ innominate osteotomy.
- Fig. 8a Fig. 8b
- Question 96.41
- Answer = 5
- Reference(s)
- Herring JA: Congenital dislocation of the hip, in Morrissy RT (ed): Lovell and Winter's Pediatric Orthopaedics, ed 3. Philadelphia, PA, JB Lippincott, 1990, vol 2, pp 815‑850. Salter RB, Dubos JP: The first fifteen years' personal experience with innominate osteotomy in the treatment of congenital dislocation and subluxation of the hip. Clin Orthop 1974;98:72‑103.
- 96.42 What acute findings are expected following closed rupture of the central slip
- insertion into the base of the middle phalanx of a finger?
- 1‑ Swan neck deformity
- 2‑ Boutonniere deformity
- 3‑ Mallet finger
- 4‑ Proximal interphalangeal joint swelling with mild extensor lag
- 5‑ Proximal interphalangeal joint subluxation
- Question 96.42
- Answer = 4
- Reference(s)
- Carducci AT: Potential boutonniere deformity: Its recognition and treatment. Orthop Rev 1981;10:121‑123. Lovett WL, McCalla MA: Management and rehabilitation of extensor tendon injuries. Orthop Clin North Am 1983;14:811‑826.
- 96.43 The purpose of obtaining informed consent during clinical research on pediatric
- patients is to make sure that the child's guardian is
- 1- notified of the outcome following completion of the study.
- 2- unable to withdraw consent once participation in a study has begun.
- 3- aware that the child's name and photograph may be used when the research is
- published.
- 4- aware of potential risks that may result from participation in a research protocol.
- 5- aware of whether the child is receiving the experimental treatment or a placebo.
- answer
- back
- Question 96.43
- Answer = 4
- back to this question
- next question
- Reference(s)
- Riecken HW, Ravich R: Informed consent to biomedical research in Veterans Administration Hospitals. JAMA 1982;248:344-348. Brahams D: Randomized trials and informed consent. Lancet 1988;2:1033-1034. Grodin MA, Alpert JJ: Children as participants in medical research. Pediatr Clin North Am 1988;35:1389-1401.
- 96. 44 A 16‑year‑old girl has a 2‑week history of increasing pain in her back and left buttock region that radiates down her left leg to the knee. She had a low‑grade fever 3 weeks ago. Straight leg raising is positive at 50 degrees, popliteal compression test is negative, and neurologic examination is normal. Patrick test (figure‑of‑4 test) is positive. Internal rotation of the hip is normal. Routine lumbar spine films and an anteroposterior radiograph of the pelvis are normal. CBC is normal but the erythrocyte sedimentation rate is 100 mm/hr (normal up to 20 mm/hr). Which of the following studies should be performed to confirm a diagnosis?
- 1‑ Frog lateral views of both hips
- 2‑ Myelogram
- 3‑ CT scan of the spine
- 4‑ MRI scan of the lumbar spine
- 5‑ Bone scan
- answer
- back
- Question 96.44
- Answer = 5
- back to this question
- next question
- Reference(s)
- Warner WC Jr: Infectious arthritis, in Crenshaw AH (ed): Campbell's Operative Orthopedics, ed 8. St Louis, MO, CV Mosby, vol 1, 1992, pp 151‑175. Schaad UB, McCracken GH Jr, Nelson JD: Pyogenic arthritis of the sacroiliac joint in pediatric patients. Pediatrics 1980;66:375‑379.
- 96.45 While using a paper cutter, a 25‑year‑old man sustains a complete amputation
- through the thumb metacarpophalangeal joint. Replantation was accomplished.
- Thirty‑six hours after replantation, sluggish refill is noted, and the temperature
- of the pulp has gone from 35° to 29 °C (95 ° to 84.2 °F) over the last 3 hours.
- Recommended treatment should be
- 1‑ a heparin bolus and systemic anticoagulation.
- 2‑ surgical exploration.
- 3‑ application of medical grade leeches.
- 4‑ stellate ganglion blocks.
- 5‑ loosening of dressing and reevaluation in 2 hours.
- Question 96.45
- Answer = 2
- Reference(s)
- Goldner RD: Postoperative management. Hand Clin 1985;1:205‑215.
- 96. 46 A 7‑year‑old boy with no history of trauma has had neck stiffness for 1 week. Cervical flexion and extension are limited by discomfort. Neurologic examination is normal. Anteroposterior and lateral radiographs of the cervical spine are shown in Figures 9a and 9b. Treatment should consist of
- 1‑ anterior diskectomy.
- 2‑ halo traction.
- 3‑ IV antibiotics.
- 4‑ rest and analgesics.
- 5‑ irradiation of the lesion.
- Fig. 9a
- Fig. 9b
- answer
- back
- Question 96.46
- Answer = 4
- back to this question
- next question
- Reference(s)
- Sonnabend DH, Taylor TK, Chapman GK: Intervertebral disc calcification syndromes in children. J Bone Joint Surg 1982;64B:25‑31. Wong CC, Pereira B, Pho RW: Cervical disc calcification in children: A long‑term review. Spine 1992;17:139‑144.
- 96.47 What is the most common location of adamantinoma?
- 1‑ Fibula
- 2‑ Tibia
- 3‑ Femur
- 4‑ Humerus
- 5‑ Radius
- Question 96.47
- Answer = 2
- Reference(s)
- Hazelbag HM, Taminiau AH, Fleuren GJ, et al: Adamantinoma of the long bones: A clinicopathological study of thirty‑two patients with emphasis on histological subtype, precursor lesion, and biological behavior. J Bone Joint Surg 1994;76A:1482‑1499.
- 96.48 Figure 10 is an MRI scan of the shoulder. The structure indicated by the arrow
- is innervated by what nerve?
- 1‑ Axillary
- 2‑ Lower subscapular
- 3‑ Upper subscapular
- 4‑ Suprascapular
- 5‑ Long thoracic
- Figure 10
- Question 96.48
- Answer = 2
- Reference(s)
- Netter F: Atlas of Human Anatomy. Ciba‑Geigy, 1989, plate 401. Sinclari DC: Muscles and fasciae, in Romanes GI (ed): Cunningham's Textbook of Anatomy. London, England, Oxford University Press, 1972, p 311. Magnetic Resonance Imaging in Orthopaedic & Sports Medicine. JB Lippincott 1993, pp 555‑556.
- 96.49 A 28‑year‑old man injures his dominant shoulder when his arm is forcibly elevated by machinery. Active and passive motion are limited due to diffuse pain. MRI scans with T,‑weighted and fat saturation sequences are seen in Figures l la and l lb. Appropriate treatment should include which of the following?
- 1‑ Nonsteroidal anti‑inflammatory drugs and outpatient physical therapy
- 2‑ Shoulder immobilization for 1 month
- 3‑ Arthroscopic shoulder stabilization
- 4‑ Rotator cuff repair
- 5‑ Coracoclavicular ligament reconstruction
- answer
- back
- Question 96.49
- Answer = 4
- back to this question
- next question
- Reference(s)
- Bassett RW, Cofield RH: Acute tears of the rotator cuff: The timing of surgical repair. Clin Orthop 1983;175:18‑24. Beltran J: The use of magnetic resonance imaging about the shoulder. J Shoulder Elbow Surg 1992;1:321.
- 96.50 When taken prior to conception, what dietary supplement reduces the chance of
- having a second child with myelomeningocele?
- 1‑ Selenium
- 2‑ Folic acid
- 3‑ Vitamin A
- 4‑ Vitamin B 12
- 5‑ Vitamin E
- Question 96.50
- Answer = 2
- Reference(s)
- Kasser JR (ed): Orthopaedic Knowledge Update 5. Rosemont, IL, American Academy of Orthopaedic Surgeons, 1996, pp 195‑202.
- 96.51 A 7‑year‑old girl presents for evaluation and possible treatment of a unilateral
- congenital hand deformity (Figure 12). What other physical finding is most
- likely?
- 1‑ Cardiac septal defect
- 2‑ Premature craniosynostosis
- 3‑ Ipsilateral chest wall abnormalities
- 4‑ Tracheoesophagealfistula
- 5‑ Similar findings in the toes on the same side
- Figure 12
- Question 96.51
- Answer = 3
- Reference(s)
- Flatt AE: The Care of Congenital Hand Anomalies, ed 2. St Louis, MO. Quality Medical Publishers, 1994, pp 270‑275.
- 96.52 A 12‑year‑old boy has a disabling recurrent lateral dislocation of the patella. Quadriceps
- rehabilitation and bracing have failed to prevent daily episodes of dislocation. Examination \
- reveals normal varus/valgus at the knee but an increased Q angle of the patellar ligament,
- and the patella can easily be dislocated laterally. In addition to proximal quadriceps
- realignment, treatment should include which of the following?
- 1‑ Hauser medial transfer of the tibial tubercle
- 2‑ Emshe‑Trillat medial transfer of the tibial tubercle
- 3‑ Partial patellar tendon transfer medially
- 4‑ Lateral femoral trochlear elevation
- 5‑ Maquet tibial tubercle elevation
- answer
- back
- Question 96.52
- Answer = 3
- back to this question
- next question
- Reference(s)
- Staheli LT: The lower limb, in Morrissy RT (ed): Lovell and Winter's Pediatric Orthopaedics, ed 3. Philadelphia, PA, JB Lippincott, 1990, vol 2, pp741‑766.
- 96.53 Which of the following best describes the location of the mechanical axis of the
- lower extremity in a normal limb?
- 1‑ Center of the femoral head, center of the knee, and center of the ankle
- 2‑ Center of the femoral head, slightly medial of the knee center, and center of
- the ankle
- 3‑ Center of the femoral head, slightly lateral of the knee center, and center of
- the ankle
- 4‑ Medial of the femoral head, center of the knee, and center of the ankle
- 5‑ Lateral of the femoral head, center of the knee, and center of the ankle
- Question 96.53
- Answer = 2
- Reference(s)
- Rand JA: The knee: Total knee arthroplasty‑techniques, in Morrey B (ed): Joint Replacement Arthroplasty. New York, NY, Churchill‑Livingstone, 1991, p 989.
- 96.54 After injury to the posterior cruciate ligament, increased osteoarthrosis is most
- commonly seen in which of the following compartments?
- 1‑ Lateral only
- 2‑ Lateral and patellofemoral
- 3‑ Medial and lateral
- 4‑ Medial and patellofemoral
- 5‑ Medial, lateral, and patellofemoral
- answer
- back
- Question 96.54
- Answer = 4*
- back to this question
- next question
- Reference(s)
- Skyhar MJ, Warren RF, Ortiz GJ, et al: The effects of sectioning of the posterior cruciate ligament and the posterolateral complex on the articular contact pressures within the knee. J Bone Joint Surg 1993;75A:694‑699.
- * item deleted after statistical review
- 96.55 Which of the following refers to a supervising physician's responsibility for the
- actions of an orthopaedic resident?
- 1‑ Ad damnum clause
- 2‑ Comparative negligence
- 3‑ Contributory negligence
- 4‑ Vicarious liability
- 5‑ Indemnity agreement
- Question 96.55
- Answer = *4
- Reference(s)
- AAOS Committee on Professional Liability: Medical Malpractice: A Primer for Orthopaedic Residents and Fellows. Rosemont, IL, American Academy of Orthopaedic Surgeons, 1993, p 14. *item deleted after statistical review
- 96.56 Which structure is encountered between the popliteus tendon and fibular
- collateral ligament during exposure of the posterolateral knee corner?
- 1‑ Peroneal nerve
- 2‑ Iliotibial band
- 3‑ Biceps tendon
- 4‑ Oblique popliteal ligament
- 5‑ Inferior geniculate artery
- Question 96.56
- Answer = 5
- Reference(s)
- Seebacher JR, Inglis AE, Marshall JL, et al: The structure of the posterolateral aspect of the knee. J Bone Joint Surg 1982;64A:536‑541.
- 96.57 When balancing soft tissue in a total knee replacement, what is the most
- appropriate next step when the knee is correctly balanced in flexion but loose in
- extension?
- 1‑ Augment the distal portion of the femoral component.
- 2‑ Augment the posterior condyles of the femoral component.
- 3‑ Advance the tibial tubercle.
- 4‑ Insert a bone graft on the proximal tibial surface.
- 5‑ Insert a thicker tibial component.
- Question 96.57
- Answer = 1
- Reference(s)
- Rand JA: The knee: Total knee arthroplasty‑techniques, in Morrey B (ed): Joint Replacement Arthroplasty. New York, NY, Churchill‑Livingstone, 1991, p 994.
- 96.58 A 65‑year‑old man undergoes primary total knee arthroplasty and has a routine
- postoperative course (Figure 13). Six months after surgery he is fully ambulatory,
- has little or no pain, and has a range of motion from 0 to 100 degrees. After
- experiencing the acute onset of anterior knee pain when arising from a chair, he is
- able to do a straight leg raise with full extension, but has a moderate amount of
- pain. Radiographs show a patella fracture with the patellar component still well
- fixed. The best course of treatment is
- 1‑ patellectomy with retinacular repair.
- 2‑ open reduction and cerclage wiring.
- 3‑ immobilization in extension for 6 weeks.
- 4‑ patellar component revision.
- 5‑ a hinged knee brace with no weightbearing.
- Fig. 13
- Question 96.58
- Answer = 3
- Reference(s)
- Scuderi GR, Insall JN, Scott WN: Patellofemoral pain after total knee arthroplasty. J Am Acad Orthop Surg 1994;2:239‑246. Windsor RE, Scuderi GR, Insall JN: Patellar fractures in total knee arthroplasty. J Arthroplasty 1989;4:63‑67.
- 96.59 An 18‑year‑old woman presents with a 2‑month history of increasing shoulder pain that occurs both day and night and is not relieved by nonsteroidal anti‑inflammatory medications. An anteroposterior radiograph of the proximal humerus is shown in Figure 14a. A photomicrograph is shown in Figure 14b. The most likely diagnosis is
- 1‑ Ewing's tumor.
- 2‑ osteosarcoma.
- 3‑ bone infarct.
- 4‑ osteomyelitis.
- 5‑ enchondromatosis.
- Fig. 14b
- Fig. 14a
- Question 96.59
- Answer = 4
- Reference(s)
- Bone tumors, in Dahlin DC, Unni KK (eds): Conditions That Commonly Simulate Primary Neoplasms of Bone. General aspects and data on 8,542 cases, ed 4. Springfield, IL, Charles C Thomas Publishers, 1986, pp 406‑481.
- 96.60 The most common cause of synovitis of the second metatarsophalangeal joint is
- attritional changes in the volar plate related to what factor?
- 1‑ Pressure from a valgus great toe
- 2‑ A long second metatarsal
- 3‑ Rheumatoid arthritis
- 4‑ Traumatic dislocation
- 5‑ Hammering of the second toe proximal interphalangeal joint
- Question 96.60
- Answer = 2
- Reference(s)
- Fortin PT, Myerson MS: Second metatarsophalangeal joint instability. Foot Ankle Int1995;16:306‑313. Mann RA, Coughlin MJ: Surgery of the Foot and Ankle, ed 6. St Louis, MO, CV Mosby, 1993, p 372.
- 96.61 The lowest hoop stresses occurring in the bone during nail insertion for a
- femoral shaft fracture would be produced by a nail with which of the following
- characteristics?
- 1‑ Closed section, thick‑walled, stainless steel
- 2‑ Closed section, thin‑walled, stainless steel
- 3‑ Slotted, thin‑walled, titanium alloy
- 4‑ Slotted, thick‑walled, titanium alloy
- 5‑ Slotted, thin‑walled, stainless steel
- Question 96.61
- Answer = 3
- Reference(s)
- Harper MC, Carson WL: Curvature of the femur and the proximal entry point for an intramedullary rod. Clin Orthop 1987;220:155‑161. Johnson KD: Femoral shaft fractures, in Browner BD, Jupiter JB, Levine AM, et al (eds): Skeletal Trauma: Fractures, Dislocations, Ligamentous Injuries. Philadelphia, PA, WB Saunders, 1992, pp 1525‑1642.
- 96.62 Recommended treatment for a 5‑year‑old boy with cerebral palsy and neuromuscular hip dysplasia, includes was bilateral adductor tenotomies and femoral osteotomies. The insurance company requires a second surgical opinion. The second physician believes an adductor tenotomy alone will be satisfactory. What would be the most appropriate course of action for the second physician?
- 1- Describe any deficiencies in the first surgeon's professional qualification in order to save the
- patient the additional surgery.
- 2- Avoid any further involvement with the patient because the patient was referred only for a
- second opinion.
- 3- Try to get the insurance carrier to steer the patient away from the first surgeon, using the
- premise that he would be more costly.
- 4- Allow the family to consider the two recommendations and accept care of the patient if they
- choose the second physician's plan of action
- 5‑ In order to maintain a good relationship, agree with the first surgeon, who
- sometimes refers patients.
- Question 96.62
- Answer = 4
- Reference(s)
- AAOS Committee on Ethics: Guide to the Ethical Practice of Orthopaedic Surgery. Park Ridge, IL, American Academy of Orthopaedic Surgeons, 1992, p 37.
- 96. 63 Reduction of the deformity shown in Figure 15 in a patient without neurologic deficit is
- most commonly associated with which neurologic complication?
- 1‑ Loss of L3 nerve root function
- 2‑ Loss of L4 nerve root function
- 3‑ Loss of L5 nerve root function
- 4‑ Loss of S 1 nerve root function
- 5‑ Cauda equina syndrome
- Fig. 15
- answer
- back
- Question 96.63
- Answer = 3
- back to this question
- next question
- Reference(s)
- Edwards C, Steffee A: Controversies of instrumentated reduction of spondylolisthesis. Spine 1994;19:1535‑1537. Lehmer SM, Steffee AD, Gaines RW Jr: Treatment of L5‑S1 spondyloptosis by staged L5 resection with reduction and fusion of L4 onto S1. Spine, 1994;19:1916‑25.
- 96.64 A 29‑year‑old man falls off his bicycle and injures his elbow as shown in Figures
- 16a and 16b. Closed reduction is performed under sedation with subsequent
- radiographs shown in Figures 16c and 16d. The reduction is maintained as the
- elbow is taken from 20 to 90 degrees of flexion. At 20 degrees, valgus stress shows
- 10 degrees of laxity. The most appropriate treatment is
- 1‑ use of a posterior splint for 10 days, followed by active range of motion
- exercises.
- 2‑ use of a hinged external fixator, followed by active range of motion exercises.
- 3‑ casting for 6 weeks, followed by active assisted range of motion exercises.
- 4‑ medial collateral ligament repair with casting for 3 weeks, followed by active
- range of motion exercises.
- 5- repair of the medial collateral ligament and pronator teres origin with
- immediate range of motion exercises.
- Go to next slide for figures and answer link
- back
- Fig. 16a
- Fig. 16b
- Fig. 16c
- Fig. 16d
- answer
- back
- Question 96.64
- Answer = 1
- back to this question
- next question
- Reference(s)
- Josefsson PO, Gentz CF, Johnell O et al: Surgical versus nonsurgical treatment of ligamentous injuries following dislocation of the elbow joint. Clin Orthop 1987;214:165‑169. Jupiter JB: Trauma to the adult elbow and fractures of the distal humerus, in Browner BD, Jupiter JB, Levine AM (eds): Skeletal Trauma: Fractures, Dislocations, Ligamentous Injuries. Philadelphia, PA, WB Saunders, 1992, pp 1129‑1176.
- 96.65 What is the most common long‑term complication of cemented total hip
- arthroplasty in patients under 50 years of age?
- 1‑ Infection
- 2‑ Dislocation
- 3‑ Periprosthetic femur fracture
- 4‑ Acetabular component loosening
- 5‑ Femoral stem fracture
- Question 96.65
- Answer = 4
- Reference(s)
- Barrack RL, Mulroy RD Jr, Harris WH: Improved cementing techniques and femoral component loosening in young patients with hip arthroplasty: A 12‑year radiographic review. J Bone Joint Surg 1992;74B:385‑389. Callaghan JJ: Results of primary total hip arthroplasty in young patients. J Bone Joint Surg 1993;75A:1728‑1734.
- 96.66 The anteroposterior and forced dorsiflexion lateral radiographs of a 4‑month‑old
- infant born with idiopathic talipes equinovarus are shown in Figures 17a and 17b,
- respectively. Treatment, which has consisted of serial casting since shortly after
- birth, should now include
- 1‑ continued serial casting.
- 2‑ straight last shoes.
- 3‑ an ankle‑foot orthosis.
- 4‑ an elective complete surgical release.
- 5‑ surgical hindfoot release.
- Fig. 17a Fig. 17b
- Question 96.66
- Answer = 4
- Reference(s)
- Simons GW: Complete subtalar release in club feet: Part II ‑ Comparison with less extensive procedures. J Bone Joint Surg 1985;67A:1056‑1065 Simons GW: A standardized method for the radiographic evaluation of clubfeet. Clin Orthop 1978;135:107‑118.
- 96.67 A 50‑year‑old man who lacerated the dorsum of his hand over the ring and small
- finger metacarpals 3 months ago (Figure 18) now complains of a weak grip.
- Primary tenorrhaphy has been performed. When the metacarpophalangeal joint is
- flexed 70 degrees, the PIP joint cannot be flexed more than 70 degrees. Passive
- metacarpophalangeal extension allows full PIP flexion. What is the most likely
- cause of the patient's weak grip?
- 1‑ Intrinsic tightness
- 2‑ Intrinsic muscle compartment syndrome
- 3‑ Extrinsic extensor tightness
- 4‑ Metacarpophalangeal joint contracture
- 5‑ Proximal interphalangeal joint contracture
- Figure 18
- Question 96.67
- Answer = 3
- Reference(s)
- Burton RI: Extensor tendons: Late reconstruction, in Green DP (ed): Operative Hand Surgery, ed 3. New York, NY, Churchill‑Livingstone, 1993, vol 2, pp 1955‑1988.
- 96.68 A 13‑year‑old boy has back pain, diffuse tenderness in the lower thoracic spine,
- and pain with forward bending after falling at school. Neurologic examination is
- normal. A spine radiograph and skull film are shown in Figures 19a and 19b,
- respectively. Which of the following studies is most likely to confirm the diagnosis?
- 1‑ Biopsy of the ilium
- 2‑ Serum protein electrophoresis
- 3‑ Hemoglobin electrophoresis
- 4‑ Bone marrow smear
- 5‑ Analysis of vitamin B12 levels
- Fig. 19a Fig. 19b
- Question 96.68
- Answer = 3*
- Reference(s)
- Aegerter E, Kirkpatrick JA: Orthopaedic Diseases, ed 2. Philadelphia, PA, WB Saunders, 1968, pp 356‑359. Niemann KMW: Diseases related to the hematopoietic system, in Morrissey RT (ed): Lovell and Winter's Pediatric Orthopaedics, ed 3. Philadelphia, PA, JB Lippincott, 1990, vol 1, pp 263‑296. *item deleted after statistical review
- 96.69 A 34‑year‑old woman has a densely ossified, lobulated mass in the posterior aspect of the distal femur. The biopsy shows a low‑grade parosteal osteogenic sarcoma. Which of the following would be the most appropriate treatment regimen?
- 1‑ Wide surgical resection
- 2‑ External beam irradiation
- 3‑ Multi‑agent chemotherapy
- 4‑ Multi‑agent chemotherapy and wide surgical resection
- 5‑ Multi‑agent chemotherapy and external beam irradiation
- Question 96.69
- Answer = 1
- Reference(s)
- Campanacci M, Bertoni F, Bacchini P: Bone and soft‑tissue tumors: Parosteal osteosarcoma. New York, NY, Springer‑Verlag, 1990, pp 433‑454. Okada K, Frassica FJ, Sim FH, et al: Parosteal osteosarcoma: A clinicopathological study. J Bone Joint Surg 1994;76A:366‑378. Wold LE, Unni KK, Beabout JW, et al: Dedifferentiated parosteal osteosarcoma. J Bone Joint Surg 1984;66A:53‑59.
- 96.70 Arterial Doppler screening of a patient with diabetes and dry gangrene of the
- forefoot reveals an ankle to brachial artery index of 1.2. This index is strongly
- suggestive of what condition?
- 1‑ Satisfactory perfusion of the foot
- 2‑ Calcification of the ankle vessels
- 3‑ Normal wave form
- 4‑ Diabetic hypertension
- 5‑ Nondiabetic peripheral vascular disease
- Question 96.70
- Answer = 2
- Reference(s)
- Brodsky JW: The diabetic foot, in Mann RA, Coughlin MJ (eds): Surgery of the Foot and Ankle, ed 6. St Louis, MO. CV Mosby, 1993, pp 877‑958.
- 96.71 A feature differentiating epiphyseal dysplasias from nonepiphyseal dysplasias
- (achondroplasia, metaphyseal dysplasia, etc) is that the former are more likely to
- lead to the development of what condition?
- 1‑ Varus or valgus at knees
- 2‑ Premature degenerative joint disease
- 3‑ Spinal stenosis
- 4‑ Thoracolumbar kyphosis
- 5‑ Rhizomelic shortening
- Question 96.71
- Answer = 2
- Reference(s)
- Beals RK, Horton W: Skeletal dysplasias: An approach to diagnosis. J Am Acad Orthop Surg 1995;3:174‑181.
- 96.72 A 66‑year‑old woman has a mass (Figure 20) overlying the distal phalanx of her
- index finger. Radiographs are most likely to reveal what condition?
- 1‑ Lucency in the distal phalanx
- 2‑ Pressure erosion of the distal phalanx
- 3‑ Osteophytes at the distal interphalangeal joint
- 4‑ Normal bone and joint
- 5‑ Resorption of the distal phalangeal tuft
- Figure 20
- Question 96.72
- Answer = 3
- Reference(s)
- Gingrass MK, Brown RE, Zook EG: Treatment of fingernail deformities secondary to ganglions of the distal interphalangeal joint. J Hand Surg 1995;20A:502‑505.
- 96.73 A 13‑year‑old boy with no history of trauma complains of left foot pain while running. The subtalar motion is 25% of that observed on the right side, and there is no swelling. Anteroposterior and lateral radiographs of the left foot are shown in Figures 21a and 21b, respectively. Which of the following imaging studies should be performed next?
- 1‑ Bone scan
- 2‑ MRI scan
- 3‑ Axial and oblique radiographs of the foot
- 4‑ Ankle stress radiographs
- 5‑ Ankle arthrography
- Fig. 21a Fig. 21b
- Question 96.73
- Answer = 3
- Reference(s)
- Jayakumar S, Cowell HR: Rigid flatfoot. Clin Orthop 1977;122:77‑84. Sullivan JA: Tarsal coalition, in Morrissy RT (ed): Lovell and Winter's Pediatric Orthopaedics, ed 3. Philadelphia, PA, JB Lippincott, 1990, vol 2, pp 963‑969.
- 96.74 What is the most common complication of cementless hip arthroplasty?
- 1‑ Infection
- 2‑ Prosthetic dislocation
- 3‑ Pulmonary embolism
- 4‑ Thigh pain
- 5‑ Femoral fracture
- Question 96.74
- Answer = 4
- Reference(s)
- Bourne RB, Rorabeck CH. Ghazal ME, et al: Pain in the thigh following total hip replacement with a porous‑coated anatomic prosthesis for osteoarthritis: A five‑year follow‑up study. J Bone Joint Surg 1994;76A:1464‑1470.
- 96.75 What is the first response to injury in hemostasis?
- 1‑ Fibrin formation
- 2‑ Platelet plug formation
- 3‑ Vascular constriction
- 4‑ Fibrinolysis
- 5‑ Formation of megakaryocytes
- Question 96.75
- Answer = 3
- Reference(s)
- Kasser JR (ed): Orthopaedic Knowledge Update 5. Rosemont, IL, American Academy of Orthopaedic Surgeons, 1996, pp 53‑59. Lieberman JR, Geerts WH: Prevention of venous thromboembolism after total hip and knee arthroplasty. J Bone Joint Surg 1994;76A:1239‑1250.
- 96.76 A 10‑year‑old boy has Pseudomonas osteomyelitis after sustaining a puncture
- wound of the foot. Appropriate treatment should include which of the following?
- 1‑ High‑dose oral antibiotics for 6 weeks
- 2‑ Antibiotics for 2 to 3 weeks
- 3‑ Surgical debridement and parenteral antibiotics
- 4‑ A bone scan and parenteral antibiotics for 6 weeks
- 5‑ Short leg cast and parenteral antibiotics
- Question 96.76
- Answer = 3
- Reference(s)
- Jacobs RF, Adelman L, Sack CM, et al: Management of Pseudomonas osteochondritis complicating puncture wounds of the foot. Pediatrics 1982;69:432‑435.
- 96.77 A newborn who was found to have bilateral dislocated hips that could easily be reduced wore a Pavlik harness full‑time for 12 weeks. Both hips now have a full range of motion and no instability. The hip radiograph obtained after 12 weeks in the harness is shown in Figure 22. Recommended management should now consist of
- 1‑ full‑time use of the Pavlik harness.
- 2‑ decreasing the time spent in the Pavlik harness to 12 hours per day.
- 3‑ closed reduction and application of a hip spica cast under anesthesia.
- 4‑ open reduction and innominate osteotomy.
- 5‑ discontinuation of all treatment.
- Figure 22
- Question 96.77
- Answer = 3
- Reference(s)
- Mubarak S, Garfin S, Vance R, et al: Pitfalls in the use of the Pavlik harness for treatment of congenital dysplasia, subluxation, and dislocation of the hip. J Bone Joint Surg 1981;63A:1239‑1248. Ramsey PL, Lasser S, MacEwen GD: Congenital dislocation of the hip. Use of the Pavlik harness in the child during the first six months of life. J Bone Joint Surg 1976;58A:1000‑1004.
- 96.78 The patient with rheumatoid arthritis whose radiographs are shown in Figures
- 23a and 23b cannot find comfortable shoes and has plantar forefoot pain at heel
- rise that restricts her weightbearing activity to 10 minutes. Treatment should
- consist of
- 1‑ ankle arthrodesis.
- 2‑ triple arthrodesis.
- 3‑ total ankle arthroplasty.
- 4‑ forefoot reconstruction.
- 5‑ hammer toe correction.
- Fig. 23a Fig. 23b
- Question 96.78
- Answer = 4
- Reference(s)
- Mann RA, Thompson FM: Arthrodesis of the first metatarsophalangeal joint for hallux valgus in rheumatoid arthritis. J Bone Joint Surg 1984;66A:687‑692. Thompson FM, Mann RA: Arthritides, in Mann RA, Coughlin NJ (eds): Surgery of the Foot and Ankle, ed 6. St Louis, MO, CV Mosby, 1993, pp 615‑672.
- 96.79 Which of the following anatomic structures provides a dynamic contribution to
- shoulder stability?
- 1‑ Glenohumeral ligaments
- 2‑ Rotator cuff muscles
- 3‑ Deltoid muscle
- 4‑ Glenoid labrum
- 5‑ Joint capsule
- Question 96.79
- Answer = 2
- Reference(s)
- Morrey BF, An K: Biomechanics of the shoulder, in Rockwood CA, Matsen FA (eds): The Shoulder. Philadelphia, PA, WB Saunders, 1990, vol 1, pp 226‑245.
- 96.80 During total knee arthroplasty, the patellar component should be placed in which
- position on the residual patellar bone to prevent patellar instability?
- 1‑ Central
- 2‑ Medial and superior
- 3‑ Lateral and superior
- 4‑ Lateral and inferior
- 5‑ Central and inferior
- Question 96.80
- Answer = 2
- Reference(s)
- Scuderi GR, Insall JN, Scott WN: Patellofemoral pain after total knee arthroplasty. J Am Acad Orthop Surg 1994;2:239‑245.
- 96.81 A 3‑year‑old girl with myelomeningocele has a dislocated left hip. Motor examination reveals quadriceps power grade 2 out of 5 bilaterally, with grade 5 hip flexors and 4/5 adductors. There is no power in the hamstrings or hip abductors or any ankle or foot motors. Recommended treatment should be
- 1‑ open reduction of the left hip.
- 2‑ open reduction and muscle transfers.
- 3‑ open reduction, femoral or iliac osteotomy, and muscle transfers.
- 4‑ observation.
- 5‑ abduction bracing at night.
- Question 96.81
- Answer = 4
- Reference(s)
- Fraser RK, Bourke HM, Broughton NS, et al: Unilateral dislocation of the hip in spina bifida: A long‑term follow‑up. J Bone Joint Surg 1995;77B:615‑619.
- 96.82 Lisfranc's ligament in the forefoot anatomically connects which of the following
- structures?
- 1‑ Middle cuneiform to the lateral cuneiform
- 2‑ Medial cuneiform to the middle cuneiform
- 3‑ Base of the first metatarsal to the base of the second metatarsal
- 4‑ Plantar aspect of the medial cuneiform to the base of the second metatarsal
- 5‑ Dorsal surface of the medial cuneiform to the peroneus longus tendon sheath
- Question 96.82
- Answer = 4
- Reference(s)
- Schenck RC, Heckman JD: Fractures and dislocations of the forefoot: Operative and non‑operative treatment. J Am Acad Orthop Surg 1995;3:70‑78. Arntz CT, Veith RG, Hansen ST Jr. Fractures and fracture‑dislocations of the tarsometatarsal joint. J Bone Joint Surg 1988;70A:173‑181.
- 96.83 What is the most common nerve injury following a Monteggia
- fracture‑dislocation of the forearm in adults?
- 1‑ Posterior interosseous
- 2‑ Anterior interosseous
- 3‑ Radial
- 4‑ Median
- 5‑ Ulnar
- Question 96.83
- Answer = 1
- Reference(s)
- Anderson LD, Meyer FN: Fractures of the shafts of the radius and ulna, in Rockwood CA Jr. Green DP, Bucholz R W (eds): Fractures in Adults, ed 3. Philadelphia, PA, JB Lippincott, 1991.
- 96.84 Why is a cobalt‑chrome alloy preferred over a titanium alloy for a cemented
- femoral component in a total hip arthroplasty?
- 1‑ Less particulate metal debris
- 2‑ Less stiffness
- 3‑ Elastic modulus closer to bone cement
- 4‑ Cost‑effectiveness
- 5‑ Better cement bonding ability
- Question 96.84
- Answer = 1
- Reference(s)
- Friedman RJ, Black J, Galante JO, et al: Current concepts in orthopaedic biomaterials and implant fixation. J Bone Joint Surg 1993;75A:1086‑1109.
- 96.85 Orthopaedic residents and fellows are held to the same standard of care as what
- group?
- 1‑ Residents at their year level
- 2‑ Physicians licensed to practice in their home state
- 3‑ Emergency department physicians
- 4‑ Family practitioners
- 5‑ Practicing orthopaedic surgeons
- Question 96.85
- Answer = 5
- Reference(s)
- AAOS Committee on Professional Liability: Medical Malpractice: A Primer for Orthopaedic Residents and Fellows. Rosemont, IL, American Academy of Orthopaedic Surgeons, 1994, p 14.
- 96.86 A 78‑year‑old woman with a history of osteoporosis has a 4‑month
- history of back and buttock pain without antecedent trauma. Plain
- radiographs, technetium bone scans, and CT scans are consistent with
- bilateral, nondisplaced sacral insufficiency fractures. Recommended
- treatment should be
- 1‑ bilateral percutaneous screw fixation to prevent displacement.
- 2‑ bed rest for 2 weeks followed by physical therapy.
- 3‑ anterior pelvic external fixation.
- 4‑ spica cast fixation for 8 weeks.
- 5‑ use of crutches, activity modification, and analgesics.
- back
- Question 96.86
- Answer = 5
- Reference(s)
- Newhouse KE, el‑Khoury GY, Buckwalter JA: Occult sacral fractures in osteopenic patients. J Bone Joint Surg 1992;74A:1472‑1477.
- 96.87 A 30‑year‑old woman has had intermittent left hip pain for 2 years. She is otherwise healthy. There is no history of night pain, and hip range of motion is full and painless. A plain radiograph is shown in Figure 24a. A transverse CT scan image is shown in Figure 24b, while a CT scan reconstruction is shown in Figure 24c. Low‑ and highpower photomicrographs of the biopsy material are shown in Figures 24d and 24e. The best management for this patient is
- 1‑ wide surgical resection.
- 2‑ chemotherapy and external beam irradiation.
- 3‑ curettage and bone grafting.
- 4‑ incision and debridement.
- 5‑ diphosphonate therapy.
- Go to next slide for figures and answer link
- Fig. 24a
- Fig. 24d
- Fig. 24e
- Fig. 24b
- Fig. 24c
- Question 96.87
- Answer = 5
- Reference(s)
- Bone Tumors, in Dahlin DC, Unni KK (eds): Conditions That Commonly Simulate Primary Neoplasms of Bone: General aspects and data on 8,0542 cases, ed 4. Springfield, IL, Charles C Thomas Publishers, 1986, pp 406‑481.
- 96.88 A 6‑year‑old boy who was injured in a motor vehicle accident was wearing a
- lap belt, but not the shoulder strap, at the time of the injury. Radiographs are
- seen in Figures 25a and 25b. What is the most likely mechanism of injury?
- 1‑ Shear
- 2‑ Axial load
- 3‑ Flexion
- 4‑ Flexion‑distraction
- 5‑ Flexion‑rotation
- Fig. 25a Fig. 25b
- Question 96.88
- Answer = 4
- Reference(s)
- Kostuik JP, Huler RJ, Esses SI, et al: Thoracolumbar spine fracture, in Frymoyer JW (ed): The Adult Spine: Principles and Practice. New York, NY, Raven Press, 1991, vol 2, pp 1269‑1329.
- 96.89 Ultra‑high molecular weight polyethylene is defined as what type of material?
- 1‑ Elastic
- 2‑ Viscoelastic‑plastic
- 3‑ Rigid
- 4‑ Shear thinning
- 5‑ High friction
- Question 96.89
- Answer = 2
- Reference(s)
- Chao EYS, Aro HT: Biomechanics of fracture fixation, in Mou VC, Hayes WC (eds): Basic Orthopaedic Biomechanics. New York, NY, Raven Press, 1991. Frankel VS, Burstein AH: Orthopaedic Biomechanics. Philadelphia, PA, Lea and Febiger, 1970. Simon SR (ed): Orthopaedic Basic Science. Rosemont, IL, American Academy of Orthopaedic Surgeons, 1994, pp 447‑486.
- 96.90 Plain radiographs of which of the following soft‑tissue sarcomas is most likely to show peripheral mineralizations?
- 1‑ Malignant fibrous histiocytoma
- 2‑ Synovial sarcoma
- 3‑ Rhabdomyosarcoma
- 4‑ Clear cell sarcoma
- 5‑ Epitheloid sarcoma
- Question 96.90
- Answer = 2
- Reference(s)
- Enzinger FM, Weiss SW: Soft tissue tumors. St. Louis, MO, Mosby, 1983.
- 96.91 A right‑handed, 27‑year‑old man is examined 1 year after an attempted wrist
- fusion for arthritis. The wrist is swollen and grossly unstable, and synovial tissue
- feels boggy and is somewhat warm to the touch. Radiographs are shown in
- Figure 26. Which of the following studies is most likely to confirm a diagnosis?
- 1‑ MRI scan of the wrist
- 2‑ MRI scan of the cervical spine
- 3‑ Analysis of HLA‑B27
- 4‑ Analysis of rheumatoid factor and erythrocyte sedimentation rate
- 5‑ Arthroscopy and synovial biopsy
- Fig. 26
- Question 96.91
- Answer = 2
- Reference(s)
- Parker RD, Froimson AI: Neurogenic arthropathy of the hand and wrist. J Hand Surg 1986;11A:706‑710.
- 96.92 An 8‑year‑old boy has had persistent pain in his right knee over a 5‑month
- period that is worse at night but responds to pediatric ibuprofen. Previous plain
- radiographs of the knee have been normal. A bone scan reveals the pattern
- shown in Figure 27a, plain radiographs of the right proximal femur are shown in
- Figures 27b and 27c, and a CT scan of the sclerotic area is shown in Figure 27d.
- A reasonable alternative to surgical treatment would be
- 1‑ nonsteroidal anti‑inflammatory treatment.
- 2‑ low‑dose methotrexate.
- 3‑ radiotherapy.
- 4‑ steroids.
- 5‑ vitamin D and calcium supplementation.
- Go to next slide for figures and answer link
- Fig. 27a Fig. 27d
- Fig. 27b
- Fig. 27c
- Question 96.92
- Answer = 1
- Reference(s)
- Wold LE, McLeod RA, Sim FH, et al: Atlas of Orthopedic Pathology. Philadelphia, PA, WB Saunders, 1990, pp 2‑7. Kneisl JS, Simon MA: Medical management compared with operative treatment for osteoid‑osteoma. J Bone Joint Surg 1992;74A:179‑185.
- 96.93 A patient with a left clavicle fracture treated by plate fixation complains of postoperative numbness of the left anterior chest wall below the clavicle. What is the most likely etiology of this complication?
- 1‑ Surgical laceration of the supraclavicular nerve
- 2‑ Traction injury to the cervical plexus
- 3‑ Traction injury to the brachial plexus, upper trunk
- 4‑ Traction injury to the thoracic segmental nerves
- 5‑ Compression injury to the infraclavicular nerve
- Question 96.93
- Answer = 1
- Reference(s)
- Jupiter JB, Leffert RD: Non‑union of the clavicle. Associated complications and surgical management. J Bone Joint Surg 1987;69A:753‑760. Poigenfurst J, Reiler T, Fischer W: Plating of fresh clavicular fractures: Experience with 60 operations. Unfallchirurgie 1988;14:26‑37. Last RJ: Anatomy: Regional and Applied, ed 6. New York, NY, Churchill‑Livingstone,1978, pp 363‑366.
- 96.94 When used as treatment of cerebral palsy, selective rhizotomy decreases spasticity
- by interruption of
- 1‑ afferent signals from intrafusal fibers.
- 2‑ efferent signals from anterior horn cells to extrafusal muscle fibers.
- 3- descending signals from the cortex.
- 4‑ cutaneous sensory afferents.
- 5‑ axons from Renshaw cells.
- Question 96.94
- Answer = 1
- Reference(s)
- Oppenheim WL: Selective posterior rhizotomy for spastic cerebral palsy: A review. Clin Orthop 1990;253:20‑29. Marty GR, Dias LS, Gaebler‑Spira D: Selective posterior rhizotomy and soft‑tissue procedures for the treatment of cerebral diplegia. J Bone Joint Surg 1995;77A:713‑718.
- 96.95 What is the most appropriate initial treatment of peroneal nerve palsy following
- total knee arthroplasty?
- 1‑ Immediate nerve exploration and release of adhesions
- 2‑ Administration of high dosages of IV dexamethasone
- 3‑ Flexion of the knee and release of any compressive dressings
- 4‑ Revision knee arthroplasty with smaller components
- 5‑ Casting of the knee in full extension
- Question 96.95
- Answer = 3
- Reference(s)
- Kasser JR (ed): Orthopaedic Knowledge Update 5. Rosemont, IL, American Academy of Orthopaedic Surgeons, 1996, pp 481‑502.
- 96.96 While waiting for induction of anesthesia during a routine hip replacement arthroplasty, the surgeon writes routine postoperative orders and a brief operative note describing the anticipated procedure on the same sheet as the written admission orders. The patient arrests intraoperatively, and the procedure is not completed. What should be done with the operative note and orders?
- 1‑ Remove the sheet from the record and write new ones on clean sheets.
- 2‑ Leave the original note and orders and write new ones below the originals.
- 3‑ Cross out the original note and orders so that they cannot be read and write
- new ones.
- 4‑ Cross out the original note and orders so that they are still legible, append a
- note of explanation, and write new ones.
- 5- Notify risk management before making additional entries in the record.
- Question 96.96
- Answer = 4
- Reference(s)
- AAOS Committee on Professional Liability: Medical Malpractice: A Primer for Orthopaedic Residents and Fellows. Rosemont, IL, American Academy of Orthopaedic Surgeons, 1993, p 23.
- 96.97 A patient with numbness of the anterior thigh and shin, weakness of the
- quadriceps, and loss of patella reflex has involvement of what nerve root?
- 1‑ L1
- 2‑ L2
- 3‑ L3
- 4‑ L4
- 5‑ L5
- Question 96.97
- Answer = 4
- Reference(s)
- Rothman RH, Simeone FA, Bernini PM: Lumbar disc disease, in Rothman RH, Simeone FA (eds): The Spine, ed 2. 1982, vol 1, pp 508‑521.
- 96.98 Which of the following occurs during contraction of skeletal muscle fibers?
- 1‑ Significant decrease in calcium concentration in the muscle cell cytoplasm
- 2‑ Increase in the distance between each contractile unit of the muscle
- 3‑ Influx of extracellular calcium into the muscle cell via membrane channels
- 4‑ Myosin thick filaments and actin thin filaments slide in relation to each other
- 5‑ Shortening of myosin thick filaments and actin thin filaments
- Question 96.98
- Answer = 4
- Reference(s)
- Huxley HE: Electron microscope studies on the structure of natural and synthetic protein filaments from striated muscle. J Mol Biol 1963;7:281‑308. Huxley HE: The mechanism of muscular contraction. Science 1969;164:1356‑1365.
- 96.99 Indirect reduction of the intracanal fragment secondary to a burst fracture is
- accomplished by which of the following forces?
- 1‑ Flexion
- 2‑ Distraction
- 3‑ Compression
- 4‑ Extension
- 5‑ Rotation
- Question 96.99
- Answer = 2
- Reference(s)
- Fredrickson BE, Edwards WT, Rauschning W, et al: Vertebral burst fractures: An experimental morphologic, and radiographic analysis. Spine 1992;17:1012‑1021. Fredrickson BE, Mann KA, Yuan HA, et al: Reduction of the intracanal fragment in experimental burst fractures. Spine 1988;13:267‑271.
- 96.100 Which of the following is the greatest risk factor for contracting non‑A, non‑B
- hepatitis?
- 1‑ IV drug use
- 2‑ Close personal contact
- 3‑ Transfusion
- 4‑ Hemodialysis
- 5‑ Inadvertent needle sticks in medical personnel
- Question 96.100
- Answer = 1*
- Reference(s)
- Frymoyer JW (ed): Orthopaedic Knowledge Update 4. Rosemont, IL, American Academy of Orthopaedic Surgeons, 1993, pp 155‑168. *item deleted after statistical review
- 96.101 An eccentric, isotonic muscle contraction occurs when a contracting muscle
- undergoes which of the following processes?
- 1‑ Shortens against constant resistance
- 2‑ Shortens against variable resistance
- 3‑ Lengthens with constant velocity
- 4‑ Lengthens against constant resistance
- 5‑ Remains at constant length against variable resistance
- Question 96.101
- Answer = 4
- Reference(s)
- Cahalan TD, Johnson ME, Liu S, et al: Quantitative measurement of hip strength in different age groups. Clin Orthop 1989;246:136‑145.
- 96.102 A 20‑year‑old man has a severely comminuted closed tibial plafond fracture with
- a segmental fibular fracture.‑ The goal of surgical management at this time should
- consist of
- 1‑ maintenance of the tibial length and alignment.
- 2‑ rigid fixation of the tibial plafond.
- 3‑ rigid fixation of the fibula and tibial plafond.
- 4‑ restoration of fibular length and alignment.
- 5‑ restoration of the tibial‑fibular syndesmosis and ankle arthrodesis.
- Question 96.102
- Answer = 1
- Reference(s)
- Bourne RB, Rorabeck CH, Macnab J: Intra‑articular fractures of the distal tibia: The pilon fracture. J Trauma 1983;23:591‑596. Russell TA, Taylor JC, LaVelle DG: Fractures of the tibia and fibular, in Rockwood CA Jr, Green DP, Bucholz RW (eds): Fractures in Adults, ed 3. Philadelphia, PA, JB Lippincott, 1991, pp 1915‑1982. Mast JW, Spiegel PG. Pappas JN: Fractures of the tibial pilon. Clin Orthop 1988;230:68‑82. Bonar SK, Marsh JL: J Am Acad Orthop Surg 1994;2:297‑305.
- 96.103 A 44‑year‑old woman has a symptomatic bunion that has failed to respond to all conservative treatment measures. A standing anteroposterior radiograph of the foot reveals a 35‑degree hallux valgus angle, an 18‑degree intermetatarsal angle, and an incongruent joint without arthrosis. Which of the following procedures is indicated?
- 1‑ Chevron osteotomy of the distal first metatarsal
- 2‑ Proximal phalangeal osteotomy and exostectomy
- 3‑ Distal soft‑tissue release with proximal metatarsal osteotomy
- 4‑ First metatarsophalangeal fusion
- 5‑ Resection arthroplasty of the first metatarsophalangeal joint
- Question 96.103
- Answer = 3
- Reference(s)
- Mann RA: Disorders of the first metatarsophalangeal joint. J Am Acad Orthop Surg 1995;3:34‑43. Brodsky HW: The diabetic foot, in Mann RA, Coughlin MJ: Surgery of the Foot and Ankle, ed 6. St Louis, MO, CV Mosby, 1993, pp 877‑958.
- 96.104 A 9‑year‑old girl has had an aching pain in her left leg for the past 4 months.
- Plain radiographs of the tibia are shown in Figures 28a and 28b, a coronal
- T,‑weighted MRI scan is shown in Figure 28c, and a sagittal T2 weighted image
- is shown in Figure 28d. A bone scan shows this to be the only area with
- increased uptake. An incisional biopsy yields histology as shown in Figure 28e.
- An electron microscopic study is shown in Figure 28f. What is the most likely
- diagnosis?
- 1‑ Primitive neuroectodermal tumor (PNET)
- 2‑ Eosinophilic granuloma (Langerhans histiocytosis)
- 3‑ Metastatic neuroblastoma
- 4‑ Ewing sarcoma
- 5‑ Osteosarcoma
- Figure 28a
- Figure 28b
- Go to next slide for remaining figures and answer link
- Fig. 28c Fig. 28d
- Fig. 28e
- Fig. 28f
- Question 96.104
- Answer = 2
- Reference(s)
- Langerhans cell granulomatosis; langerhans cell histiocytosis; solitary and multifocal eosinophilic granuloma of bone, in Huvos AG (ed): Bone Tumors, Diagnosis, Treatment and Prognosis, ed 2. Philadelphia, PA, WB Saunders, 1991, pp 695‑711.
- 96.105 During a surgical approach to the lateral aspect of the distal fibula, what two
- nerves are potentially at risk for injury?
- 1‑ Peroneal and tibial
- 2‑ Sural and lateral plantar
- 3‑ Superficial peroneal and sural
- 4‑ Lateral plantar and superficial peroneal
- 5‑ Deep peroneal and superficial peroneal
- Question 96.105
- Answer = 3
- Reference(s)
- Huene DB, Bunnell WP: Operative anatomy of nerves encountered in the lateral approach to the distal part of the fibula. J Bone Joint Surg 1995;77A:1021‑1024.
- 96.106 During what age range (in years) does the medial clavicular epiphysis fuse with the
- clavicle?
- 1‑ 11 to 12
- 2‑ 13 to 15
- 3‑ 16 to 18
- 4‑ 19 to 21
- 5‑ 22 to 25
- Question 96.106
- Answer = 5
- Reference(s)
- O'Brien SJ, Arnoczky SP, Warren RD, et al: Developmental anatomy of the shoulder and anatomy of the glenohumeral joint, in Rockwood CA, Matsen FA (eds): The Shoulder. Philadelphia, PA, WB Saunders, 1990, vol 1, pp 1‑33.
- 96.107 A 10‑year‑old boy with hemophilia A has had a swollen left knee for the past 3 months,
- despite periodic home administration of factor VIII. A radiograph shows slight epiphyseal
- enlargement and osteopenia. Recommended management should consist of
- 1‑ checking inhibitor status.
- 2‑ aspiration, compression, and intravenous factor administration.
- 3‑ arthrodesis, sparing the physis.
- 4‑ infra‑articular factor administration.
- 5‑ epiphysiodesis.
- Question 96.107
- Answer = 1
- Reference(s)
- Houghton GR, Duthie RB: Orthopedic problems in hemophilia. Clin Orthop 1979;138:197‑216.
- 96.108 Which of the following is a characteristic of articular cartilage in osteoarthritis?
- 1‑ Reduced water content
- 2‑ Reduced proteoglycan in aggregating form
- 3‑ Increased glycosaminoglycan length on aggrecan
- 4‑ Absent hyaluronate
- 5‑ Absent link protein
- Question 96.108
- Answer = 2
- Reference(s)
- Simon SR (ed): Orthopaedic Basic Science. Rosemont, IL, American Academy of Orthopaedic Surgeons, 1994, pp 1‑44.
- 96.109 During normal spine motion, each motion unit can have how many degrees
- of freedom?
- 1‑ 1
- 2‑ 2
- 3‑ 4
- 4‑ 6
- 5‑ 12
- Question 96.109
- Answer = 4
- Reference(s)
- Kapandju IA: The trunk and the vertebral column, in The Physiology of the Joint. Edinburgh and London, Churchill‑Livingstone, 1974, pp 105‑109.
- 96.110 A 65‑year‑old man complains of burning and dysesthesia in a stocking glove distribution
- from the ankles down that is unaffected by activity. What is the most likely cause of this
- patient's condition?
- 1‑ Hypertension
- 2‑ Degenerative lumbar stenosis
- 3‑ Degenerative lumbar spondylolisthesis
- 4‑ Ulcerative colitis
- 5‑ Diabetes mellitus
- Question 96.110
- Answer = 5
- Reference(s)
- Mooney V: Differential diagnosis of low back disorders: Principles of classification, in Frymoyer JW (ed): The Adult Spine: Principles and Practice. New York, NY, Raven Press, 1991, vol 2, pp 1551‑1566.
- 96.111 After undergoing revision total hip arthroplasty through a posterior approach, a
- patient is unable to dorsiflex the foot on the operated side. This is most likely
- caused by what perioperative factor?
- 1‑ Positioning of the lumbar spine, causing radiculopathy
- 2‑ Tension on the sciatic nerve‑peroneal division
- 3‑ Postoperative hematoma compressing the sciatic nerve‑tibial division
- 4‑ Stretch injury to the femoral nerve
- 5‑ Compression injury to the obturator nerve
- Question 96.111
- Answer = 2
- Reference(s)
- Shaw JA, Greer RB III: Complications of total hip replacement, in Epps CH Jr (ed): Complications in Orthopaedic Surgery, ed 3. Philadelphia, PA, JB Lippincott, 1994, vol 2, pp 1013‑1056. Schmalzried TP, Amstutz HC, Dorey FJ: Nerve palsy associated with total hip replacement: Risk factors and prognosis. J Bone Joint Surg 1991;73A:1074‑1080.
- 96.112 What genicular artery is the main blood supply to the cruciate ligaments?
- 1‑ Supreme
- 2‑ Lateral
- 3‑ Middle
- 4‑ Medial inferior
- 5‑ Medial superior
- Question 96.112
- Answer = 3
- Reference(s)
- Stuart M: The knee/anatomy, in Money B (ed): Joint Replacement Arthroplasty. New York, NY, Churchill‑Livingstone, 1991, p 949.
- 96.113 A 28‑year‑old woman injures her left ankle and the resulting distal fibula fracture
- is treated with open reduction and internal fixation. Two days later, the incision is
- healing well, and the ankle is placed in well‑molded below‑knee cast. A mortise
- radiograph is shown in Figure 29. The most appropriate next step in management
- is
- 1‑ application of a new cast with the ankle in less equinus.
- 2‑ closed reduction and placement in a long leg, bent knee cast.
- 3‑ placement of a lag screw through the most distal hole in the plate.
- 4‑ repair of the superficial deltoid ligament.
- 5‑ reduction and placement of a syndesmosis screw.
- Fig. 29
- Question 96.113
- Answer = 5
- Reference(s)
- Trafton PG, Bray TJ, Simpson LA: Fractures and soft tissue injuries of the ankle, in Browner BD, Jupiter JB, Levine AM, et al (eds): Skeletal Trauma: Fractures, Dislocations, Ligamentous Injuries. Philadelphia, PA, WB Saunders, 1992, pp 1871‑1958.
- 96.114 A 4‑year‑old boy with no history of injury has had an intermittent limp for the past 6 months. He has no complaints and participates in all activities without apparent problems. Examination reveals internal rotation of both hips to 50 degrees, abduction to 50 degrees, and normal flexion with no contracture. A radiograph is shown in Figure 30. Recommended treatment should be
- 1‑ observation.
- 2‑ use of crutches and traction at night.
- 3‑ use of a Scottish Rite brace.
- 4‑ proximal femoral varus osteotomy.
- 5‑ innominate osteotomy.
- Fig. 30
- Question 96.114
- Answer = 1
- Reference(s)
- Green NE, Beauchamp RD, Griffin PP: Epiphyseal extrusion as a prognostic index in Legg‑Calve‑Perthes disease. J Bone Joint Surg 1981;63A:900‑905. Wenger DR, Ward WT, Herring IT: Legg‑Calve‑Perthes disease. J Bone Joint Surg 1991;73A:778‑788.
- 96.115 A knee arthrodesis is preferred over prosthetic reimplantation after removal of an
- infected total knee arthroplasty in a patient with which of the following
- conditions?
- 1‑ Low functional demands
- 2‑ Multiple joint arthrosis in the lower extremities
- 3‑ Presence of mixed Gram‑positive organisms that cause infection
- 4‑ Gross instability
- 5‑ Extensor mechanism disruption
- Question 96.115
- Answer = 5
- Reference(s)
- Rand JA: Sepsis following total knee arthroplasty, in Rand JA (ed): Total Knee Arthroplasty. New York, NY, Raven Press, 1993, p 363.
- 96.116 In hyaline cartilage, the proteoglycans are attached to hyaluronic acid via
- 1‑ chondroitin sulfate.
- 2‑ keratan sulfate.
- 3‑ link protein.
- 4‑ peptide bonds.
- 5‑ disulfide bonds.
- Question 96.116
- Answer = 3
- Reference(s)
- Buckwalter JA, Rosenberg LC: Electron microscopic studies of cartilage proteoglycans: Direct evidence for the variable length of the chondroitin sulfate‑rich region of proteoglycan subunit core protein. J Biol Chem 1982;257:9830‑9839. Simon SR (ed): Orthopaedic Basic Science. Rosemont, IL, American Academy of Orthopaedic Surgeons, 1994, p 9. Kimura JH, Hardingham TE, Hascall VC: Assembly of newly synthesized proteoglycan and link protein into aggregates in cultures of chondrosarcoma chondrocytes. J Biol Chem 1980;255:7134‑7143.
- 96.117 A 14‑year‑old boy has a large high‑grade osteosarcoma of the distal femur that
- measures 6 x 9 cm on the anteroposterior radiograph. There is a large posterior
- soft‑tissue extension that does not engulf the neurovascular bundle. The chest
- radiograph and CT scan of the chest are normal. According to the staging
- system of the Musculoskeletal Tumor Society, this patient's tumor would be
- classified as what stage?
- 1‑ IB
- 2‑ IIA
- 3‑ IIB
- 4‑ 2
- 5‑ 3
- Question 96.117
- Answer = 3
- Reference(s)
- Simon SR (ed): Orthopaedic Basic Science. Rosemont, IL. American Academy of Orthopaedic Surgeons, 1994, pp 219‑276. Enneking WF, Spanier SS, Goodman MA: Current concepts review: The surgical staging of musculoskeletal sarcoma. J Bone Joint Surg 1980;62A:1027‑1030
- 96.118 Which of the following surgical approaches in primary total hip arthroplasty is
- associated with the highest incidence of hip dislocation?
- 1‑ Anterior
- 2‑ Anterolateral
- 3‑ Posterior
- 4‑ Transtrochanteric
- 5‑ Lateral
- Question 96.118
- Answer = 3
- Reference(s)
- Morrey B: Perioperative factors: Surgical approach, in Morrey B (ed): Joint Replacement Arthroplasty. New York, NY, Churchill‑Livingstone, 1991, p 853.
- 96.120 Which of the following anatomic structures is at greatest risk for injury when a
- posterior portal is used in shoulder arthroscopy?
- 1‑ Infrascapular nerve
- 2‑ Axillary nerve
- 3‑ Anterior humeral circumflex artery
- 4‑ Suprascapular artery
- 5‑ Recurrent branch of the axillary artery
- Question 96.120
- Answer = 1*
- Reference(s)
- Kasser JR (ed): Orthopaedic Knowledge Update 5. Rosemont, IL, American Academy of Orthopaedic Surgeons, 1996, pp 245‑257. *item deleted after statistical review
- 96. 121 A 60‑year‑old woman has disabling symptoms of neurogenic claudication and
- radiographs that reveal a grade I degenerative spondylolisthesis L4‑5 with 75% of
- normal disk height. Treatment should consist of
- 1‑ decompression at L4 and L5.
- 2‑ decompression at L4 and L5, with fusion at L4‑5.
- 3‑ posterior fusion at L4 and L5.
- 4‑ posterior fusion at L4 to S 1.
- 5‑ anterior and posterior fusion at L4‑5.
- answer
- back
- Question 96.121
- Answer = 2
- back to this question
- next question
- Reference(s)
- Spengler DM: Lumbar decompression for spinal stenosis: Surgical indications and technique, in Frymoyer JW (ed): The Adult Spine: Principles and Practice. New York, NY, Raven Press, 1991, vol 2, pp 1811‑1820.
- 96.122 A 45‑year‑old man undergoes atraumatic reduction of a dislocated shoulder in the
- emergency department. Twelve weeks after injury, passive range of motion is
- normal, but the patient is still unable to actively abduct the shoulder beyond 40
- degrees. The most likely diagnosis is
- 1‑ disuse atrophy.
- 2‑ rotator cuff tear.
- 3‑ frozen shoulder.
- 4‑ axillary nerve palsy.
- 5‑ recurrent subluxation of the shoulder.
- answer
- back
- Question 96.122
- Answer = 2
- back to this question
- next question
- Reference(s)
- Neviaser RJ, Neviaser TJ, Neviaser JS: Anterior dislocation of the shoulder and rotator cuff rupture. Clin Orthop 1993;291:103‑106.
- 96.123 Three months after undergoing open reduction and plate fixation of a distal fibula fracture, a patient complains of dysesthesias on the top of the foot. Sensitivity to light touch is decreased on the dorsum of the foot, and motor examination is normal. The most likely diagnosis is
- 1‑ residual effects of lateral compartment syndrome.
- 2‑ reflex sympathetic dystrophy.
- 3‑ complete peroneal nerve injury at the level of the fibular neck.
- 4‑ surgical injury of the superficial peroneal nerve.
- 5‑ entrapment of the sural nerve at the distal fibula.
- Question 96.123
- Answer = 4
- Reference(s)
- Huene DB, Bunnell WP: Operative anatomy of nerves encountered in the lateral approach to the distal part of the fibula. J Bone Joint Surg 1995;77A:1021‑1024.
- 96.124 A 35‑year‑old man is evaluated 9 months after sustaining a laceration of both
- flexor tendons over the proximal interphalangeal (PIP) crease of his index finger.
- Active range of motion is 0 to 90 degrees, 0 to 20 degrees, and 0 to 15 degrees in
- the metacarpophalangeal, PIP, and distal interphalangeal (DIP) joints, respectively;
- passive range of motion is 0 to 90 degrees, 0 to 95 degrees, and 0 to 50 degrees,
- respectively. Both tendons were repaired the day of injury, and postoperative
- management consisted of dynamic traction with an extension block splint. The
- physician should recommend
- 1‑ flexor tendon graft.
- 2‑ staged tendon reconstruction.
- 3‑ tenolysis of the sublimis and profundus.
- 4‑ proximal interphalangeal capsulotomy.
- 5‑ secondary tenorrhaphy.
- Question 96.124
- Answer = 3
- Reference(s)
- Schneider LH, Hunter JM: Flexor tendons: Late reconstruction, in Green DP (ed): Operative Hand Surgery, ed 3. New York, NY, Churchill‑Livingstone, 1993, vol 2, pp 1853‑1924.
- 96.125 A 19‑year‑old man who plays basketball is 66" tall and has long, slender limbs and digits,
- myopia, and a pectus excavatum. What is the most important finding to confirm if the
- presence of Marfan's syndrome is being considered?
- 1‑ Unstable patella
- 2‑ Maintenance of visual acuity
- 3‑ Enlarged aortic root
- 4‑ Scoliosis
- 5‑ Increased metacarpal index
- Question 96.125
- Answer = 3
- Reference(s)
- Pyeritz RE: Marfan syndrome, in Royce PM, Steinman B (eds): Connective Tissue and Its Heritable Disorders. New York, NY, Wiley‑Less, 1993.
- 96.126 A 65‑year‑old man is brought to the emergency department after a motor vehicle
- accident. Neurologic examination is normal. Radiographs are shown in Figures
- 31a and 31 b. What is the most likely diagnosis?
- 1‑ Unilateral facet dislocation
- 2‑ Burst fracture of C6
- 3‑ Bilateral perched facets
- 4‑ Bilateral fracture‑dislocations of the facets
- 5‑ Previous ligamentous injury
- Fig. 31a
- Fig. 31b
- answer
- back
- Question 96.126
- Answer = 3
- back to this question
- next question
- Reference(s)
- McAfee PC: Cervical spine trauma, in Frymoyer JW (ed): The Adult Spine: Principles and Practice. New York, NY, Raven Press, 1991, pp 1063‑1106.
- 96.127 A 45‑year‑old man, a nonsmoker in excellent health, is evaluated after a
- traumatic thumb amputation (Figures 32a and 32b). What is the best treatment
- method for this patient's injury?
- 1‑ Tubed groin flap and iliac crest graft
- 2‑ Thumb web space deepening
- 3‑ Index finger pollicization
- 4‑ Toe to hand transfer
- 5‑ Metacarpal lengthening
- Fig 32a Fig 32b
- Question 96.127
- Answer = 4
- Reference(s)
- Wei FC, Chen HC, Chuang CC, et al: Microsurgical thumb reconstruction with toe transfer. Selection of various techniques. Plast Reconstr Surg 1994;93:345‑357. Lister G: The choice of procedure following thumb amputation. Clin Orthop 1985;195:45‑51.
- 96.128 A 54‑year‑old woman sustained a displaced femoral neck fracture after a minor
- fall. Additional radiographs are shown in Figures 33a through 33c, because of the
- concern about associated injuries. The underlying process that most likely
- contributed to her hip fracture is
- 1‑ hypophosphatasia.
- 2‑ osteomalacia.
- 3‑ postmenopausal osteoporosis.
- 4‑ multiple myeloma.
- 5‑ Paget's disease.
- Fig. 33a
- Fig. 33b Fig. 33c
- Question 96.128
- Answer = 4
- Reference(s)
- Wold LE, McLeod RA, Sim FH, et al: Atlas of Orthopedic Pathology. Philadelphia, PA, WB Saunders, 1990,pp 150‑155. Frassica FJ, Beabout JW, Until KK, et al: Myeloma of bone. Orthopedics 1985;8:1184‑1186. Myeloma, in Dahlin DC, Unni KK (ed): Bone Tumors: General Aspects and Data on 8,542 Cases, ed 4. Springfield, IL, Charles C. Thomas Publishers, 1986, pp 193‑207.
- 96.129 A 38‑year‑old woman who had a spinal fusion 5 years ago has mild thoracic pain. A
- radiograph is shown in Figure 34. The broken rod shown in the radiograph is most likely
- caused by
- 1‑ chronic infection.
- 2‑ progression of the curvature.
- 3‑ pseudarthrosis.
- 4‑ flatback syndrome.
- 5‑ hook pullout.
- Fig. 34
- answer
- back
- Question 96.129
- Answer = 3
- back to this question
- next question
- Reference(s)
- Transfeldt EE: Complications of treatment, in Lonstein JE, Bradford DS, Winter RB, et al (eds): Moe's Textbook of Scoliosis, ed 3. Philadelphia, PA, WB Saunders, pp 451‑481.
- 96.130 In analyzing joint fluid under polarized light, which disease process is
- characterized by short, rhomboidal‑shaped crystals that are weakly birefringent
- and appear blue when parallel to the microscopic compensator?
- 1‑ Gout
- 2‑ Ochronosis
- 3‑ Chondrocalcinosis
- 4‑ Hemachromatosis
- 5‑ Tumoral calcinosis
- Question 96.130
- Answer = 3
- Reference(s)
- Frymoyer JW (ed): Orthopaedic Knowledge Update 4. Rosemont, IL, American Academy of Orthopaedic Surgeons, 1993, pp 89‑106.
- 96.131 A 16‑year‑old girl has chronic pain in her right hip. A frog lateral radiograph is
- shown in Figure 35a, and T2 weighted coronal MRI scan is shown in Figure
- 35b. A gross histologic specimen obtained at the time of hip arthrotomy is
- shown in Figure 35c. The most likely diagnosis is
- 1‑ synovial chondromatosis.
- 2‑ synovial sarcoma.
- 3‑ pigmented villonodular synovitis.
- 4‑ rheumatoid synovitis.
- 5‑ chondrocalcinosis.
- Fig. 35b
- Fig. 35a
- Fig. 35c
- Question 96.131
- Answer = 1
- Reference(s)
- Milgram JW: Synovial ostoechondromatosis: A histopathological study of thirty cases. J Bone Joint Surg 1977;59A:792‑801.
- 96.132 After replacing the proximal femur with an endoprosthesis for hip fracture in an
- elderly man prone to heterotopic ossification and on warfarin for DVT
- prophylaxis, recommended treatment to prevent heterotopic bone formation
- should be
- 1‑ aggressive physical therapy and range of motion exercises.
- 2‑ administration of indomethacin.
- 3‑ administration of diphosphonates.
- 4‑ low‑dose irradiation.
- 5‑ early surgical excision.
- Question 96.132
- Answer = 4
- Reference(s)
- Pellegrini VD Jr. Konski AA, Gastel JA, et al: Prevention of heterotopic ossification with irradiation after total hip arthroplasty: Radiation therapy with a single dose of eight hundred centigray administered to a limited field. J Bone Joint Surg 1992;74A:186‑200. Ayers DC, Pellegrini VD Jr, Evarts CM, et al: Prevention of heterotopic ossification in high‑risk patients by radiation therapy. Clin Orthop 1991;263:87‑93. Coventry MB, Scanlon PW: The use of radiation to discourage ectopic bone: A nine‑year study in surgery about the hip. 1 Bone Joint Surg 1981;63A:201‑208.
- 96.133 During a surgical approach to the hip between the tensor fascia lata and the
- sartorius, a plane is made between what two nerves?
- 1‑ Superior gluteal and femoral
- 2‑ Superior gluteal and inferior gluteal
- 3‑ Inferior gluteal and femoral
- 4‑ Femoral and lateral femoral cutaneous
- 5‑ Femoral and obturator
- Question 96.133
- Answer = 1
- Reference(s)
- Hoppenfeld S, deBoer P: The hip, in Hoppenfeld S, deBoer P (eds): Surgical Exposures in Orthopaedics: The Anatomic Approach. Philadelphia, PA, JB Lippincott, 1984, pp 301‑356.
- 96.134 A 12‑month‑old boy with achondroplasia has a hump in the middle to lower part
- of the back. The child is just beginning to sit; he does not pull to stand. A sitting
- lateral radiograph of the spine reveals a kyphosis of 40 degrees from T12 to L2.
- There is mild rounding of the anterosuperior and anteroinferior corners of the first
- lumbar vertebra. Recommended treatment should consist of
- 1‑ use of a thoracolumbosacral orthosis.
- 2‑ posterior spinal fusion from T12 to L2.
- 3‑ anterior spinal fusion from T12 to L2.
- 4‑ corpectomy of Ll.
- 5‑ observation.
- Question 96.134
- Answer = 5
- Reference(s)
- Bassett GS. Scott CI Jr: The osteochondrodysplasias, in Morrissy RT (ed): Lovell and Winter's Pediatric Orthopaedics, ed 3. Philadelphia, PA, JB Lippincott, 1990, vol 1, pp 91‑142.
- 96.135 What is the single strongest deforming force leading to subluxation and
- dislocation of the second metatarsophalangeal joint?
- 1‑ Extensor digitorum longus
- 2‑ Extensor digitorum brevis
- 3‑ Flexor digitorum longus
- 4‑ Two dorsal interosseous muscles
- 5‑ Lumbrical muscle
- Question 96.135
- Answer = 1
- Reference(s)
- Fortin PT, Myerson MS: Second metatarsophalangeal joint instability. Foot Ankle Int 1995;16:306‑313. Coughlin MJ: Subluxation and dislocation of the second metatarsophalangeal joint Orthop Clin North Am 1989;20:535‑551.
- 96.136 A 50‑year‑old man with a malunion of a Colles' fracture presents with a painful
- click on ulnar deviation. Radiographs are most likely to reveal what condition?
- 1‑ Ulnar translation of the carpus•
- 2‑ Ulnar negative variance
- 3‑ Lunotriquetral widening
- 4‑ Ununited ulnar styloid
- 5‑ Dorsal intercalated segmental instability
- Question 96.136
- Answer = 5
- Reference(s)
- Taleisnik J, Watson HK: Midcarpal instability caused by malunited fractures of the distal radius. J Hand Surg 1984;9A:350‑357.
- 96.137 An abducted posture of the small finger, following complete ulnar nerve
- transection just proximal to the wrist, is attributed to what factor?
- 1‑ Unopposed pull of the extensor digiti minimi
- 2‑ Unopposed pull of the third volar interosseous
- 3‑ Median innervation of the abductor digiti minimi
- 4‑ Paralysis of the lumbrical to the small finger
- 5‑ Ulnar moment of the extrinsic flexors to the small finger
- Question 96.137
- Answer = 1
- Reference(s)
- Blacker GJ, Lister GD, Kleinert HE: The abducted little finger in low ulnar nerve palsy. J Hand Surg 1976;1A:190‑196.
- 96.138 Insufficiency of which of the following structures leads to primary
- osteoarthritis of the trapeziometacarpal joint?
- 1‑ Dorsal radial capsule
- 2‑ Posterior oblique ligament
- 3‑ Anterior intermetacarpal ligament
- 4‑ Ulnar collateral ligament
- 5‑ Palmar beak ligament
- Question 96.138
- Answer = 5
- Reference(s)
- Pellegrini VD Jr: Osteoarthritis of the trapeziometacarpal joint: The pathophysiology of articular cartilage degeneration. I: Anatomy and pathology of the aging joint J Hand Surg 1991;16A:967‑974.
- 96.139 What property of methylmethacrylate debris is the most significant factor
- leading to bone resorption?
- 1‑ Type of cement
- 2‑ Prostaglandin E2 concentration
- 3‑ Cement particle size
- 4‑ Presence of antibiotics in the cement
- 5‑ Cement viscosity
- Question 96.139
- Answer = 3
- Reference(s)
- Horowitz SM, Doty SB, Lane JM, et al: Studies of the mechanism by which the mechanical failure of polymethylmethacrylate leads to bone resorption. J Bone Joint Surg 1993;75A;802‑813.
- 96.140 When performing a chevron osteotomy with the apex in the middle of the first
- metatarsal head, it is important to avoid overpenetration of the lateral cortex of
- the first metatarsal with the oscillating saw to prevent injury to what structure?
- 1‑ Perforators into the metatarsal head
- 2‑ Superficial branch of the medial plantar artery
- 3‑ First dorsal metatarsal artery
- 4‑ First plantar metatarsal artery
- 5‑ Dorsalis pedis artery
- Question 96.140
- Answer = 3
- Reference(s)
- Mann RA, Coughlin MJ: Adult hallux valgus, in Mann RA, Coughlin MJ (eds): Surgery of the Foot and Ankle, ed 6. St Louis, MO, CV Mosby, 1993, pp 167‑296. Jones KJ, Feiwell LA, Freedman EL, et al: The effect of chevron osteotomy with lateral capsular release on the blood supply to the first metatarsal head. J Bone Joint Surg 1995;77A:197‑204.
- 96.141 Aminoglycoside antibiotics exhibit their bactericidal effect on bacteria by what
- mechanism?
- 1‑ Stimulating plasmid function
- 2‑ Binding cytoplasmic ribosomes
- 3‑ Disrupting cell wall synthesis
- 4‑ Blocking spheroplast growth
- 5‑ Forming complexes with bacterial DNA
- Question 96.141
- Answer = 2
- Reference(s)
- Sande MA, Mandell GL: Antimicrobial agents: The aminoglycosides, in Gilman, Goodman, Gilman (eds): The Pharmacological Basis of Therapeutics, ed 6. New York, NY, MacMillan Publishing Co, 1980, pp 1162‑1180.
- 96.142 A 2‑week‑old infant is undergoing evaluation of a left hip dislocation. The history reveals a premature, emergency delivery in which the infant sustained trauma. In addition, the infant has required ventilatory support. Examination reveals few spontaneous movements of the left hip, some swelling of the left proximal thigh, and pain on motion of the left hip. Radiographs show lateral dislocation of the left hip and early periosteal reaction around the left proximal femur. Management should include
- 1‑ closed reduction and a spica cast.
- 2‑ a Pavlik harness.
- 3‑ overhead traction.
- 4‑ hip aspiration.
- 5‑ a bone scan.
- Question 96.142
- Answer = 4
- Reference(s)
- Morrissy RT: Bone and joint infections, in Morrissy RT (ed): Lovell and Winter's Pediatric Orthopaedics, ed 3. Philadelphia, PA, JB Lippincott, 1990, pp 539‑561.
- 96.143 What is the most important factor in designing a Z‑plasty?
- 1‑ Equal length of all limbs
- 2‑ A central limb that is twice the length of the side limbs
- 3‑ Length of the side limbs twice that of the central limb
- 4‑ A 45‑degree angle between limbs
- 5‑ A 75‑degree angle between limbs
- Question 96.143
- Answer = 1
- Reference(s)
- Furnas DW: Z‑plasties and related procedures for the hand and upper limb. Hand Clin 1985;1:649‑665. Lister GD: Skin flaps, in Green DP (ed): Operative Hand Surgery, ed 3. New York, NY, Churchill‑Livingstone, 1993, vol 2, pp 1741‑1822.
- 96.144 A 42‑year‑old man presents with a long history of right arm pain over the thumb
- and forearm and the recent onset of morning stiffness in his legs. Physical
- examination reveals sustained clonus in the left leg, several beats of clonus in the
- right leg, but no demonstrable weakness of his arms or legs. A plain radiograph
- and MRI scans are shown in Figures 36a through 36c. Treatment should consist of
- 1‑ a physical therapy strengthening program:
- 2‑ epidural steroid injection.x
- 3‑ posterior cervical fusion
- 4‑ cervical laminectomy.
- 5‑ anterior cervical vertebrectomy and fusion.
- Fig. 36a
- Fig. 36b Fig. 36c
- Question 96.144
- Answer = 5
- Reference(s)
- Hanai K, Fujiyoshi F, Kamei K: Subtotal vertebrectomy and spinal fusion for cervical spondylolytic myelopathy. Spine 1986;11:310‑315. Whitecloud TS III: Anterior surgery for cervical spondylotic myelopathy: Smith‑Robinson, Cloward, and vertebrectomy. Spine 1988;13:861‑863.
- 96.145 Which of the following identifies the capacity of the bone to absorb energy
- as illustrated in Figure 37?
- 1‑ Slope of line a
- 2‑ Slope of line b
- 3‑ Slope of line a minus slope of line b
- 4‑ Area under the curve
- 5‑ Stress at the intersection of lines a and b
- Fig. 37
- Question 96.145
- Answer = 4
- Reference(s)
- Hipp JA, Cheal EJ, Hayes WC: Biomechanics of fractures, in Browner BD, Jupiter JB (eds): Skeletal Trauma: Fractures, Dislocations, Ligamentous Injuries. Philadelphia, PA, WB Saunders, 1992, pp 95‑126.
- 96.146 Which of the following radiographic signs is present if there is instability of
- Lisfranc's joint?
- 1‑ The medial border of the second metatarsal aligns with the medial border of
- the middle cuneiform.
- 2‑ The medial border of the fourth metatarsal aligns with the medial border of
- the cuboid on the oblique view.
- 3‑ The space between the proximal first and second metatarsals measures 3 mm
- on a standing anteroposterior radiograph.
- 4‑ The first metatarsal is aligned with the borders of the medial cuneiform.
- 5‑ The first intermetatarsal space aligns with the intertarsal space between the
- medial and middle cuneiforms.
- Question 96.146
- Answer = 3
- Reference(s)
- Schenck RC, Heckman JD: Fractures and dislocations of the forefoot: Operative and non‑operative treatment. J Am Acad Orthop Surg 1995;3:70‑78. Faciszewski T, Burks RT, Manastet BJ: Subtle injuries to the Lisfranc joint. J Bone Joint Surg 1990;72A:1519‑1522.
- 96.147 A 20‑year‑old man injures his right knee while playing football. Examination shows a large effusion and increased anterior‑posterior translation of the tibia in relation to the femur. Figures 38a (knee tested at 30 degrees of flexion) and 38b (knee tested at 90 degrees of flexion) were obtained during an examination under anesthesia. Results of the examination indicate injury to which of the following structures?
- 1‑ Anterior cruciate ligament
- 2‑ Anterior cruciate ligament and posterolateral corner
- 3‑ Anterior cruciate ligament and posterior cruciate ligament
- 4‑ Posterior cruciate ligament and posterolateral corner
- 5‑ Posterior cruciate ligament
- Fig. 38a Fig. 38b
- Question 96.147
- Answer = 4
- Reference(s)
- Veltri DM, Warren RF: Isolated and combined posterior cruciate ligament injuries. J Am Acad Orthop Surg 1993;1:67‑75.
- 96.148 A 22‑year‑old woman with no history of acute trauma has pain and weakness in
- her dominant wrist, generalized ligamentous laxity, and a click is elicited as the
- wrist is axially loaded and moved from radial to ulnar deviation. She relates the
- onset of these symptoms to a new job in a manufacturing plant. Plain
- radiographs, an MRI scan, and three‑compartment wrist arthrography are
- normal. The most likely diagnosis is
- 1‑ static dissociative wrist instability.
- 2‑ static nondissociative wrist instability.
- 3‑ dynamic dissociative wrist instability.
- 4‑ dynamic nondissociative wrist instability.
- 5‑ peripheral detachment of triangular fibrocartilage.
- Question 96.148
- Answer = 4
- Reference(s)
- Kasser JR (ed): Orthopaedic Knowledge Update 5. Rosemont, IL, American Academy of Orthopaedic Surgeons, 1996, pp 311‑328.
- 96.149 A 67‑year‑old woman undergoes total shoulder arthroplasty for osteoarthritis in the right shoulder. Five years later, she has progressive atraumatic shoulder pain and difficulty raising her arm to shoulder level (Figure 39). These symptoms are most likely due to
- 1‑ joint sepsis.
- 2‑ a rotator cuff tear.
- 3‑ anterior capsular avulsion.
- 4‑ loosening of the glenoid component.
- 5‑ loosening of the humeral component
- Fig. 39
- answer
- back
- Question 96.149
- Answer = 2
- back to this question
- next question
- Reference(s)
- Cofield RH, Edgerton BC: Total shoulder arthroplasty: Complications and revision surgery, in Greene WB (ed): Instructional Course Lectures XXXIX. Park Ridge, IL, American Academy of Orthopaedic Surgeons, 1990, pp 449‑462. Wirth MA, Rockwood CA Jr: Complications of shoulder arthroplasty. Clin Orthop 1994;307:47‑69.
- 96.150 A 6‑year‑old boy with no history of trauma has been limping and had unilateral
- foot pain for the past 4 weeks. Radiographs of the painful foot are shown in
- Figures 40a and 40b. Recommended treatment should be
- 1‑ talonavicular fusion.
- 2‑ navicular excision.
- 3‑ bone graft to the navicular.
- 4‑ IV antibiotics.
- 5‑ use of a short leg cast.
- Fig. 40a
- Fig. 40b
- Question 96.150
- Answer = 5
- Reference(s)
- Williams GA, Cowell HR: Kohler's disease of the tarsal navicular. Clin Orthop 1981;158:53‑58. Meehan PL: Other conditions of the foot, in Morrissy RT (ed): Lovell and Winter's Pediatric Orthopaedics, ed 3. Philadelphia, PA, JB Lippincott, 1990, vol 2, pp 991‑1021.
- 96.151 A 65‑year‑old man with a history of lumbar spinal stenosis now has hyperreflexia
- in his upper extremities. The most likely cause of this patient's condition is
- 1‑ increased lumbar stenosis.
- 2‑ herniated disk at C7‑Tl.
- 3‑ cervical spinal stenosis.
- 4‑ cerebral vascular insufficiency.
- 5‑ instability at CI‑2.
- answer
- back
- Question 96.151
- Answer = 3
- back to this question
- next question
- Reference(s)
- Clark CR: Degenerative conditions of the spine: Differential diagnosis and nonsurgical treatment, in Frymoyer JW (ed): The Adult Spine: Principles and Practice. New York, NY, Raven Press, 1991, vol 2, pp 1145‑1164.
- 96.152 A 1‑1/2‑year‑old boy falls and injures his elbow. Radiographs were interpreted as
- normal, but because of pain and swelling with ecchymosis, a distal humeral
- epiphyseal injury was suspected, and the patient's elbow was placed in a cast. Three
- weeks later, the cast was removed; current radiographs are shown in Figure 41.
- Further management should consist of
- 1‑ open reduction and internal fixation.
- 2‑ Dunlop traction.
- 3‑ observation.
- 4‑ closed reduction and use of a long arm cast.
- 5‑ closed reduction and percutaneous pinning.
- Fig. 41
- Question 96.152
- Answer = 3
- Reference(s)
- DeLee JC, Wilkins KE, Rogers LF, et al: Fracture‑separation of the distal humeral epiphysis. J Bone Joint Surg 1980;62A:46‑51. Wilkins KE: Fractures and dislocation of the elbow region, in Rockwood CA, Wilkins KE, King RE (eds): Fractures in Children, ed 3. Philadelphia, PA, JB Lippincott, 1991, pp 509‑828.
- 96.153 A 44‑year‑old woman with a long history of rheumatoid arthritis has increasing pain in the upper cervical spine despite conservative treatment over the past several months. The pain does not radiate into the upper extremities. Examination reveals slight limitation of cervical rotation. The neurologic examination is normal, and there is no tingling in her arms on maximum flexion of her neck. Figures 42a through 42d show anteroposterior, lateral, and flexion and extension lateral views of the cervical spine. An MRI scan shows no significant pannus formation at the C1‑C2 level and no compression of the upper spinal cord or brainstem. Treatment at this time should consist of
- 1‑ transoral decompression.
- 2‑ transoral decompression and posterior occiput‑C3 fusion.
- 3‑ posterior CI‑C2 fusion.
- 4‑ posterior occiput‑C2 fusion.
- 5‑ posterior occiput‑C3 fusion.
- Fig. 42a
- Fig. 42b
- Go to next slide
- for remaining figures
- and answer link
- back
- Fig. 42c Fig. 42d
- Question 96.153 continued
- answer
- back
- Question 96.153
- Answer = 3
- back to this question
- next question
- Reference(s)
- Zeidman SM, Ducker TB: Rheumatoid arthritis: Neuroanatomy, compression, and grading of deficits. Spine 1994;19:2259‑2266. Boden SD: Rheumatoid arthritis of the cervical spine: Surgical decision making based on predictors of paralysis and recovery. Spine 1994;19:2275‑2280.
- 96.154 A 25‑year‑old woman has had a painful forefoot for 6 months and has been
- using felt pads in her shoes to relieve the pain. Radiographs show degenerative
- changes of the second metatarsal head with flattening and fragmentation. Initial
- treatment should be
- 1‑ metatarsal head resection.
- 2‑ protected immobilization with an orthosis or shoe modifications.
- 3‑ dorsiflexion osteotomy of the metatarsal shaft.
- 4‑ basal hemiphalangectomy.
- 5‑ steroid injection and physiotherapy modalities.
- Question 96.154
- Answer = 2
- Reference(s)
- Helal B, Gibb P: Frieberg's disease: A suggested pattern of management. Foot Ankle Int 1987;8:94‑102. Lutter LD, Pfeffer GR, Mizel MS (eds): Orthopaedic Knowledge Update: Foot and Ankle. Rosemont, IL, American Academy of Orthopaedic Surgeons, 1994, pp 89‑100.
- 96.155 Which of the following describes tension in the collateral ligaments of the
- metacarpophalangeal joints?
- 1‑ Taut in flexion, lax in extension
- 2‑ Taut in flexion and extension
- 3‑ Taut in midflexion only
- 4‑ Lax in flexion and extension
- 5‑ Lax in flexion, taut in extension
- Question 96.155
- Answer = 1
- Reference(s)
- Ishizuki M: Injury to collateral ligament of the metacarpophalangealjoint of a finger. J Hand Surg 1988;13A:444‑448. Schubiner JM, Mass DP: Operation for collateral ligament ruptures of the metacarpophalangeal joints of the fingers. J Bone Joint Surg 1989;71B:388‑389.
- 96.156 A 4‑year‑old boy with no previous symptoms falls and has acute hip pain with
- or without motion and is unable to bear weight. A radiograph is shown in Figure
- 43. Recommended treatment should be
- 1‑ biopsy, bone graft, and hip spica cast immobilization.
- 2‑ hip spica cast immobilization.
- 3‑ crutches, advance weightbearing as tolerated.
- 4‑ aspiration, injection with steroid, and hip spica cast immobilization.
- 5‑ aspiration, injection with bone‑marrow slurry, hip spica cast immobilization.
- Fig. 43
- Question 96.156
- Answer = 2
- Reference(s)
- Makley JT, Joyce MJ: Unicameral bone cyst (simple bone cyst). Orthop Clin North Am 1989;20:407‑415. Wold LE, McLeod RA, Sim FH, et al: Atlas of Orthopedic Pathology. Philadelphia, PA, WB Saunders, 1990, pp 238‑241.
- 96.157 A 36‑year‑old man has had a mass on his left index finger that has slowly enlarged over the past 6 months. There is no history of trauma or penetrating injury. The mass currently has the clinical appearance shown in Figure 44a. Recently he has had moderate discomfort and limitation of proximal interphalangeal joint motion. An oblique plain radiograph is shown in Figure 44b, and a histologic section from the mass is shown in Figure 44c. The most likely diagnosis is
- 1‑ malignant fibrous histiocytoma.
- 2‑ liposarcoma.
- 3‑ rhabdomyosarcoma.
- 4‑ rhabdomyoma.
- 5‑ giant cell tumor of the tendon sheath.
- Go to next slide for figures and answer link
- Fig. 44a
- Fig. 44b
- Fig. 44c
- Question 96.157
- Answer = 5
- Reference(s)
- Enzinger FM, Weiss SW: Soft Tissue Tumors, ed 3. St Louis, MO, CV Mosby, 1995.
- 96.158 Which of the following musculoskeletal neoplasms is considered a low‑grade
- lesion with less than a 25% risk of systemic metastases?
- 1‑ Periosteal osteosarcoma
- 2‑ Parosteal osteosarcoma
- 3‑ Postirradiation osteosarcoma
- 4‑ Dedifferential chondrosarcoma
- 5‑ Osteosarcoma occuring in Paget's disease
- Question 96.158
- Answer = 2
- Reference(s)
- Simon SR (ed): Orthopaedic Basic Science. Rosemont, IL, American Academy of Orthopaedic Surgeons, 1994,pp 219‑276. Enneking WF, Spanier SS, Goodman MA: Current concepts review: The surgical staging of musculoskeletal sarcoma. J Bone Joint Surg 1980;62A:1027‑1030.
- 96.159 What is the best treatment for a chordoma located at the S3 and S4 levels?
- 1‑ Surgical excision
- 2‑ Systemic chemotherapy
- 3‑ Regional chemotherapy
- 4‑ Radiation
- 5‑ Arterial embolization
- Question 96.159
- Answer = 1
- Reference(s)
- Mindell ER, Karakousis CP: Benign and malignant tumors of the sacrum, in Frymoyer JW (ed): The Adult Spine: Principles and Practice. New York, NY, Raven Press, 1991, vol 2, pp 2131‑2143.
- 96. 160 An asymptomatic 15‑year‑old girl was recently found to have idiopathic scoliosis.
- Menarche was 2 1/2 years ago. Radiographs reveal a thoracic curve with a Cobb
- angle of 37 degrees, and the patient is Risser 5. Recommended management
- should consist of
- 1‑ posterior spinal fusion and instrumentation.
- 2‑ anterior spinal fusion, followed by posterior spinal fusion and instrumentation.
- 3‑ cervical thoracolumbosacral orthosis (Milwaukee brace).
- 4‑ thoracolumbosacral orthosis.
- 5‑ observation.
- answer
- back
- Question 96.160
- Answer = 5
- back to this question
- next question
- Reference(s)
- Lonstein JE, Carlson JM: The prediction of curve progression in untreated idiopathic scoliosis during growth. J Bone Joint Surg 1984;66A:1061‑1071. Weinstein SL, Ponseti IV: Curve progression in idiopathic scoliosis. J Bone Joint Surg 1983;65A:447‑455.
- 96.161 A 10‑year‑old boy with a history of hip pain is diagnosed with septic arthritis of the
- sacroiliac joint. He has a temperature of 102.2°F (39"C), and a WBC of 10,000/mm'
- (normal 3,500 to 10,500/mm'). Management should include antibiotics and which of the
- following?
- 1‑ Rest with observation
- 2‑ Spica cast immobilization
- 3‑ Repeated aspiration on a daily basis
- 4‑ Open drainage and debridement of the sacroiliac joint
- 5‑ Arthroscopic drainage and debridement of the sacroiliac joint
- Question 96.161
- Answer = 1
- Reference(s)
- Reilly JP, Gross RH, Emans JB, et al: Disorders of the sacra‑iliac joint in children. J Bone Joint Surg 1988;70A:31‑40. Morrissy RT: Bone and joint infections, in Morrissy RT (ed): Lovell and Winter's Pediatric Orthopaedics, ed 3. Philadelphia, PA, JB Lippincott 1990, vol 1, pp 539‑561.
- 96.162 Of the following, which lesions may metastasize to the lungs?
- 1‑ Chondroma and fibrous dysplasia
- 2‑ Nonossifying fibroma and fibrous dysplasia
- 3- Chondromyxoid fibroma and nonossifying fibroma
- 4‑ Giant cell tumor and chondroblastoma
- 5‑ Osteofibrous dysplasia and aneurysmal bone cyst
- Question 96.162
- Answer = 4
- Reference(s)
- Simon SR (ed): Orthopaedic Basic Science. Rosemont, IL, American Academy of Orthopaedic Surgeons, 1994, pp 219‑276.
- 96.163 Which of the following is a property of polymethylmethacrylate (PMMA) bone
- cement?
- 1‑ Endurance limit higher in tension than in compression
- 2‑ Functions primarily as an adhesive
- 3‑ Reaches its ultimate strength within 24 hours
- 4‑ Ultimate tensile strength equals that of cortical bone
- 5‑ Strength decreased by about 50% with addition of antibiotic powder.
- Question 96.163
- Answer = 3
- Reference(s)
- Simon SR (ed): Orthopaedic Basic Science. Rosemont,, IL, American Academy of Orthopaedic Surgeons, 1994, pp 474‑479.
- 96.164 A stainless steel plate has been secured by closely applied screws. What type of
- corrosion occurs after an oxygen concentration cell is established between closely
- fitted areas when the passive film over the metal is lost?
- 1‑ Stress
- 2‑ Galvanic
- 3‑ Intergranular
- 4‑ Crevice
- 5‑ Fretting
- Question 96.164
- Answer = 4
- Reference(s)
- Simon SR (ed): Orthopaedic Basic Science. Rosemont, IL, American Academy of Orthopaedic Surgeons, 1994, pp 447‑486.
- 96.165 During counseling about activity levels after total hip replacement, the patient
- should be advised that peak forces at the hip during jogging may be
- 1‑ equal to body weight.
- 2‑ two times body weight.
- 3‑ three times body weight.
- 4‑ six times body weight.
- 5‑ twelve times body weight.
- Question 96.165
- Answer = 4
- Reference(s)
- Kasser JR (ed): Orthopaedic Knowledge Update 5. Rosemont, IL, American Academy of Orthopaedic Surgeons, 1996, pp 29‑40.
- 96.166 Which of the following factors has the most influence on the development of
- osteonecrosis following fractures of the femoral head and neck in the skeletally
- immature patient?
- 1‑ Degree of displacement of the fracture at the time of injury
- 2‑ Intracapsular pressure
- 3‑ Rapidity of reduction
- 4‑ Type of hardware used for fixation
- 5‑ Postoperative weightbearing status
- Question 96.166
- Answer = 1
- Reference(s)
- Hughes LO, Beaty JH: Fractures of the head and neck of the femur in children. J Bone Joint Surg 1994;76A:283‑292.
- 96.167 In the anterolateral (Watson‑Jones) approach to the hip, the intermuscular plane
- is between what structures?
- 1‑ Gluteus medius and tensor fascia lata
- 2‑ Gluteus medius and rectus femoris
- 3‑ Gluteus maximus and gluteus medius
- 4‑ Gluteus minimus and rectus femoris
- 5‑ Gluteus minimus and tensor fascia lata
- Question 96.167
- Answer = 1
- Reference(s)
- Hoppenfeld S, deBoer P: The hip, in Hoppenfeld S, deBoer P (eds): Surgical Exposures in Orthopaedics: The Anatomic Approach. Philadelphia, PA, JB Lippincott, 1984, pp 301‑356.
- 96.168 A 3‑year‑old boy is immobilized in a short leg cast following a minor injury that
- resulted in acute left ankle pain. After 2 months in the cast, there is tenderness
- over the lateral malleolus. Physical examination also reveals some scattered macular
- skin lesions. A radiograph is shown in Figure 45. The physician should recommend
- 1‑ use of a long leg brace.
- 2‑ continued use of the short leg cast.
- 3‑ open reduction of the fibula with plate and bone graft.
- 4‑ intramedullary rod to the fibula with bone graft.
- 5‑ observation.
- Fig. 45
- Question 96.168
- Answer = 1
- Reference(s)
- Boyd HB: Pathology and natural history of congenital pseudarthrosis of the tibia. Clin Orthop 1982;166:5‑13. Murray HH, Lovell WW: Congenital pseudoarthrosis of the tibia: A long‑term follow‑up study. Clin Orthop 1982;166:14‑20.
- 96.169 Figure 46a shows a lateral elbow radiograph. What structure is represented in
- the line drawn in Figure 46b?
- 1‑ Capitellum
- 2‑ Lateral trochlea
- 3‑ Central trochlea
- 4‑ Medial trochlea
- 5‑ Medial epicondyle
- Fig. 46a Fig. 46b
- Question 96.169
- Answer = 3
- Reference(s)
- Berquist TH: Diagnostic radiographic techniques of the elbow, in Morrey BF (ed): The Elbow and Its Disorders, ed 2. Philadelphia, PA, WB Saunders, 1993, pp 98‑119.
- 96.170 The key histologic finding in eosinophilic granuloma is
- 1‑ plasma cells.
- 2‑ multinucleated giant cells.
- 3‑ eosinophils.
- 4‑ eosinophilic histiocytes.
- 5‑ lymphocytes.
- Question 96.170
- Answer = 4*
- Reference(s)
- Sessa S, Sommelet D, Lascombes P, et al: Treatment of Langerhans‑cell histiocytosis in children: Experience at the Children's Hospital of Nancy. J Bone Joint Surg 1994;76A:1513‑1525. *item deleted after statistical review
- 96.171 Which of the following terms best describes the combining of several smaller
- cohort studies into one larger cohort to examine treatment and outcome
- variables?
- 1‑ Outcomes assessment
- 2‑ Small area analysis
- 3‑ Meta‑analysis
- 4‑ Randomized clinical trial
- 5‑ Arthritis impact measurement research
- Question 96.171
- Answer = 3
- Reference(s)
- Kasser JR (ed): Orthopaedic Knowledge Update 5. Rosemont, IL, American Academy of Orthopaedic Surgeons, 1996, pp 85‑88.
- 96.172 A 3‑year‑old girl with pneumonitis is referred for treatment after chest radiographs
- reveal a spinal abnormality. She is asymptomatic and neurologic examination is
- normal. Radiographs of her spine are shown in Figures 47a and 47b.
- Recommended treatment of the spine should be
- 1‑ posterior fusion.
- 2‑ anterior convex epiphyseodesis.
- 3‑ combined anterior epiphyseodesis and posterior fusion.
- 4‑ observation.
- 5‑ thoracolumbosacral orthosis.
- Fig. 47a Fig. 47b
- answer
- back
- Question 96.172
- Answer = 4
- back to this question
- next question
- Reference(s)
- Winter RB: Congenital Deformities of the Spine. New York, NY, Thieme‑Stratton, 1983. McMaster MJ, Ohtsuka K: The natural history of congenital scoliosis: A study of two hundred and fifty‑one patients. J Bone Joint Surg 1982;64A:1128‑1147.
- 96.173 A femoral shaft fracture in a 10‑year‑old boy is treated with a reamed, locked intramedullary
- nail. Four months after surgery, the fracture is healing well, but the patient has been
- experiencing pain and limited motion of the ipsilateral hip. On physical examination, the hip
- is irritable and abduction and internal rotation are limited. The most likely cause of these
- finding is
- 1‑ postfracture growth stimulation.
- 2‑ trochanteric growth disturbance.
- 3‑ late postoperative infection.
- 4‑ osteonecrosis of the femoral head.
- 5‑ loosening of the fixation device.
- Question 96.173
- Answer = 4
- Reference(s)
- Beaty JH, Austin SM, Warner WC, et al: Interlocking intramedullary nailing of femoral‑shaft fractures in adolescents: preliminary results and complications. J Pediatr Orthop 1994;14:178‑183. O'Malley DE, Mazur JM, Cummings RJ: Femoral head avascular necrosis associated with intramedullary nailing in an adolescent. J Pediatr Orthop 1995;15:21‑23.
- 96.174 A 60‑year‑old woman has pain along the medial aspect of the ankle.
- Examination confirms pain along the posterior tibial tendon with normal single
- toe raise. She has had nonsurgical treatment for 8 weeks, including nonsteroidal
- anti‑inflammatory agents, physical therapy, and cast immobilization; however,
- she continues to have pain. What is the next appropriate step in management?
- 1‑ Posterior tibial tendon reconstruction
- 2‑ Anterior tibial tendon transfer and calcaneal cuboid arthrodesis
- 3‑ Steroid injection
- 4‑ Subtalar joint arthrodesis
- 5‑ Synovectomy of the posterior tibial tendon
- Question 96.174
- Answer = 5
- Reference(s)
- Mann RA, Thompson FM: Rupture of the posterior tibia] tendon causing flat foot: Surgical treatment. J Bone Joint Surg 1985;67A:556‑561. Lutter LD, Mizel MS, Pfeffer GB (eds): Orthopaedic Knowledge Update: Foot and Ankle. Rosemont, IL, American Academy of Orthopaedic Surgeons, 1994, pp 269‑282.
- 96.175 A 16-year-old boy with osteogenesis imperfecta who was previously a household
- ambulator with crutches has stopped walking and has developed increasing flexion
- contractures of the lower extremities over a 2‑year period. Recommended
- management should consist of
- 1‑ a bone scan to look for stress fractures.
- 2‑ a physical therapy program for stretching.
- 3‑ a daily standing program.
- 4‑ extension osteotomies of both femora.
- 5‑ cervical spine radiographs.
- answer
- back
- Question 96.175
- Answer = 5
- back to this question
- next question
- Reference(s)
- Harkey HL, Crockard HA, Stevens JM, et al: The operative management of basilar impression in osteogenesis imperfecta. Neurosurgery 1990;27:782‑786.
- 96.176 In a young adult with a displaced distal radius fracture, which of the following
- complications is most predictive of a poor outcome?
- 1‑ Nonunion of the ulnar styloid
- 2‑ 10‑degree loss of radial tilt (apex ulnar malunion)
- 3‑ 10‑degree loss of volar tilt (apex volar malunion)
- 4‑ 5‑mm depression of the lunate fossa
- 5‑ 5‑mm shortening of the radius
- Question 96.176
- Answer = 4
- Reference(s)
- Jupiter JB: Fractures of the distal end of the radius. J Bone Joint Surg 1991;73A:461‑469. Knirk JL, Jupiter JB: Intra‑articular fractures of the distal end of the radius in young adults. J Bone Joint Surg 1986;68A:647‑659.
- 96.177 What is the definition of strain?
- 1‑ Physical quantity defined as force per unit area (F/A)
- 2‑ Resistance of a structure to deformation
- 3‑ Amount of energy stored per unit volume in a viscoelastic material
- 4‑ Measure of deformation having six components
- 5‑ Deformation caused by twisting of a shaft
- Question 96.177
- Answer = 4
- Reference(s)
- Simon SR (ed): Orthopaedic Basic Science. Rosemont, IL, American Academy of Orthopaedic Surgeons, 1994, pp 397‑446.
- 96.178 You are reviewing an article in a scientific journal where the authors identified a nondiseased population and characterized them with respect to hypothesized risk factors and potential confounders. The population was then observed for a period of time and new cases of disease were recorded. Which of the following describes this type of study design?
- 1‑ Case‑control study
- 2‑ Cross‑sectional study
- 3‑ Cohort study
- 4‑ Clinical trial
- 5‑ Randomized control trial
- Question 96.178
- Answer = 3
- Reference(s)
- Kasser JR (ed): Orthopaedic Knowledge Update 5. Rosemont, IL, American Academy of Orthopaedic Surgeons, 1996, pp 71‑80.
- 96.179 What does the term spasticity most accurately refer to?
- 1‑ Shortening of muscle length
- 2‑ Increased muscle strength
- 3‑ Increased muscle resistance above normal
- 4‑ Increased muscle activity with increasingly rapid stretch
- 5‑ Failure of muscle coordination
- Question 96.179
- Answer = 4
- Reference(s)
- Oppenheim WL: Selective posterior rhizotomy for spastic cerebral palsy: A review. Clin Orthop 1990;253:20‑29.
- 96.180 A 13‑year‑old girl has pain and swelling, but no deformity in her dominant elbow
- after a fall. Radiographs are shown in Figures 48a and 48b. Management should
- consist of which of the following?
- 1‑ Observation
- 2‑ Closed reduction and casting
- 3‑ Closed reduction and percutaneous pinning
- 4‑ Open reduction and internal fixation
- 5‑ Arthrotomy with excision of the fragment
- Fig. 48a Fig. 48b
- answer
- back
- Question 96.180
- Answer = 4
- back to this question
- next question
- Reference(s)
- Collert S: Surgical management of fracture of the capitulum humeri. Acta Orthop Scand 1977;48:603‑606. Simpson LA, Richards RR: Internal fixation of a capitellar fracture using Herbert screws: A case report. Clin Orthop 1986;209:166‑168.
- 96.181 Examination of a 25‑year‑old man who has knee pain after a motorcycle accident
- reveals an effusion, normal stability to varus/valgus stress, a negative Lachman
- test, and a grade III posterior drawer. Radiographs are shown in Figures 49a and
- 49b. Treatment should consist of which of the following?
- 1‑ Repair of the injured structures
- 2‑ Posterior cruciate reconstruction with patellar tendon graft
- 3‑ Immobilization for 4 weeks
- 4‑ Physical therapy focused on quadriceps strengthening and range of motion
- 5‑ Diagnostic arthroscopy followed by rehabilitation
- Fig. 49a Fig. 49b
- Question 96.181
- Answer = 1
- Reference(s)
- Veltri DM, Warren RF: Isolated and combined posterior cruciate ligament injuries. J Am Acad Orthop Surg 1993;1:67‑75.
- 96.182 A 4‑year‑old girl fell and injured her neck while rollerskating 10 days ago. Since
- her injury she has been reluctant to move her head and has neck pain. On
- examination, the patient's head is held with about 20 degrees of rotation to the
- left, and she is hesitant with any attempt at active or passive motion. She is
- ambulatory and neurologic examination is normal. Radiographs are shown in
- Figure 50a, and a CT scan of C1‑2 is shown in Figure 50b. Recommended
- management of this patient's injury should consist of
- 1‑ use of a Philadelphia collar.
- 2‑ use of a Minerva jacket.
- 3‑ cervical traction with a halter.
- 4‑ open reduction with Gallie fusion.
- 5‑ manipulative reduction.
- Fig. 50a Fig. 50b
- Question 96.182
- Answer = 3
- Reference(s)
- Lebwohl NH, Eismont FJ: Cervical spine injuries in children, in Weinstein SL (ed): The Pediatric Spine. New York, NY, Raven Press, 1994, p 732. Herzenberg JE, HensingerRN, Fielding JW: Fracture of the spine, in Rockwood CA, Wilkins KE, King RE (eds): Fractures in Children, ed 3. Philadelphia, PA, JB Lippincott, 1991, pp 921‑989.
- 96.183 A 31‑year‑old woman in the third trimester of pregnancy develops pain in her left
- groin that radiates to the anterior thigh. For the last 5 days, the patient has been
- confined to bed because of the pain. Physical examination is unremarkable with
- no limitation of hip motion. No neurologic deficit is present. Initial radiographs of
- the hip are unremarkable, but MRI scans reveal decreased signal intensity in the
- T1‑weighted images and increased signal intensity in the T2‑weighted images in the
- femoral head, neck, and intertrochanteric area. The most likely diagnosis is
- 1‑ osteonecrosis of the femoral head.
- 2‑ osteomyelitis of the femoral head.
- 3‑ septic arthritis of the hip.
- 4‑ stress fracture of the femoral neck.
- 5‑ transient osteoporosis of the hip.
- Question 96.183
- Answer = 5
- Reference(s)
- Heckman JD, Sassard R: Musculoskeletal considerations in pregnancy. J Bone Joint Surg 1994;76A:1720‑1730. Takatori Y, Kokubo T, Ninomiya S, et al: Transient osteoporosis of the hip: Magnetic resonance imaging. Clin Orthop 1991;271:190‑194.
- 96.184 A 2‑year‑old boy is referred for scoliosis. Physical examination is otherwise normal. A radiograph reveals a 30‑degree curve convex to the left between T5 and T11. No congenital anomalies are seen. Rib vertebral angle difference is 10 degrees. Recommended treatment is
- 1‑ observation.
- 2‑ bracing.
- 3‑ serial derotation casts.
- 4‑ posterior fusion.
- 5‑ anterior and posterior fusion.
- Question 96.184
- Answer = 1
- Reference(s)
- Dickson RA: Early onset idiopathic scoliosis, in Weinstein SL (ed): The Pediatric Spine. New York, NY, Raven Press, 1994, p 421.
- 96.185 Which of the following terms is defined as maximal contraction of a muscle at a
- constant velocity throughout its functional range of motion?
- 1‑ Eccentric
- 2‑ Isotonic
- 3‑ Isometric
- 4‑ Concentric
- 5‑ Isokinetic
- Question 96.185
- Answer = 5
- Reference(s)
- Sapega AA: Muscle performance evaluation in orthopaedic practice. J Bone Joint Surg 1990;72A:1562‑1574.
- 96.186 A physician is asked to testify in a malpractice suit involving a colleague and a
- patient who was referred by an attorney. The patient was evaluated and charged
- for the evaluation; the fee was paid. The physician is subsequently asked by the
- patient's attorney to testify in court. What should be the physician's next course of
- action?
- 1‑ Refuse to testify because payment was already received for the evaluation.
- 2‑ Testify and support the patient's position in the litigation.
- 3‑ Testify and support the colleague's position in the litigation,
- 4‑ Testify with the understanding that compensation is contingent on the patient's
- winning the suit.
- 5‑ Testify only to the facts.
- Question 96.186
- Answer = 5
- Reference(s)
- AAOS Committee on Professional Liability: Medical Malpractice: A Primer for Orthopaedic Residents and Fellows. Rosemont, IL, American Academy of Orthopaedic Surgeons, 1993, p 68.
- 96.187 An 86‑year‑old man has had pain in his right buttock that has increased in
- severity for the past 6 months. Prior to this, his pain had been responsive to
- calcitonin treatments. An anteroposterior radiograph of his pelvis is shown in
- Figure 51 a, and a CT scan of the pelvis is shown in Figure 51 b. A biopsy of the
- soft‑tissue mass associated with his right ilium would most likely reveal
- 1‑ metastatic adenocarcinoma.
- 2‑ multiple myeloma.
- 3‑ lymphoma.
- 4‑ Paget's osteosarcoma.
- 5‑ liposarcoma.
- Figure 51a
- Fig. 51b
- Question 96.187
- Answer = 4
- Reference(s)
- Frassica FJ, Sim FH, Frassica DA, et al: Survival and management considerations in postirradiation osteosarcoma and Paget's osteosarcoma. Clin Orthop 1991;270:120‑127. Tumors associated with Paget's disease of bone, in Huvos AG (ed): Bone Tumors: Diagnosis, Treatment, and Prognosis, ed 2. Philadelphia, PA, WB Saunders, 1991, pp 201‑222.
- 96.188 The most common complication associated with the use of closed treatment
- of complete Achilles tendon ruptures is
- 1‑ higher rerupture rate.
- 2‑ excessive lengthening of the musculotendinous unit.
- 3‑ atrophy of the gastrocnemius‑soleus muscle.
- 4‑ lack of push‑off power in running.
- 5‑ loss of ankle motion.
- Question 96.188
- Answer = 1
- Reference(s)
- Soma CA, Mandelbaum BR: Repair of acute Achilles tendon ruptures. Orthop Clin North Am 1995;26:239‑247.
- 96.189 What is the most common musculoskeletal complaint reported during pregnancy?
- 1‑ Leg cramps
- 2‑ Hand and wrist pain
- 3‑ Low back pain
- 4‑ Hip pain
- 5‑ Progression of scoliosis
- Question 96.189
- Answer = 3
- Reference(s)
- Heckman JD, Sassard R: Musculoskeletal considerations in pregnancy. J Bone Joint Surg 1994;76A:1720‑1730.
- 96.190 A 16‑year‑old boy who underwent an uncomplicated posterior spinal fusion and
- instrumentation 2 years ago now has a 4‑cm area of redness and swelling over the
- central portion of the surgical scar. The area is fluctuant and nontender. There is
- no past history of swelling or drainage. The erythrocyte sedimentation rate is 22
- mm/hr (normal up to 20 mm/hr) and the WBC is 8,000/mm' (normal 3,500 to
- 10,500/mm'). The spinal instrumentation consists of two rods with multiple
- hooks. Which of the following methods should be used to manage this patient's
- condition?
- 1- Local incision and packing of the open area
- 2‑ Administration of antibiotics
- 3‑ Removal of hardware, irrigation and debridement, wound closure, and
- administration of antibiotics
- 4‑ Extensive irrigation and debridement, open packing of the wound, and
- administration of antibiotics
- 5‑ Extensive irrigation and debridement, wound closure, and administration of
- antibiotics
- answer
- back
- Question 96.190
- Answer = 3
- back to this question
- next question
- Reference(s)
- Richards BS: Delayed infections following posterior spinal instrumentation for the treatment of idiopathic scoliosis. J Bone Joint Surg 1995;77A:524‑529.
- 96.191 An active 41‑year‑old man underwent an uncemented total hip arthroplasty for osteonecrosis 7 years ago and has been asymptomatic for the past 6 years. During the course of routine follow‑up, aggressive lysis was noted on both the pelvic and femoral side (Figure 52). Aspiration was negative and the erythrocyte sedimentation rate was 8 mm/hr (normal up to 20 mm/hr). The most likely cause of the lyric lesion is
- 1‑ late hematogenous infection.
- 2‑ periprosthetic sarcoma.
- 3‑ metastatic hypernephroma.
- 4‑ aseptic loosening.
- 5‑ osteolysis from particulate debris.
- Fig. 52
- Question 96.191
- Answer = 5
- Reference(s)
- Jacobs JJ, Shanbhag A, Glant IT, et al: Wear debris in total joint replacements. J Am Acad Orthop Surg 1994;2:212‑220.
- 96.192 Compared to fresh‑frozen allograft bone, the use of freeze‑dried allograft bone
- in a major reconstructive procedure is advantageous because of what factor?
- 1‑ Lower likelihood of viral transmission
- 2‑ Greater load‑bearing capacity
- 3‑ Greater osteogenic contribution
- 4‑ Better osteoinductive properties
- 5‑ Better osteoconductive properties
- Question 96.192
- Answer = 1
- Reference(s)
- Einhorn TA: Enhancement of fracture‑healing. J Bone Joint Surg 1995;77A:940‑956. Skinner HB: Alternatives in the selection of allograft bone. Orthopedics 1990;13:843‑846.
- 96.193 Patellar resurfacing in total knee replacement is most strongly indicated in what
- patient type?
- 1‑ A thin female with osteoarthritis
- 2‑ A young patient with spontaneous osteonecrosis of the knee
- 3‑ An older patient with inflammatory arthritis
- 4‑ An overweight male with posttraumatic arthritis of the knee
- 5‑ A young male with osteoarthritis
- Question 96.193
- Answer = 3
- Reference(s)
- Scuderi GR, Insall IN, Scott WN: Patellofemoral pain after total knee arthroplasty. 1 Am Acad Orthop Surg 1994;2:239‑245.
- 96.194 A 27‑year‑old man has a humeral shaft fracture and no radial nerve motor
- function as the result of a high‑velocity gunshot wound to his dominant arm. At
- the time of debridement, a transection of the nerve is noted 12 cm proximal to
- the elbow. For the most predictable recovery of function, recommended
- treatment is
- 1‑ immediate direct microsurgical repair.
- 2‑ delayed (6 weeks) direct microsurgical repair.
- 3‑ delayed (6 weeks) interfascicular sural nerve grafts.
- 4‑ delayed (6 weeks) autologous vein interposition graft.
- 5‑ elective tendon transfers.
- Question 96.194
- Answer = 5
- Reference(s)
- Kallio PK, Vastamaki M, Solonen KA: The results of secondary microsurgical repair of the radial nerve in 33 patients. J Hand Surg 1993;1813:320‑322.
- 96.195 A 40‑year‑old man has dermatomal pain at the T1 level and numbness and weakness in the L5 and S 1 distribution on the same side with bowel and bladder disturbance. The most likely diagnosis is
- 1‑ T11‑12 herniated nucleus pulposus.
- 2- L5‑S1 herniated nucleus pulposus.
- 3‑ degenerative stenosis at L4‑5.
- 4‑ congenital stenosis at L3‑S1.
- 5‑ thoracic syrinx at T4‑8.
- Question 96.195
- Answer = 1
- Reference(s)
- Skubic JW, Kostuik JP: Thoracic pain syndromes and thoracic disc herniations, in Frymoyer JW (ed): The Adult Spine: Principles and Practice. New York, NY, Raven Press, 1991, vol 2, pp 1443‑1461.
- 96.196 Glycocalyx is best described as
- 1‑ an exopolysaccharide coating that envelopes bacteria.
- 2‑ glucose‑derived renal stones associated with hypercalcemia.
- 3‑ a third‑generation cephalosporin antibiotic.
- 4‑ a slime‑producing bacteria strain.
- 5‑ particulate debris associated with prosthetic loosening.
- Question 96.196
- Answer = 1
- Reference(s)
- Kasser JR (ed): Orthopaedic Knowledge Update 5. Rosemont, IL, American Academy of Orthopaedic Surgeons, 1996, pp 149‑161.
- 96.197 Figure 53a is the radiograph of an 11‑year‑old girl who fell on an outstretched arm.
- Figure 53b shows postreduction radiographs at 72 hours, and Figure 53c shows the
- wrist 4 weeks after fracture. Recommended treatment should consist of
- 1‑ a short arm cast.
- 2‑ closed manipulation and casting.
- 3‑ closed reduction and external fixation.
- 4‑ closed reduction and percutaneous pinning.
- 5‑ open reduction and internal fixation.
- Fig. 53b
- Fig. 53c
- Fig. 53a
- Question 96.197
- Answer = 1
- Reference(s)
- O'Brien ET: Fractures of the hand and wrist region, in Rockwood CA Jr. Wilkins KE, King RE (eds): Fractures in Children. Philadelphia, PA, 1B Lippincott, 1984,vo13,pp 229‑299.
- 96.198 A 24‑year‑old man whose dominant right arm is caught in a rubber vulcanizing
- machine sustains a below‑elbow amputation with the bones transected midway
- between the elbow and wrist. He also has third‑degree burns extending 5 cm
- proximal to the amputation. The most appropriate initial surgical procedure is
- 1‑ above‑elbow amputation.
- 2‑ through‑elbow amputation.
- 3‑ below‑elbow amputation just distal to the biceps tuberosity.
- 4‑ excision of the burned skin, dressing the wound, and skin grafting later.
- 5‑ excision of the burned skin and shortening of the bones to allow for normal
- skin closure.
- Question 96.198
- Answer = 4
- Reference(s)
- Harris WR: Amputations of the upper extremity, in Kostuik JP (ed): Amputation Surgery and Rehabilitation. New York, NY, Churchill‑Livingstone, 1981, pp 99‑103.
- 96.199 Which of the following collagen types is believed to be the glue that holds
- together the collagen latticework?
- 1‑ I
- 2‑ II
- 3‑ IX
- 4‑ X
- 5‑ XI
- Question 96.199
- Answer = 5
- Reference(s)
- Petrera P, Rubash HE: Revision total hip arthroplasty: The acetabular component J Am Acad Orthop Surg 1995;3:15‑21.
- 96.200 Which of the following collagen types is believed to be the glue that holds
- together the collagen latticework?
- 1‑ I
- 2‑ II
- 3‑ IX
- 4‑ X
- 5‑ XI
- Question 96.200
- Answer = 5 *
- Reference(s)
- Kasser JR (ed): Orthopaedic Knowledge Update 5. Rosemont, IL, American Academy of Orthopaedic Surgeons, 1996, pp 163‑175. *item deleted after statistical review
- 96.201 The central event in the development of rheumatoid arthritis is the reaction
- between the antigen presenting cell (APC) macrophage and which of the
- following cells?
- 1‑ B (antibody production)
- 2‑ T (helper/inducer)
- 3‑ Mast
- 4‑ Synovial type A
- 5‑ Synovial type B
- Question 96.201
- Answer = 2
- Reference(s)
- Kasser JR (ed): Orthopaedic Knowledge Update 5. Rosemont, IL, American Academy of Orthopaedic Surgeons, 1996, pp 163‑175.
- 96.202 What is the axonal response to complete transection of a peripheral nerve?
- 1‑ Hypertrophy of the proximal axon
- 2‑ Axonal sprouting from the distal nerve segment
- 3‑ Axonal degeneration extending distally to the somatosensory receptor
- 4‑ Axonal degeneration extending distally to the first node of Ranvier
- 5‑ Axonal degeneration extending proximally to the cell nucleus
- Question 96.202
- Answer = 3
- Reference(s)
- Fawcett JW, Keynes RJ: Peripheral nerve regeneration. Annu Rev Neurosci 1990;13:43‑60. Nerve regeneration, in Lundborg G (ed): Nerve Injury and Repair. New York, NY, Churchill‑Livingstone, 1988, pp 149‑195.
- 96.203 A surgeon inadvertently places two different types of metal in contact with each
- other. What type of corrosion may occur as a result of the different metal types?
- 1‑ Crevice
- 2‑ Galvanic
- 3‑ Fretting
- 4‑ Stress
- 5‑ Pitting
- Question 96.203
- Answer = 2
- Reference(s)
- Simon SR (ed): Orthopaedic Basic Science. Rosemont IL, American Academy of Orthopaedic Surgeons, 1994, pp 447‑486.
- 96.204 A 6‑year‑old girl has an area of painless swelling in the medial popliteal fossa that
- is 4 cm in diameter. Aspiration reveals the swelling to be consistent with a
- popliteal Baker's cyst, and the cyst disappears following aspiration. Three months
- later the cyst recurs but is still painless. What is the most appropriate treatment at
- this time?
- 1‑ Excision
- 2‑ Observation
- 3‑ Repeat aspiration
- 4‑ Aspiration and steroid injection
- 5‑ Aspiration and phenol injection
- Question 96.204
- Answer = 2
- Reference(s)
- Staheli LT: The lower limb, in Morrissy RT (ed): Lovell and Winter's Pediatric Orthopaedics, ed 3. Philadelphia, PA, JB Lippincott, 1990, vol 2, pp 741‑766.
- 96.205 The mechanism of facilitating bone healing with hydroxyapatite biomatrix is an
- example of
- 1‑ osteoconduction.
- 2‑ osteogenesis.
- 3‑ osteoinduction.
- 4‑ osteoresorption.
- 5‑ osteoclasis.
- Question 96.205
- Answer = 1
- Reference(s)
- Einhorn TA: Enhancement of fracture‑healing. J Bone Joint Surg 1995;77A:940‑956. Bucholz RW, Carlton A, Holmes R: Interporous hydroxyapatite as a bone graft substitute in tibial plateau fractures. Clin Orthop 1989;240:53‑62.
- 96.206 A 45‑year‑old man has a 3‑year history of wrist pain that has not been relieved by a
- 6 month regimen of splinting and administration of anti‑inflammatory medication.
- Examination reveals a dorsal‑radial tenderness and a 40% reduction in the wrist
- flexionextension arc. A radiograph is shown in Figure 55. What is the best
- treatment?
- 1‑ Dorsal capsulodesis
- 2‑ Scapholunate fusion
- 3‑ Scaphotrapezial‑trapezoid fusion
- 4‑ Radial styloidectomy
- 5‑ Proximal row carpectomy
- Fig. 55
- Question 96.206
- Answer = 5*
- Reference(s)
- Krakauer JD, Bishop AT, Cooney WP: Surgical treatment of scapholunate advanced collapse. J Hand Surg 1994;19A:751‑759. *item deleted after statistical review
- 96.207 Which of the following patients with Legg‑Calve‑Perthes disease would have the most
- guarded prognosis?
- 1‑ A 4‑year‑old girl with partial head involvement
- 2‑ A 4‑year‑old boy with partial head involvement
- 3‑ A 4‑year‑old girl with collapse of the lateral pillar
- 4‑ A 6‑year‑old boy with total head involvement
- 5‑ A 9‑year‑old boy with collapse of the lateral pillar
- Question 96.207
- Answer = 5
- Reference(s)
- Frymoyer JW (ed): Orthopaedic Knowledge Update 4. Rosemont IL, American Academy of Orthopaedic Surgeons, 1993, pp 505‑513. Herring JA: The treatment of Legg‑Calve‑Perthes disease: A critical review of the literature. J Bone Joint Surg 1994;76A:448‑458. Wenger DR, Ward WT, Herring JA: Legg‑Calve‑Perthes disease. J Bone Joint Surg 1991;73A:778‑788.
- 96.208 What is claims‑made professional liability insurance coverage?
- 1- Covers the insured for all claims made resulting from the period of time covered by
- the policy regardless of when the claim is made
- 2‑ Covers the insured for all claims made during the period of coverage for
- incidents that occurred during the period of coverage
- 3‑ Covers the injured for claims made after the liability insurance has expired
- 4‑ Coverage exclusively for residents and fellows during their period of training
- 5‑ Supplemental insurance coverage available for physicians who have a claim
- made against them
- Question 96.208
- Answer = 2
- Reference(s)
- AAOS Committee on Professional Liability: Medical Malpractice: A Primer for Orthopaedic Residents and Fellows. Rosemont, IL, American Academy of Orthopaedic Surgeons, 1994, p 3.
- 96.209 What is the most reliable parameter used to estimate skeletal muscle contractile
- force potential?
- 1‑ Length of the muscle
- 2‑ Lever arm of the muscle
- 3‑ Lever arm times length of the muscle
- 4‑ Cross‑sectional area of the muscle
- 5‑ Distribution of slow and fast twitch fibers
- Question 96.209
- Answer = 4
- Reference(s)
- Brand PW, Beach RB, Thompson DE: Relative tension and potential excursion of muscle in the forearm and hand J Hand Surg 1981;6:209‑219. An KN, Chao EYC, Kaufman KR: Analysis of muscle and joint loads, in Mow VC, Hayes WC (eds): Basic Orthopaedic Biomechanics. New York, NY, Raven Press, 1991, pp 1‑50.
- 96.210 Which of the following is the most common mode of failure of fixation in intertrochanteric
- hip fractures using a compression hip screw?
- 1‑ Fracture of the side plate
- 2‑ Fracture of the femur below the plate
- 3‑ Superior cutout of the screw from the femoral head
- 4‑ Disengagement of the lag screws and side plate
- 5‑ Pulling out of the cortical screws from the femoral shaft
- Question 96.210
- Answer = 3
- Reference(s)
- Kyle RF, Cabanela ME, Russell TA, et al: Fractures of the proximal part of the femur, in Jackson DW (ed): Instructional Course Lectures. Rosemont, IL, American Academy of Orthopaedic Surgeons, 1995, pp 227‑253. Larsson S, Friberg S, Hansson LL Trochanteric fractures: Influence of reduction and implant position on impaction and complications. Clin Orthop 1990;259:130‑139.
- 96.211 For 3 months, a 35‑year‑old man has had pain in the posteromedial ankle when running, walking, or climbing stairs. Physical examination reveals tenderness and swelling behind the medial malleolus. Passive extension of the great toe is greater when the foot is plantar flexed. The most likely diagnosis is
- 1‑ tarsal tunnel syndrome.
- 2‑ posterior tibial tendinitis.
- 3‑ flexor digitorum longus tendinitis.
- 4‑ flexor hallucis longus tendinitis.
- 5‑ sustentaculum talus impingement.
- Question 96.211
- Answer = 4
- Reference(s)
- Jones DC: Tendon disorders of the foot and ankle. J Am Acad Orthop Surg 1993;1:87‑94. Hamilton WG: Foot and ankle injuries in dancers, in Mann RA, Coughlin MJ(eds): Surgery of the Foot and Ankle, ed 6. St Louis, MO, CV Mosby, 1993, pp 1241‑1276.
- 96.212 A 26‑year‑old man sustains a fracture‑dislocation at the C5‑C6 levels in a motor
- vehicle accident. Approximately 48 hours after the injury, the bulbocavemosus
- reflex returns but there is no motor function below the C6 level, and there is only
- some sensation in the sacral region. Which of the following is the most likely
- outcome?
- 1‑ Wheelchair dependent and able to transfer independently
- 2‑ Wheelchair dependent but unable to transfer independently
- 3‑ Full neurologic recovery
- 4‑ Unknown extent of neurologic recovery
- 5‑ Minimal neurologic recovery
- answer
- back
- Question 96.212
- Answer = 4
- back to this question
- next question
- Reference(s)
- Bauer RD, Errico TJ: Cervical spine injuries, in Errico TJ, Bauer RD, Waugh T (eds): Spinal Trauma. Philadelphia, PA, JB Lippincott, 1990, pp 71‑121.
- 96.213 A patient presents with the radiograph shown in Figure 56. Collagen and/or
- molecular testing indicate what finding?
- 1‑ Excessive peptidase enzyme, which cleaves procollagen
- 2‑ A mutation in the DNA that codes for the chains of type I collagen
- 3‑ Tropocollagen bundles, which contain an ordered triple‑helical structure
- 4‑ Defective messenger RNA transcriptase
- 5‑ An increase in the number of cross‑links within the collagen structure
- Fig. 56
- Question 96.213
- Answer = 2
- Reference(s)
- Prockop DJ: Seminars in medicine of the Beth Israel Hospital, Boston: Mutations in collagen genes as a cause of connective‑tissue diseases. N Engl J Med 1992;326:540‑546. Sillence DO, Barlow KK, Cole WG, et al: Osteogenesis imperfecta type III: Delineation of the phenotype with reference to genetic heterogeneity. Am J Med Genet 1986;23:821‑832.
- 96.214 A 38‑year‑old woman with a 4‑month history of increasing hip pain is now
- unable to walk. An anteroposterior radiograph of the hip is shown in Figure 57a.
- Low‑ and highpower photomicrographs of the biopsy specimen are shown in
- Figures 57b and 57c. The most likely diagnosis is
- 1‑ multiple myeloma.
- 2‑ mastocytosis.
- 3‑ Paget's disease.
- 4‑ metastatic adenocarcinoma.
- 5‑ osteonecrosis.
- Go to next slide for figures and answer link
- Fig. 57a
- Figure 57b
- Fig. 57c
- Question 96.214
- Answer = 4
- Reference(s)
- Bone Tumors, in Dahlin DC, Unni KK (eds): Conditions That Commonly Simulate Primary Neoplasms of Bone: General Aspects and Data on 8,542 Cases, ed 4. Springfield, IL, Charles C Thomas, 1986, pp 406‑481.
- 96.215 Normal mineralization of the matrix occurs in which area of the physis?
- 1‑ Throughout the growth plate
- 2‑ Reserve zone
- 3‑ Proliferative zone
- 4‑ Upper hypertrophic zone
- 5‑ Lower hypertrophic zone
- Question 96.215
- Answer = 5
- Reference(s)
- Boskey AL. Maresca M, Armstrong AL, et al: Treatment of proteoglycan aggregates with physeal enzymes reduces their ability to inhibit hydroxyapatite proliferation in a gelatin gel. J Orthop Res 1992;10:313‑319. Simon SR (ed): Orthopaedic Basic Science. Rosemont, IL, American Academy of Orthopaedic Surgeons, 1994, pp 185‑217.
- 96.216 A 12‑year‑old girl with myelodysplasia has diminished ambulatory capacity and her scoliosis
- has progressed (Figures 58a and 58b). The physician should recommend
- 1‑ observation for further progression.
- 2‑ orthotic adjustment.
- 3‑ a bone scan.
- 4‑ an MRI scan.
- 5‑ fusion and instrumentation.
- Fig. 58a
- Fig. 58b
- answer
- back
- Question 96.216
- Answer = 4
- back to this question
- next question
- Reference(s)
- Lindseth RE: Myelomeningocele spine, in Weinstein SL (ed): The Pediatric Spine: Principles and Practice. New York, NY, Raven Press, 1994, pp 1043‑1067.
- 96.217 What is the most common anatomic relationship in the posterior aspect of the
- hip?
- 1‑ Superior and inferior gluteal nerves exit superior to the piriformis.
- 2‑ Superior and inferior gluteal nerves exit inferior to the piriformis.
- 3‑ Superior and inferior gluteal nerves and the sciatic nerve exit inferior to the
- piriformis.
- 4‑ Superior gluteal nerve exits superior to the pmformis, and the sciatic nerve
- exits inferior to the piriformis.
- 5‑ Sciatic, superior, and inferior gluteal nerves exit superior to the piriformis.
- Question 96.217
- Answer = 4
- Reference(s)
- Hollinshead WH: Anatomy for Surgeons: Back and Limbs. New York, NY, Hamper & Row, 1969, p 680.
- 96.218 A 68‑year‑old woman with breast cancer undergoes prophylactic intramedullary
- nailing of the femur. When she is seen for suture removal 2 weeks later, her
- family states that she is anorexic, lethargic, nauseous, and has been vomiting.
- Which of the following serum levels should be checked?
- 1‑ Albumin
- 2‑ Hemoglobulin
- 3‑ Serum protein electrophoresis
- 4‑ Calcium
- 5‑ Amylase
- Question 96.218
- Answer = 4
- Reference(s)
- Frassica FJ: Metastatic Bone Disease: General Principles, Pathophysiology, Evaluation, and Biopsy, in Frassica FJ, Gitelis S, Sim FH (ed): Instructional Course Lectures 41. Rosemont, IL, American Academy of Orthopaedic Surgeons, 1992, pp 293‑300.
- 96.219 An 8‑year‑old boy has a closed midcervical fracture of the proximal femur that is displaced
- 1 mm but not angulated. Recommended treatment should be
- 1‑ use of a hip spica cast.
- 2‑ open reduction and internal fixation with threaded screws and use of a cast.
- 3‑ internal fixation in situ with threaded screws and use of a cast.
- 4‑ skin traction for 3 weeks followed by use of a cast.
- 5‑ a prophylactic vascularized fibula graft.
- Question 96.219
- Answer = 3
- Reference(s)
- Hughes LO, Beaty JH: Fractures of the head and neck of the femur in children. J Bone Joint Surg 1994;76A:283‑292.
- 96.220 Which of the following methods of extensor tendon repair is most likely to
- result in normal hand function?
- 1‑ Over the dorsum of the finger with static splinting
- 2‑ Over the dorsum of the finger with dynamic splinting
- 3‑ Over the dorsum of the hand with static splinting
- 4‑ Over the dorsum of the hand with dynamic splinting
- 5‑ Over the dorsum of the wrist with dynamic splinting
- Question 96.220
- Answer = 4
- Reference(s)
- Browne EZ Jr, Ribik CA: Early dynamic splinting for extensor tendon injuries. J Hand Surg 14A:72‑76. Newport ML, Blair WF, Steyers CM Jr: Long‑term results of extensor tendon repair. J Hand Surg 1990;15A:961‑966.
- 96.221 A 45‑year‑old woman with insulin‑dependent diabetes mellitus has a 2 x 2‑cm
- area of dry gangrene on the plantar surface of the first metatarsophalangeal joint.
- The Doppler ankle‑brachial index is 0.2, and the toe pressures are 35 mm Hg.
- The next step in management should be
- 1‑ below‑knee amputation.
- 2‑ transmetatarsal amputation.
- 3‑ vascular surgery consultation.
- 4‑ total contact casting.
- 5‑ fitting for an ankle‑foot orthosis.
- Question 96.221
- Answer = 3
- Reference(s)
- Brodsky JW: The diabetic foot, in Mann RA, Coughlin MJ: Surgery of the Foot and Ankle, ed 6. St Louis, MO. CV Mosby, 1993, pp 877‑958. Karmody AM, Jacobs RL: Salvage of the diabetic foot by vascular reconstruction. Orthop Clin North Am 1976;7:957‑977. Serletti JM, Hurwitz SR, Jones JA, et al: Extension of limb salvage by combined vascular reconstruction and adjunctive free‑tissue transfer. J Vasc Surg 1993;18:972‑980.
- 96.222 Excision of the superior tuberosity of the calcaneus through a lateral approach
- may result in laceration of what nerve or neural structure?
- 1‑ Deep peroneal
- 2‑ Superficial peroneal
- 3‑ Lateral calcaneal branch of the sural
- 4‑ Lateral dorsal cutaneous
- 5‑ Saphenous
- Question 96.222
- Answer = 3
- Reference(s)
- Sarrafian SK: Anatomy of the Foot and Ankle: Descriptive, Topographic, Functional. Philadelphia, PA, JB Lippincott, 1983, p 313.
- 96.223 Which of the following would most likely be seen in a synovial biopsy specimen
- obtained from a patient with electrically documented idiopathic carpal tunnel
- syndrome?
- 1‑ Fibrous tissue with variable edema
- 2‑ Chronic inflammation
- 3‑ Acute and chronic inflammation
- 4‑ Hypervascularity
- 5‑ Giant cell synovitis
- Question 96.223
- Answer = 1
- Reference(s)
- Kerr CD, Sybert DR, Albarracin NS: An analysis of the flexor synovium in idiopathic carpal tunnel syndrome: Report on 625 cases. J Hand Surg 1992;17A:1028‑1030. Fuchs PC, Nathan PA, Myers LD: Synovial histology in carpal tunnel syndrome. J Hand Surg 1991;16A:753‑758.
- 96.224 A 22‑year‑old man sustains a closed fracture of the left femoral shaft, and the
- femur is placed in skeletal traction. Two days later, intramedullary nailing is
- performed. One hour after surgery, the patient goes into respiratory distress with
- dyspnea, tachycardia, and anxiety. He has a pulse of 110/min; respirations of
- 40/min, blood pressure of 126/66 mm Hg, and a temperature of 38.5°C (101.3').
- Pulse oximeter saturation is 80% on 2 L of nasal oxygen. Hematocrit is 22%0. A
- chest radiograph shows mild, diffuse infiltrates. Mild swelling of both lower
- extremities is noted but there are no petechiae. Prothrombin time is 13 seconds
- with an International Normalized Ratio of 1.1 (normal 0.9 to 1.1). Initial treatment
- should include
- 1‑ transfusion with 2 units of packed red blood cells and 8 units of fresh frozen
- plasma.
- 2‑ ventilation‑perfusion scan.
- 3‑ anticoagulation with heparin.
- 4‑ increase in ventilatory support and oxygenation with close monitoring.
- 5‑ IV furosemide.
- Question 96.224
- Answer = 4
- Reference(s)
- Gleis GE, Seligson D: Diagnosis and treatment of complications, in Browner BD, Jupiter JB, Levine AM, et al (eds): Skeletal Trauma: Fractures, Dislocations, Ligamentous Injuries. Philadelphia, PA, WB Saunders, 1992, pp 443‑470.
- 96.225 A 60‑year‑old man has osteoarthritis of the knee and is scheduled to undergo total
- knee arthroplasty. The surgeon chooses not to resurface the patella. What is the
- most common complication that may occur at the patellofemoral joint?
- 1‑ Fracture of the patella
- 2‑ Subluxation of the patella
- 3‑ Rupture of the patellar tendon
- 4‑ Chronic peripatellar pain
- 5‑ Tibial component loosening
- Question 96.225
- Answer = 4
- Reference(s)
- Boyd AD Jr, Ewald FC, Thomas WH, et al: Long‑term complications after total knee arthroplasty with or without resurfacing of the patella. J Bone Joint Surg 1993;75A:674‑681.
- 96.226 Figure 59 is the Moseley straight line
- graph of a patient with a congenital
- short femur. How many centimeters
- is the expected limb length discrepancy
- at maturity?
- 1‑ 4
- 2‑ 7
- 3‑ 10
- 4‑ 15
- 5‑ 20
- Fig. 59
- Question 96.226
- Answer = 2
- Reference(s)
- Moseley CF: Leg‑length discrepancy, in Morrissy RT (ed): Lovell and Winter's Pediatric Orthopaedics, ed 3. Philadelphia, PA, JB Lippincott, 1990, vol 2, pp 767‑813.
- 96.227 A healthy, active, 10‑year‑old girl has a left scapular prominence, slight truncal
- asymmetry, and hyperreflexia in the lower extremities. A radiograph is shown in
- Figure 60. Recommended management should be
- 1‑ observation.
- 2‑ thoracolumbar orthosis.
- 3‑ supine bending radiographs.
- 4‑ MRI scan of the spine.
- 5‑ CT scan of the spine.
- Fig. 60
- answer
- back
- Question 96.227
- Answer = 4
- back to this question
- next question
- Reference(s)
- Bradford DS, Hu SS: Neuromuscular spinal deformity, in Moe's Textbook of Scoliosis and Other Spinal Deformities, ed 3. Philadelphia, PA, WB Saunders, 1995, pp 295‑322. Gurr KR, Taylor TK, Stobo P: Syringomyelia and scoliosis in childhood and adolescence. J Bone Joint Surg 1988;70B:159.
- 96.228 A 34‑year‑old man complains of persistent swelling and pain in the anterolateral
- ankle for 3 months following a sprain that occurred during a basketball game. He
- has had multiple sprains of both ankles in the past. Radiographs of the ankle are
- shown in Figures 61a and 61b. Arthroscopy was performed and a microscopic
- section of the synovium is shown in Figure 61c. Recommended treatment should
- be
- 1‑ synovectomy.
- 2‑ synovectomy and rifampin/ethanbutol.
- 3‑ synovectomy and cephalosporin.
- 4‑ arthrodesis.
- 5‑ lateral ligament reconstruction.
- Go to next slide for figures and answer link
- Fig. 61a
- Fig. 61b
- Fig. 61c
- Question 96.228
- Answer = 1
- Reference(s)
- Ferkel RD: Arthroscopy of the ankle and foot, in Mann RA, Coughlin MJ: Surgery of the Foot and Ankle, ed 6. St Louis, MO, CV Mosby, 1993, pp 1277‑1312. Rosenberg AE, Schiller AL, Kelley WN, et al: Textbook of Rheumatology. Philadelphia, PA, WB Saunders, 1993, p 1663.
- 96.229 Which noninvasive bone density study will give the most reliable indication of
- risk (positive predictive value) for hip fracture?
- 1‑ Quantitative CT scan of the spine
- 2‑ Dual photon absorptiometry of the spine
- 3‑ Dual energy x‑ray absorptiometry of the lumbar spine
- 4‑ Dual energy x‑ray absorptiometry of the distal radius
- 5‑ Dual energy x‑ray absorptiometry of the proximal femur
- Question 96.229
- Answer = 5*
- Reference(s)
- Cummings SR, Black DM, Nevin MC, et al: Bone density at various sites for prediction of hip fractures: The study of the Osteoporotic Fractures Research Group. Lancet 1993;341:72‑75. Lane JM, Healey JH: Fractures of the hip, in Lane JM (ed): Diagnosis and Management of Pathologic Fractures. New York, NY, Raven Press, 1993, pp122‑127. *item deleted after statistical review
- 96.230 A 30‑year‑old man requires an above‑knee amputation following an industrial accident in which his limb was amputated 15 cm above the tendon insertion of the adductor magnus. The major deforming force results in abduction and flexion. Which of the following procedures will most effectively prevent this complication?
- 1‑ Gluteus medius and minimus tenotomies
- 2‑ Psoas major tenotomy
- 3‑ Groin adductor tissue roll
- 4‑ Adductor myodesis to the femur
- 5‑ Adductor brevis and longus tenotomies
- answer
- back
- Question 96.230
- Answer = 4
- back to this question
- next question
- Reference(s)
- Gottchalk FA, Kourosh S, Stills M, et al: Does socket configuration influence the position of the femur in above knee amputations. J Prosthet Orthop 1989;2:94‑102. Frymoyer JW (ed): Orthopaedic Knowledge Update 4. Rosemont, IL, American Academy of Orthopaedic Surgeons, 1993, p 259‑273.
- 96.231 Which of the following statements best describes the relationship of the popliteus
- tendon to the fibular collateral ligament at the level of the lateral femoral condyle?
- 1- The popliteus runs medial to the fibular collateral ligament to insert distal to the attachment
- of the fibular collateral ligament on the lateral femoral condyle.
- 2- The popliteus runs lateral to the fibular collateral ligament to insert distal to the attachment
- of the fibular collateral ligament on the lateral femoral condyle.
- 3- The popliteus runs medial to the fibular collateral ligament to insert proximal to the
- attachment of the fibular collateral ligament on the lateral femoral condyle.
- 4- The popliteus runs lateral to the fibular collateral ligament to insert proximal to the
- attachment of the fibular collateral ligament on the lateral femoral condyle.
- 5- The popliteus runs medial to the fibular collateral ligament to insert as a conjoined structure
- with the fibular collateral ligament on the lateral femoral condyle.
- Question 96.231
- Answer = 1
- Reference(s)
- Anderson JE (ed): Grant's Atlas of Anatomy, ed 8, Anderson JE (ed): Baltimore, MD, Williams and Wilkins, 1983.
- 96.232 A 6‑year‑old girl has a history of spontaneous back pain in the lumbar region that is worse at night and not relieved by nonsteroidal and‑inflammatory drugs. Examination reveals a list to the left with forward bending and a positive popliteal nerve compression test. A bone scan and CT scan are shown in Figures 62a and 62b. Histology of an excisional biopsy of the lesion is shown in Figures 62c and 62d. The most likely diagnosis is
- 1‑ osteoblastoma.
- 2‑ aneurysmal bone cyst.
- 3‑ giant cell tumor.
- 4‑ osteosarcoma.
- 5‑ neuroblastoma.
- Fig. 62b
- Fig. 62a
- Go to next slide for remaining figures and answer link
- Fig. 62c Fig. 62d
- Question 96.232
- Answer = 1
- Reference(s)
- Benign Osteoblastoma (Giant osteoid osteoma), in Dahlin DC, Unni KK( eds): Tumors, ed 4. Springfield, IL, Charles C Thomas, 1986, pp 102‑118. Benign osteoblastoma, in Lichtenstein L (ed): Bone Tumors, ed 5. St Louis, MO, CV Mosby, 1977, pp 92‑111.
- 96.233 A 20‑year‑old man involved in a motorcycle accident sustains a degloving injury to
- the dorsolateral aspect of his foot and an ankle dislocation. Distal neurovascular
- evaluation reveals an intact posterior tibial pulse and normal sensation on the
- plantar surface of the foot. Following debridement and stabilization of the ankle
- dislocation (Figure 63), softtissue coverage is elected. Which type of soft‑tissue
- coverage is the best treatment?
- 1‑ Split‑thickness skin graft
- 2‑ Cross‑leg flap
- 3‑ Soleus muscle flap
- 4- Latissimus dorsi free flap
- 5‑ Local transposition flap
- Figure 63
- Question 96.233
- Answer = 4
- Reference(s)
- Gorman PW, Barnes CL, Fischer TJ, et al: Soft‑tissue reconstruction in severe lower extremity trauma: A review. Clin Orthop 1989;243:57‑64. Swartz WM, Mears DC: The role of free‑tissue transfers in lower‑extremity reconstruction. Plast Reconstr Surg 1985;76:364‑373.
- 96.234 A 30‑year‑old man with a 3‑year history of constant left elbow pain is unable to
- fully extend his elbow secondary to pain. An anteroposterior radiograph of the
- distal humerus is shown in Figure 64a. A technetium bone scan is shown in
- Figure 64b. A CT scan through the olecranon fossa is shown in Figure 64c.
- Low‑ and high‑power photomicrographs of a biopsy specimen are shown in
- Figures 64d and 64e. The most likely diagnosis is
- 1‑ osteomyelitis.
- 2‑ osteoid osteoma.
- 3‑ osteosarcoma.
- 4‑ malignant fibrous histiocytoma.
- 5‑ nonossifying fibroma.
- Fig. 64b
- Fig. 64a
- Go to next slide for remaining figures and answer link
- Fig. 64c
- Fig. 64d
- Fig. 64e
- Question 96.234
- Answer = 2
- Reference(s)
- Bone tumors, in Dahlin DC, Unni KK (eds): Conditions That Commonly Simulate Primary Neoplasms of Bone. General Aspects and Data on 8,542 Cases, ed 4. Springfield, IL, Charles C Thomas, 1986, pp 406‑481.
- 96.235 A 5‑year‑old girl falls from a deck and sustains the injury shown in Figure 65. Neurologic
- and vascular status are normal. The next step in treatment should consist of
- 1‑ acceptance of fracture position and cast immobilization.
- 2‑ primary open reduction and internal fixation.
- 3‑ excision of the radial head and cast immobilization.
- 4‑ attempted closed reduction and cast immobilization.
- 5‑ attempted closed reduction and transcapitellar pin fixation.
- Fig. 65
- Question 96.235
- Answer = 4
- Reference(s)
- Bernstein SM. McKeever P, Bernstein L: Percutaneous reduction of displaced radial neck fractures in children. J Pediatr Orthop 1993;13:85‑88.
- 96.236 A 21‑year‑old man noted a firm soft‑tissue mass on the posterolateral aspect of his
- distal thigh 1 year ago. The mass has remained asymptomatic, but has enlarged
- slightly over that period of time. Plain radiographs show a soft‑tissue shadow
- without mineralization or involvement of the underlying bone. An MRI study
- reveals the fusiform mass shown in Figure 66a (T2 weighted sagittal image),
- Figure 66b (T,‑weighted axial image), and Figure 66c (T2 weighted axial image). A
- core needle biopsy reveals tissue shown in Figure 66d, and at the time of the
- procedure, the mass is found to be immediately adjacent but easily separable from
- branches of the common peroneal nerve. The surgeon should perform which of
- the following procedures?
- 1‑ Excision of the tumor and nerve, with immediate nerve grafting
- 2‑ High above‑knee amputation
- 3‑ Wide resection, including an appropriate margin of normal nerve proximal and
- distal to the lesion
- 4‑ Radical resection of the entire posterior compartment of the thigh
- 5‑ Marginal excision of the tumor, preserving all the nerve fibers
- Go to next slide for figures and answer link
- Fig. 66a
- Fig. 66b
- Fig. 66c
- Fig. 66d
- Question 96.236
- Answer = 5
- Reference(s)
- Enzinger FM, Weiss SW: Soft Tissue Tumors, ed 3. St Louis, MO. CV Mosby, 1995.
- 96.237 A 34‑year‑old man falls from his bicycle and injures his right shoulder (Figure 67).
- Neurovascular examination is normal. When the patient is asked to contract the
- deltoid with the arm passively restrained, the deformity lessens. Recommended
- treatment should be
- 1‑ open reduction and repair of the acromioclavicular ligaments.
- 2‑ closed reduction of the acromioclavicular joint, followed by use of a sling for 4
- weeks.
- 3‑ arthroscopically assisted percutaneous placement of a coracoclavicular screw
- for reduction and fixation.
- 4‑ use of a sling for comfort with range of motion exercises as tolerated.
- 5‑ use of a Kenny Howard brace for 10 days, followed by use of a sling and active
- range of motion exercises for 3 weeks.
- Figure 67
- answer
- back
- Question 96.237
- Answer = 4
- back to this question
- next question
- Reference(s)
- Allman FL Jr: Fractures and ligamentous injuries of the clavicle and its articulation. J Bone Joint Surg 1967;49A:774‑784. Miller ME, Ada JR: Fractures of the scapula, clavicle, and glenoid, in Browner BD, Jupiter JB, Levine AM, et al (eds): Skeletal Trauma: Fractures, Dislocations, Ligamentous Injuries. Philadelphia, PA, WB Saunders, 1992, pp 1291‑1310.
- 96.238 At what point during walking does the highest net muscle moment at the
- ankle occur?
- 1‑ Initial contact
- 2‑ Midstance
- 3‑ Preswing
- 4‑ Terminal stance
- 5‑ Terminal swing
- Question 96.238
- Answer = 4
- Reference(s)
- Kasser JR (ed): Orthopaedic Knowledge Update 5. Rosemont, IL, American Academy of Orthopaedic Surgeons, 1996, pp 29‑40.
- 96.239 The molecular basis for achondroplasia has recently been established as a
- mutation within a receptor for what substance?
- 1‑ Fibroblast growth factor
- 2‑ Insulin‑like growth factor
- 3‑ Parathyroid‑related protein
- 4‑ Somatomedin
- 5‑ Vitamin D
- Question 96.239
- Answer = 1*
- Reference(s)
- Shiang R, Thompson LM, Zhu YZ, et al: Mutations in the transmembrane domain of FGFR3 cause the most common genetic form of dwarfism, achondroplasia. Cell1994;78:335‑342. Schipani E, Kruse K, Juppner H: A constitutively active mutant PTH‑PTHrP receptor in Jansen‑type metaphyseal chondrolysplasia. Science 1995;268:98‑100. Trippel SB: Basic science of the growth plate. COO 1990;1:279‑288. * item deleted after statistical review
- 96.240 A 40‑year‑old man has had pain and numbness radiating from his neck to his
- thumb and index forger for 6 weeks. The patient has no history of trauma or
- other medical problems. The most likely diagnosis is herniated nucleus pulposus
- at what level?
- 1‑ C3‑4
- 2‑ C4‑5
- 3‑ C5‑6
- 4‑ C6‑7
- 5‑ C7‑T1
- Question 96.240
- Answer = 3
- Reference(s)
- Clark CR: Degenerative conditions of the spine: Differential diagnosis and nonsurgical treatment, in Frymoyer JW (ed): The Adult Spine: Principles and Practice. 1991, vol 2, pp 1145‑1164.
- 96.241 Osteoporosis is reversible when it is the result of which of the following?
- 1‑ Steroid dependency
- 2‑ Systemic mastocytosis
- 3‑ Hyperthyroidism
- 4‑ Turner syndrome
- 5‑ Menopause
- Question 96.241
- Answer = 3
- Reference(s)
- Mankin HJ: Metabolic bone diseases, in Jackson DW (ed): Instructional Course Lectures 44. Rosemont, IL, American Academy of Orthopaedic Surgeons, 1995, pp 3‑29. Frymoyer JW (ed): Orthopaedic Knowledge Update 4. Rosemont, IL, American Academy of Orthopaedic Surgeons, 1993, pp 69‑88.
- next question
- 96.242 An orthopaedic surgeon and a patient disagree on the best approach for elective treatment of the patient's condition. Despite extensive discussions, this conflict cannot be resolved. The surgeon should take what course of action?
- 1‑ Perform the procedure he/she desires.
- 2‑ Perform the procedure the patient desires.
- 3‑ Proceed with the guidelines of the insurer.
- 4‑ Withdraw from the care of the patient.
- 5‑ Refer the patient for a second opinion.
- Question 96.242
- Answer = 5
- Reference(s)
- AAOS Committee on Professional Liability: Medical Malpractice: A Primer for Orthopaedic Residents and Fellows. Rosemont, IL, American Academy of Orthopaedic Surgeons, 1993, pp 63‑69.
- 96.243 A patient with median neuropathy at the wrist undergoes carpal tunnel
- injections of a steroid. Six hours later, the patient has dramatically increased pain
- and paresthesias in the hand. What is a likely cause of these symptoms?
- 1‑ Normal postinjection pain
- 2‑ Rupture of the flexor pollicus longus
- 3‑ Iatrogenic septic flexor tenosynovitis
- 4‑ Intraneural injection of steroid solution
- 5‑ Traumatic pseudoaneurysm of the ulnar artery
- Question 96.243
- Answer = 4
- Reference(s)
- Frederick HA, Carter PR, Littler JW: Injection injuries to the median and ulnar nerves at the wrist. J Hand Surg 1992;17A:645‑647. McConnell JR, Bush DC: Intraneural steroid injection as a complication in the management of carpal tunnel syndrome: A report of three cases. Clin Orthop 1990;250:181‑184.
- 96.244 A 32‑year‑old man with has had low‑grade aching in his left ankle for the past 3
- months. Plain radiographs are shown in Figures 68a and 68b, and MRI scans are
- shown in Figure 68c (coronal Tl‑weighted) and Figure 68d (sagittal T2 weighted).
- Histologic material from an incisional biopsy is shown in Figure 68e
- (hematoxylin and eosin). The most likely diagnosis is
- 1‑ nonossifying fibroma.
- 2‑ Campanacci's disease.
- 3‑ adamantinoma.
- 4‑ aneurysmal bone cyst.
- 5‑ giant cell tumor.
- Fig. 68a Fig. 68b
- Go to next slide for remaining figures and answer link
- Fig. 68c
- Fig. 68d
- Fig. 68e
- Question 96.244
- Answer = 5
- Reference(s)
- Giant cell tumor (Osteoclastoma), in Dahlin DC, Unni KK (eds): Bone tumors: General Aspects and Data on 8,542 Cases, ed 4. Springfield, IL, Charles C Thomas Publishers, 1986, pp 119‑140.
- 96.245 Which muscle has been partially resected to reveal the nerve to abductor digiti
- quinti muscle as shown in Figure 69?
- 1‑ Flexor hallucis longus
- 2‑ Flexor hallucis brevis
- 3‑ Flexor digitorum brevis
- 4‑ Adductor hallucis
- 5‑ Abductor hallucis
- Figure 69
- Question 96.245
- Answer = 5
- Reference(s)
- Baxter DE: Functional nerve disorders in the athlete's foot, ankle and leg, in Heckman JD (ed): Instructional Course Lectures 42. Park Ridge, IL, American Academy of Orthopaedic Surgeons, 1993, pp 185‑194.
- 96.246 A 65‑year‑old woman with a total knee replacement that has functioned satisfactorily for 3 years fell and sustained the fracture shown in Figures 70a and 70b. Which of the following treatment options is most likely to result in healing of the fracture and maintenance of knee function?
- 1‑ Revision with a long stem femoral component
- 2‑ Closed reduction and brace treatment
- 3‑ Open reduction and multiple interfragmentary screw fixation
- 4‑ Retrograde iniramedullary nailing through the knee
- 5‑ External fixation
- Fig. 70a Fig. 70b
- Question 96.246
- Answer = 4
- Reference(s)
- Rolston LR, Christ DJ, Halpern A, et al: Treatment of supracondylar fractures of the femur proximal to a total knee arrhroplasty: A report of four cases. J Bone Joint Surg 1995;77A:924‑931.
- next question
- 96.247 A 25‑year‑old man sustains a posterior dislocation of the hip in a motor vehicle
- accident. What nerve is most likely to be injured?
- 1‑ Femoral
- 2‑ Lateral femoral cutaneous
- 3‑ Saphenous
- 4‑ Sciatic
- 5‑ Obturator
- Question 96.247
- Answer = 4
- Reference(s)
- Levin P: Hip dislocations, in Browner BD, Jupiter JB, Levine AM, et al (eds): Skeletal Trauma. Philadelphia, PA, WB Saunders, 1992, vol 2, pp 1329‑1367.
- 96.248 Type I collagen abnormalities underlie what condition?
- 1‑ Fibrous dysplasia
- 2‑ Fibrodysplasia ossificans
- 3‑ Spondyloepiphyseal dysplasias
- 4‑ Osteogenesis imperfecta
- 5‑ Osteopetrosis
- Question 96.248
- Answer = 4
- Reference(s)
- Cole WG: Etiology and pathogenesis of heritable connective tissue diseases. J Pediatr Orthop 1993;13:392‑403.
- 96.249 Nonsurgical treatment of recurrent posterior subluxation of the glenohumeral joint
- emphasizes the strengthening of which of the following muscles?
- 1‑ Trapezius
- 2‑ Anterior deltoid
- 3‑ Latissimus dorsi
- 4‑ Serratus anterior
- 5- Infraspinatus
- answer
- back
- Question 96.249
- Answer = 5
- back to this question
- next question
- Reference(s)
- Fronek J, Warren RF, Bowen M: Posterior subluxation of the glenohumeral joint. J Bone Joint Surg 1989;71A:205‑216.
- 96.250 A 68‑year‑old man has a grade 1 chondrosarcoma of the distal femur that measures 6.5 x 8.5 cm on the anteroposterior radiograph. Cross‑sectional imaging studies show that the lesion is confined to bone. The radiograph and CT scan of the chest are normal. According to the staging studies of the Musculoskeletal Tumor Society, this patient's tumor would be classified as what stage?
- 1‑ IA
- 2‑ IB
- 3‑ I
- 4‑ IIA
- 5‑ III
- Question 96.250
- Answer = 1
- Reference(s)
- Simon SR (ed): Orthopaedic Basic Science. Rosemont, IL, American Academy of Orthopaedic Surgeons, 1994, pp 219‑276. Enneking WF, Sparrier SS, Goodman MA: Current concepts review: The surgical staging of musculoskeletal sarcoma. J Bone Joint Surg 1980;62A:1027‑1030.
- 96.251 A 33‑year‑old man sustains an open grade III‑B midtibial shaft fracture with
- soft‑tissue loss at the proximal medial aspect of the fracture as a result of a
- motorcycle accident. Which of the following procedures should be used for
- coverage of this defect?
- 1‑ Muscle flap within 24 hours
- 2‑ Muscle flap at 3 to 7 days
- 3‑ Muscle flap at 6 weeks
- 4‑ Full‑thickness skin grafting within 1 week
- 5‑ Split‑thickness skin grafting at 4 weeks
- Question 96.251
- Answer = 2
- Reference(s)
- Fischer MD, Gustilo RB, Varecka TF: The timing of flap coverage, bone‑grafting, and intramedullary nailing in patients who have a fracture of the tibial shaft with extensive soft‑tissue injury. J Bone Joint Surg 1991;73A:1316‑1322. Cierny G III, Byrd HS, Jones RE: Primary versus delayed soft tissue coverage for severe open tibial fractures: A comparison of results. Clin Orthop 1983;178:54‑63.
- 96.252 Figure 71 shows a complete transhumeral amputation in an 11‑year‑old boy. What
- is the recommended sequence of structural repair during replantation?
- 1‑ Skeletal stabilization; arterial shunt reperfusion; muscle, nerve, venous, and
- arterial repair
- 2- Skeletal stabilization; nerve, venous, arterial, and muscle repair
- 3‑ Skeletal stabilization; shunt placement to restore venous circulation; nerve and
- arterial repair
- 4‑ Skeletal stabilization; repair of the larger veins, muscles, and nerves in the arm;
- arterial repair
- 5‑ Skeletal stabilization; arterial, venous, and nerve repair, muscle suture
- Fig. 71
- Question 96.252
- Answer = 5
- Reference(s)
- Wood MB, Cooney WP III: Above elbow limb replantation: Functional results. J Hand Surg 1986;11A:682‑687.
- 96.253 An otherwise healthy 8‑year‑old boy has a 4‑month history of increasing
- shoulder pain. An anteroposterior radiograph of the shoulder is shown in Figure
- 72a. A transverse CT scan is shown in Figure 72b. Low‑ and high‑power
- photomicrographs of a biopsy specimen are shown in Figures 72c and 72d. The
- most likely diagnosis is
- 1‑ osteosarcoma.
- 2‑ osteomyelitis.
- 3‑ osteoblastoma.
- 4‑ aneurysmal bone cyst.
- 5‑ giant cell tumor.
- Fig. 72a
- Fig. 72b
- Go to next slide for remaining figures and answer link
- Fig. 72c
- Fig. 72d
- Question 96.253
- Answer = 3*
- Reference(s)
- Bone tumors, in Dahlin DC, Unni KK (eds): Conditions that Commonly Simulate Primary Neoplasms of Bone: General Aspects and Data on 8,542 Cases, ed 4. Springfield, IL, Charles C Thomas Publishers, 1986, pp 406‑481. *item deleted after statistical review
- 96.254 An otherwise healthy 11‑year‑old girl has a 3‑month history of increasing knee pain that is absent at night. An anteroposterior radiograph of the proximal tibia is shown in Figure 73a. The coronal and transverse T,_ MRI scans are shown in Figures 73b and 73c, respectively. Low‑ and high‑power photomicrographs of the biopsy specimen are shown in Figures 73d and 73e, respectively. The most likely diagnosis is
- 1‑ giant cell tumor.
- 2‑ chondroblastoma.
- 3‑ aneurysmal bone cyst.
- 4‑ unicameral bone cyst.
- 5‑ osteosarcoma.
- Figure 73a
- Go to next slide for remaining figures and answer link
- Fig. 73b
- Fig. 73c
- Fig. 73d
- Fig. 73e
- Question 96.254
- Answer = 3
- Reference(s)
- Bone tumors, in Dahlin DC, Unni KK (eds): Conditions that Commonly Simulate Primary Neoplasms of Bone. General Aspects and Data on 8,542 Cases, ed 4. Springfield, IL, Charles C Thomas Publishers, 1986, pp 406‑481.
- 96.255 The patient whose radiographs are shown in Figures 74a through 74c has anterior
- pain along the joint line, with tenderness and very little ankle motion. He is
- otherwise healthy and weighs 175 lb. He asks for a total ankle replacement.
- Arthroscopic synovectomy and removal of loose bodies were performed 2 years
- ago. Which of the following methods should be used to manage this patient's
- condition?
- 1‑ Use of a spherical design implant
- 2‑ Use of a cylindrical design implant
- 3‑ Use of a hinged implant
- 4‑ Use of an ankle‑foot orthosis
- 5‑ Repeat synovectomy
- Fig. 74a Fig. 74b
- Fig. 74c
- Question 96.255
- Answer = 4
- Reference(s)
- Kitaoka HB, Patzer GL, Ilstrup DM, et al: Survivorship analysis of the Mayo total ankle arthoplasty. J Bone Joint Surg 1994;76A:974‑979. Lachiewicz PF, Inglis AE, Ranawat CS: Total ankle replacement in rheumatoid arthritis. 1 Bone Joint Surg 1984;66A:340‑343.
- 96.256 A 14‑year‑old boy who has had back pain across the thoracolumbar junction for 6
- weeks complains of hamstring tightness and a limitation of forward bending. The
- patient has no history of trauma, and neurologic examination is normal.
- Radiographs are shown in Figures 75a and 75b. Recommended management
- should consist of
- 1‑ N antibiotics.
- 2‑ surgical debridement and N antibiotics.
- 3‑ activity limitation.
- 4‑ anterior and posterior spinal fusions.
- 5‑ posterior spinal fusion.
- Fig. 75a Fig. 75b
- answer
- back
- Question 96.256
- Answer = 3
- back to this question
- next question
- Reference(s)
- Blumenthal SL, Roach J, Herring JA: Lumbar Scheuermann's: A clinical series and classification. Spine 1987;12:929‑932. Greene TL, Hensinger RN, Hunter LY: Back pain and vertebral changes simulating Scheuermann's disease. J Pediatr Orthop 1985;5:1‑7.
- 96.257 The muscle contraction forces acting at the ankle during the normal
- stance phase of gait may be best characterized as a sequence of
- 1‑ eccentric by plantar flexors, then concentric by dorsiflexors.
- 2‑ eccentric by dorsiflexors, then concentric by plantar flexors.
- 3‑ eccentric by dorsiflexors, then plantar flexors, and then concentric by plantar
- flexors.
- 4‑ concentric by plantar flexors and then dorsiflexors.
- 5‑ concentric by dorsiflexors and then plantar flexors.
- Question 96.257
- Answer = 3
- Reference(s)
- Gage JR: The clinical use of kinetics for evaluation of pathologic gait in cerebral palsy. J Bone Joint Surg 1994;76A:622‑631.
- 96.258 Which of the following nerves is at greatest risk for injury during stabilization of
- a capsular shift for recurrent anterior shoulder instability?
- 1‑ Suprascapular
- 2‑ Radial
- 3‑ Ulnar
- 4‑ Musculocutaneous
- 5‑ Axillary
- Question 96.258
- Answer = 5
- Reference(s)
- Loomer R, Graham B: Anatomy of the axillary nerve and its relation to inferior capsular shift. Clin Orthop 1989;243:100‑105.
- 96.259 A right‑handed 78‑year‑old woman has a 2‑year history of aching in her right
- shoulder after increased activity, but she can perform activities of daily living
- and has no complaints of functional limitation. Examination shows some
- limitation of shoulder external rotation and elevation, and radiographs reveal a
- chronic posterior shoulder dislocation. Recommended treatment should be
- 1‑ observation.
- 2‑ closed reduction.
- 3‑ open reduction.
- 4‑ humeral head replacement.
- 5‑ transfer of the lesser tuberosity.
- Question 96.259
- Answer = 1
- Reference(s)
- Matsen FA III, Thomas SC, Rockwood CA Jr: Anterior glenohumeral instability, in Rockwood CA, Matsen FA HI (eds): The Shoulder. Philadelphia, PA, WB Saunders, 1990, vol 1, pp 526‑622.
- next question
- 96.260 An 18‑year‑old woman has symptomatic complete osteonecrosis of the second
- metatarsal head (Frieberg III) that has failed to respond to nonoperative
- management. The most effective surgical treatment is
- 1‑ dorsal cheilectomy.
- 2‑ arthrodesis of the metatarsophalangealjoint.
- 3‑ partial excision of the metatarsal head and joint debridement.
- 4‑ complete excision of the metatarsal head.
- 5‑ Silastic arthroplasty.
- Question 96.260
- Answer = 3
- Reference(s)
- Lutter LD, Mizel MS, Pfeffer GB (eds): Orthopaedic Knowledge Update: Foot and Ankle. Rosemont, IL, American Academy of Orthopaedic Surgeons, 1993, pp 163‑177. Mann RA, Coughlin MJ: Keratotic disorders of the plantar skin, in Mann RA, Coughlin MJ (eds): Surgery of the Foot and Ankle, ed 6. St Louis, MO, Mosby Year‑Book, 1993, pp 413‑467.
- 96.261 Which of the following is the most prevalent glycosaminoglycans in cartilage
- proteoglycans?
- 1‑ Chondroitin sulfate
- 2‑ Dermatan sulfate
- 3‑ Keratan sulfate
- 4‑ Hyaluronate
- 5‑ Fibronectin
- Question 96.261
- Answer = 1
- Reference(s)
- Simon SR (ed): Orthopedic Basic Science. Rosemont, IL, American Academy of Orthopaedic Surgeons, 1994, pp 1‑44.
- 96.262 Selective dorsal rhizotomy would be contraindicated in a child who has cerebral palsy and
- which of the following characteristics?
- 1‑ Mixed (athetoid‑spastic) movement disorder
- 2‑ Age 5 years or older
- 3‑ No prior orthopaedic surgery
- 4‑ The ability to walk
- 5‑ Spastic diplegia
- Question 96.262
- Answer = 1
- Reference(s)
- Boscarino LF, Ounpuu S, Davis RB III, et al: Effects of selective dorsal rhizotomy on gait in children with cerebral palsy. J Pediatr Orthop 1993;13:174‑179.
- 96.263 In a medical malpractice lawsuit, discovery is best defined as what process?
- 1‑ Plaintiff learns of his injury.
- 2‑ Plaintiff and defendant each find out about the other party's case.
- 3‑ Defendant receives a notice of claim from the plaintiff.
- 4‑ Defendant learns about the date of trial.
- 5‑ Defendant learns the identity of the plaintiffs expert witness.
- Question 96.263
- Answer = 2
- Reference(s)
- AAOS Committee on Professional Liability: Medical Malpractice: A Primer for Orthopedic Residents and Fellows. Rosemont, IL, American Academy of Orthopaedic Surgeons, 1993, p 8.
- 96.264 A patient with an ankle injury, which includes a spiral fracture of the distal fibula
- 5 cm proximal to the ankle, is treated with anatomic reduction and stable
- fixation of the fibula. With which of the following injuries would placement of a
- syndesmotic screw most likely be necessary in this patient?
- 1‑ A reduced and stabilized oblique medial malleolus fracture
- 2‑ A reduced and stabilized transverse medial malleolus fracture
- 3‑ A deep deltoid ligament rupture
- 4‑ An anterior tibiofibular ligament rupture
- 5‑ An avulsion of Chaput tubercle
- Question 96.264
- Answer = 3
- Reference(s)
- Solari J, Benjamin J, Wilson J, et al: Ankle mortise stability in Weber C fractures: Indications for syndesmostic fixation. J Orthop Trauma 1991;5:190‑195. Boden SD, Labrapoulos PA, McCowin P, et al: Mechanical considerations for the syndesmosis screw: A cadaver study. J Bone Joint Surg 1989;71A:1548‑1555. Yamaguchi K, Martin CH. Boden SD, et al: Operative treatment of syndesmotic disruptions without use of a syndesmotic screw: A prospective clinical study. Foot Ankle Int 1994;15:407414.
- next question
- 96.265 The primary soft‑tissue restraints to anterior glenohumeral translation in the
- shoulder include what ligaments?
- 1‑ Coracohumeral and anterior‑superior glenohumeral
- 2‑ Posterior‑inferior and anterior‑inferior glenohumeral
- 3‑ Coracohumeral and posterior‑inferior glenohumeral
- 4‑ Anterior‑superior and posterior‑inferior glenohumeral
- 5‑ Posterior‑superior and anterior‑inferior glenohumeral
- Question 96.265
- Answer = 2
- Reference(s)
- O'Brien SJ, Schwartz RS, Warren RF, et al: Capsular restraints to anterior‑posterior motion of the abducted shoulder: A biomechanical study. J Shoulder Elbow Surg 1995;4:298‑308. Turkel SJ, Panio MW, Marshall JL, et al: Stabilizing mechanisms preventing anterior dislocation of the glenohumeral joint. J Bone Joint Surg 1981;63A:1208‑1217. Warner JJP, Cahoru DNM, Berger R, et al: Dynamic capsuloligamentosis anatomy of the glenohumeral joint. J Shoulder Elbow Surg 1993;2:115.
- 96.266 What is the approximate increase in kinetic energy of a passenger in a car
- traveling 70 mph compared to one in a car traveling 50 mph?
- 1‑ One third
- 2‑ One half
- 3‑ Twofold
- 4‑ Fourfold
- 5‑ Eightfold
- Question 96.266
- Answer = 3
- Reference(s)
- Wiss DA, Gellman H: Gunshot wounds to the musculoskeletal system, in Browner BD, Jupiter JB, Levine AM, et al: Skeletal Trauma: Fractures, Dislocations, and Ligamentous Injuries. Philadelphia, PA, WB Saunders, 1992, pp 367‑400. Giancoli D: Physics: Principles With Applications, ed 3. Inglewood Cliffs, NJ, Prentice‑Hall, 1991, p 128.
- next question
- 96.267 What factor can cause circumduction during the swing phase of gait in an
- above‑knee amputation?
- 1‑ Thin medial brim
- 2‑ Stump seated too far into the socket
- 3‑ Excessive friction at the knee joint
- 4‑ Flexion contracture
- 5‑ Adduction contracture
- answer
- back
- Question 96.267
- Answer = 3
- back to this question
- Reference(s)
- Kostuik JP: Amputation Surgery and Rehabilitation. New York, NY, Churchill‑Livingstone,1981,pp 315‑330.
- next question
- 96.268 A 10‑year‑old boy with no history of trauma has had limping related to left thigh
- and knee pain for 6 weeks. Radiographs of the knee are normal. Anteroposterior
- and frog lateral hip radiographs are shown in Figures 76a and 76b, respectively.
- Recommended treatment for the left hip is
- 1‑ crutches and no weightbearing.
- 2‑ in situ screw fixation.
- 3‑ closed reduction and screw fixation.
- 4‑ femoral neck osteotomy, reduction, and screw fixation.
- 5‑ intertrochanteric osteotomy.
- Fig. 76a Fig. 76b
- Question 96.268
- Answer = 2
- Reference(s)
- Boyer DW, Mickelson MR. Ponseti IV: Slipped capital femoral epiphysis: Long‑term follow‑up study of one hundred and twenty‑one patients. J Bone Joint Surg 1981;63A:85‑95. Morrissy RT: Slipped capital femoral epiphysis, in Morrissy RT (ed): Lovell and Winter's Pediatric Orthopaedics, ed 3. Philadelphia, PA, JB Lippincott, 1990, vol 2, pp 885‑904.
- 96.269 In the preparation of an independent medical evaluation, what component is
- most important in determining a patient's impairment?
- 1‑ Medical report from the company physician
- 2‑ The workers' compensation carrier's guidelines
- 3‑ The workers' compensation carrier's evaluation
- 4‑ Physicians objective evaluation
- 5‑ Patient's account of the injury and resulting limitations
- Question 96.269
- Answer = 4
- Reference(s)
- AAOS Committee on Occupational Health: A Physician's Primer on Workers‘ Compensation. Park Ridge, IL, American Academy of Orthopaedic Surgeons, 1992, pp 33‑39.
- 96.270 A 4‑year‑old girl falls and fractures her femur. Radiographs obtained after closed
- reduction and hip spica cast application are shown in Figures 77a and 77b. The
- physician should now recommend
- 1‑ cast removal, then skin traction.
- 2‑ continued use of the current cast.
- 3‑ wedging of the cast.
- 4‑ external fixation.
- 5‑ internal fixation with a plate.
- Fig. 77a Fig. 77b
- Question 96.270
- Answer = 2
- Reference(s)
- Henderson OL, Morrissy RT, Gerdes MH, et al: Early casting of femoral shaft fractures in children. J Pediatr Orthop 1984;4:16‑21. Irani RN, Nicholson JT, Chung SM: Long‑term results in the treatment of femoral shaft fractures in young children by immediate spica immobilization. J Bone Joint Surg 1976;58A:945‑951.
- 96.271 The cricoid cartilage ring is located approximately at which level of the cervical
- spine?
- 1‑ C3
- 2‑ C4
- 3‑ C5
- 4‑ C6
- 5‑ C7
- Question 96.271
- Answer = 4
- Reference(s)
- The spine, in Hoppenfeld S, deBoer P (eds): Surgical Exposures in Orthopaedics: The Anatomic Approach Philadelphia, PA, JB Lippincott, 1984, pp 209‑300.
- 96.272 Spondylolisthesis at the L4‑5 level in adults is most likely secondary to what
- condition?
- 1‑ Trauma
- 2‑ Pars defect
- 3‑ Degenerative disease
- 4‑ Congenital abnormality of the pans
- 5‑ Elongation of the pars
- answer
- back
- Question 96.272
- Answer = 3
- back to this question
- Reference(s)
- Herkowitz HN, Kurtz LT: Degenerative lumbar spondylolisthesis with spinal stenosis: A prospective study comparing decompression with decompression and intertransverse process arthrodesis. J Bone Joint Stug 1991;73A:802‑808. Herkowitz HN, Sidhu KS: Lumbar spine fusion in the treatment of degenerative conditions: Current indications and recommendations. J Am Acad Orthop Surg 1995;3:123‑135.
- next question
- 96.273 A 25‑year‑old man sustains a closed injury to his wrist in a motorcycle accident.
- Radiographs are shown in Figures 78a and 78b. What are the names of the bones labelled
- A, B, C, respectively?
- 1‑ Lunate, scaphoid, and triquetrum
- 2‑ Scaphoid, lunate, and pisiform
- 3‑ Scaphoid, lunate, and triquetrum
- 4‑ Trapezium, capitate, and scaphoid
- 5‑ Radial styloid, scaphoid, and triquetrum
- Fig. 78a Fig. 78b
- Question 96.273
- Answer = 1
- Reference(s)
- Green DP: Carpal dislocations and instabilities, in Green DP (ed): Operative Hand Surgery, ed 3. New York, NY, Churchill‑Livingstone, 1993, vol 1, pp 861‑928.
- 96.274 A 62‑year‑old woman has swelling anteriorly in her right distal thigh that has
- slowly enlarged over the last 4 years, accompanied by periods of swelling and pain
- throughout the entire knee. There is no history of trauma, and the knee has never
- been aspirated. Examination reveals a knee effusion, diffuse joint line tenderness, a
- boggy soft‑tissue mass in the suprapatellar pouch, and another area of discrete
- fullness in the popliteal region. Plain radiographs of the knee are shown in Figures
- 79a and 79b, and MRI scans are shown in Figure 79c (sagittal T,‑weighted) and
- Figure 79d (axial T2 weighted). Histology is shown in Figure 79e. The most likely
- diagnosis is
- 1‑ synovial chondromatosis.
- 2‑ synovial sarcoma.
- 3‑ pigmented villonodular synovitis.
- 4‑ rheumatoid synovitis.
- 5‑ chondrocalcinosis.
- Fig. 79a Fig. 79b
- Go to next slide for remaining figures and answer link
- Fig. 79c
- Fig. 79d
- Fig. 79e
- Question 96.274
- Answer = 3
- Reference(s)
- Flandry F, Hughston JC: Current concepts review: Pigmented villonodular synovitis. J Bone Joint Surg 1987;69A:942‑949. Dorwart RH, Genant HK, Johnston WK et al: Pigmented villonodular synovitis of synovial joints: Clinical, pathologic, and radiologic features. Am J Roentgenol 1984;143:877‑885.
- next question
- 96.275 A 69‑year‑old man who is an avid golfer has a 5‑year history of wrist pain and
- weakness. A radiograph is shown in Figure 80. Recommended treatment should
- be
- 1‑ scapholunate ligament repair.
- 2‑ proximal row carpectomy.
- 3‑ scaphotrapezial trapezoid joint fusion.
- 4‑ radiocarpal arthroscopy and debridement.
- 5‑ scaphoid titanium arthroplasty.
- Figure 80
- Question 96.275
- Answer = 2
- End of 1996 Exam
- Reference(s)
- Tomaino MM, Delsignore J, Burton RI: Long‑term results following proximal row carpectomy. J Hand Surg 1994;19A:694‑703. Imbriglia JE, Broudy AS, Hagberg WC, et al: Proximal row carpectomy: Clinical evaluation. J Hand Surg 1990;15A;426‑430. Jorgenson EC: Proximal row carpectomy: An end result study of twenty‑two cases. J Bone Joint Surg 1969;51A:1104‑1111.